NajnovijeNajčitanijeNajviše porukaPostavi novu temu 

Zanimljiv matematicki problem(i)

   Kolona vojnika dugačka jedan kilometar počinje da maršira ravnomernom brzinom.Posle tačno jednog kilometra zaustavljaju se.U istom trenutku kad je kolona krenula, glasnik koji se nalazi na samom začelju, izdvaja se i počinje da se kreće ka početku, takodje ravnomernom brzinom.Njegova brzina je taman takva da uspe da stigne sam početak kolone , istovremeno se okrene i nastavi nazad ka začelju, gde stigne baš u trenutku kada se i sama kolona zaustavila.Celim putem, brzine kolone kao ni glasnika nisu se menjale.Pitanje glasi: -Koliko je glasnik prepešačio?

    Nadam se da će malo mučenja oko ovog zadatka mnogima doneti zadovoljstvo, kao i da će zadataka biti još.

34 za
1 protiv
raborg
2010-08-17
???

2 lm koliko god to neverovatno izgledalo, svejedno je da li se kolona krece ili stoji glasnik uvek prelazi dva kilometra - kilometar do pocetka kolone i kilometar nazad.Zadatak se dosta lako resava sa vektorima a najlakse je shvatiti zasto je resenje dva kilometra ako zamislimo da je referentna nepokretna tacka pocetak kolone a da se svet (okolina) pomera u suprotnom pravcu ravnomernom brzinom (onom kojom u stvari kolona marsira).

9 za
6 protiv
Meda
2010-08-17
???

Moje rešenje je: 2km i 414m.

5 za
2 protiv
Vladimir Obradović
2010-08-17
???

Moguce, ja sam ga radio po analogiji sa jednim slicnim zadatkom. Cucemo od raborga.

2 za
0 protiv
Carl Hanratty
2010-08-17
???

@ Vladimir: И још 21 центиметар... :)  То је то...

3 za
1 protiv
Цане Вукић
2010-08-17
???

Nema raborga, ko li je u pravu meda ili vladimir... Meni licno lici na neki logicki logicki zadatak iz srednje skole a ne na tesku aritmetiku.

4 za
2 protiv
Carl Hanratty
2010-08-17
???

@Rubeus: Владимир је у праву, Меда није. Када иде ка челу колоне, прелази 1.707 км, а кад се враћа  0.707 км. Није логички задатак, већ физички. ''Само'' треба поставити прави систем једначина, решити и...
 

5 za
1 protiv
Цане Вукић
2010-08-17
???

Evo kako sam došao do rešenja:

Da bi sustigao prvog u koloni, glasnik je prešao 1+x km. Za to vreme, prvi u koloni je prešao x km. Nakon okretanja, glasnik je prešao još x km do susreta sa poslednjim u koloni, koji je u tom trenutku bio udaljen 1 km od svoje (a takođe i glasnikove) polazne tačke. Za to vreme, lider je prešao preostalih 1-x km. Pošto se svi vojnici kreću ravnomernom brzinom, odnos pređenog puta glasnika i lidera pre i posle okretanja glasnika mora da bude isti, tj. (1+x) : x = x : (1-x). Rešavanjem ove proporcije dobija se da je x = 0,707 km, što znači da je glasnik ukupno prešao 2,414 km (1+2x).

8 za
2 protiv
Vladimir Obradović
2010-08-17
???

Vladimir Obradović napisa...

Moje rešenje je: 2km i 414m.

To je takodje i moje rešenje, Vladimir je u pravu.

3 za
1 protiv
veljko_j
2010-08-17
???

Imamo dva jata ptica koja se nalaze na dva susjesjedna stabla.Prvo jato se obraća drugom i kaže: "Dajte nam  n   svojih članova, pa će nas biti dva puta više nego vas !", a drugo jato odgovara "Dajte vi nama vaših  n  članova, pa će nas i vas biti isto!".Kako pronaći broj ptica u prvom, a kako u drugom jatu?

3 za
0 protiv
MaLin
2010-08-17
???

Zadatak je fizički, pa evo i jednog takvog rešenja.

Postavci zadatka bih zamerio to što ne naglašava da li je konstantna brzina kurira u odnosu na zemlju ili u odnosu na kolonu, mada je, s druge strane nekako jasno da je to brzina u odnosu na zemlju...

Neka je dužina kolone l, brzina vojnika u koloni u odnosu na zemlju v, a brzina kurira u odnosu na zemlju u. Ukupno vreme kretanja kurira i kolone je t=l/v. Kada se kurir kreće prema čelu kolone, on se, u referentnom sistemu vezanom za kolonu, kreće brzinom u-v, a u povratku ka začelju kolone brzinom u+v, a kako u tom sistemu u oba slučaja pređe put l, to je ukupno vreme kretanja kurira jednako l/(u-v)+l/(u+v). Izjednačavanjem sa t dobije se da je u2-v2=2uv, ili (u/v)-(v/u)=2. Odavde je, pošto odnos brzina pozitivan, u/v=1+sqrt(2). Predjeni put kurira je s=u*t=(u/v)*l, što približno jeste 2.414km.

2 za
1 protiv
veljko_j
2010-08-17
???

Vladimir je prvi pogodio, a tacno je da se moze resiti i fizički i matematički.Inače i ja sam ga rešio na potpuno isti i najjednostavniji način, kao i Vladimir, postavljanjem proporcije i dobio 1+sqr(2) ili 2,4142.

@Veljko_j:Ako se kolona krece konstantnom brzinom zar nije svejedno da li je brzina glasnika ravnomerna u odnosu na nju ili na zemlju.

4 za
2 protiv
raborg
2010-08-17
???

raborg napisa...

Vladimir je prvi pogodio, a tacno je da se moze resiti i fizički i matematički.Inače i ja sam ga rešio na potpuno isti i najjednostavniji način, kao i Vladimir, postavljanjem proporcije i dobio 1+sqr(2) ili 2,4142.

@Veljko_j:Ako se kolona krece konstantnom brzinom zar nije svejedno da li je brzina glasnika ravnomerna u odnosu na nju ili na zemlju.

Ravnomerna u onom najbezazlenijem smislu da je stalno ista, konstantna? Ako je u tom smislu, onda je svejedno... Ali bi ipak trebalo to naglasiti. Isto tako mislim da je bolje reći da je kretanje ravnomerno, a da je brzina kod takvog kretanja konstantna, a ne ravnomerna. Da je isticanje referentnog sistema u odnosu na koji je zadata brzina bitno najbolje se vidi kod zadataka gde je dat čamac koji se kreće po reci-tu bi uvek moralo biti rečeno u odnosu na šta se odnosi brzina čamca eventualno data u zadatku.

Naravno, ova primedba je samo tehničkog karaktera. Pretpostavljam da se kurir kreće na nekom motoru, pa je, naravno, potpuno razložno zaključiti da je data brzina motora (u odnosu na zemlju)...

2 za
0 protiv
veljko_j
2010-08-17
???

elnesone napisa...

Imamo dva jata ptica koja se nalaze na dva susjesjedna stabla.Prvo jato se obraća drugom i kaže: "Dajte nam  n   svojih članova, pa će nas biti dva puta više nego vas !", a drugo jato odgovara "Dajte vi nama vaših  n  članova, pa će nas i vas biti isto!".Kako pronaći broj ptica u prvom, a kako u drugom jatu?

...

Prvo jato ima 7n, a drugo 5n ptica.

3 za
1 protiv
Vladimir Obradović
2010-08-17
???

elnesone napisa...

Imamo dva jata ptica koja se nalaze na dva susjesjedna stabla.Prvo jato se obraća drugom i kaže: "Dajte nam  n   svojih članova, pa će nas biti dva puta više nego vas !", a drugo jato odgovara "Dajte vi nama vaših  n  članova, pa će nas i vas biti isto!".Kako pronaći broj ptica u prvom, a kako u drugom jatu?

...Resavanjem sistema jednacina dolazimo do 5x=7y gde su x i y brojevi ptica u jatima, tako da je resenje 7n i 5n

2 za
0 protiv
raborg
2010-08-17
???

Sad vidim da me je Vladimir preduhitrio

0 za
0 protiv
raborg
2010-08-17
???

Preporuka: ako budete postavljali nove probleme u ovoj temi, obojite tekst nekom bojom, da bude uočljivije. (ili otvorite novu temu).

Naravno, odgovore i komentare i dalje pišite standardnom plavom bojom, da ne bi tema postala totalno nepregledna.

2 za
1 protiv
Supervizor
2010-08-17
???

Imamo dvije vrste bombona. Kilogram prve košta 0,75 KM  a kilogram druge 1,25 KM.

Zadatak:

Od ove dvije vrste napraviti kilogram bombona koji će da košta 0,96 KM.

5 za
0 protiv
nobody
2010-08-17
???

@Raborg, Vladimir: mozda gresim, ali mislim da je postavka drugacija nego sto ste pretpostavili, u suprotnom ne vidim zasto bi Elnesone prvi put ispisao n   , a drugi put     n. Mislim da se broj ptica koji treba da predje iz jednog jata u drugo i obrnuto razlikuje, tj. da iz prvog jata (A) u drugo jato (B) treba da predje nm ptica da bi bilo A=2*B, odnosno da iz B u A treba da predje mn ptica da bi bilo zadovoljeno da je B=A. U tom slucaju, postavljam prostog sistema bi se dobilo mogucih 81 resenje, gde bi bilo A=43*m+34n, B=23m+32n.

0 za
2 protiv
Kalimist
2010-08-17
???

@Kalimist ,

ljudi su dobro riješili zadatak, možda sam nehotice različito podvukao n, jer koliko je prvo jato tražilo od drugog, toliko isto je drugo tražilo od prvog, iz same prirode zadatka nisam naveo da n pripada skupu prirodnih brojeva.E sad evo novog.

Umjesto slova stavite brojeve ali da jednakost bude zadovoljena

                                 SLOV,O

                           +    SLOV,O

                          =  PESMA

 

3 za
0 protiv
MaLin
2010-08-17
???

E da, slovo A treba da dodje ispod V tj ima vrijednost jedinica, M ispod O tj. desetica itd.

4 za
0 protiv
MaLin
2010-08-17
???

@Nobody: 0.58 kg i 0.42 kg.

@elnesone: Eto, nehotice greska, a meni se ucinilo da je znacajna :)

Drugi zadatak 9453.5*2=18907.
 

2 za
0 protiv
Kalimist
2010-08-17
???

Evo jedan i od mene, geometrija je u pitanju. Pretpostavite da postoji kvadar ABCDA1B1C1D1, gde su A, B, C, D temena pravougaonika u donjoj osnovi, a A1, B1, C1, D1 temena pravougraonika u gornjoj osnovi. Poznato je da je povrsina pravougaonika ABCD jednaka P, da je visina kvadra jednaka H, i da je povrsina pravougaonika ACC1A1 jednaka M. Odrediti obim pravougaonika ABCD, bez upotrebe kvadratnih jednacina ( zadatak je za prvi razred srednje skole ).

2 za
0 protiv
Kalimist
2010-08-17
???

   Gospodin i gospodja Smit (mozda čak lepa Andja i Bred) su priredili zabavu na koju je pozvano još četiri para, tako da kad su svi stigli bilo je ukupno deset ljudi u kući.Neki su se poznavali od ranije a neki ne.Rukovali su se samo oni koji se nisu poznavali.Onda je gospodin Smit pitao sve njih , uključujući i svoju ženu, koliko puta su se rukovali i svi odgovori su bili različiti.Koliko puta se rukovao gospodin Smit?

3 za
0 protiv
raborg
2010-08-17
???

Kalimist napisa...

Evo jedan i od mene, geometrija je u pitanju. Pretpostavite da postoji kvadar ABCDA1B1C1D1, gde su A, B, C, D temena pravougaonika u donjoj osnovi, a A1, B1, C1, D1 temena pravougraonika u gornjoj osnovi. Poznato je da je povrsina pravougaonika ABCD jednaka P, da je visina kvadra jednaka H, i da je povrsina pravougaonika ACC1A1 jednaka M. Odrediti obim pravougaonika ABCD, bez upotrebe kvadratnih jednacina ( zadatak je za prvi razred srednje skole ).

    O=2sqrt((M/H)2 +2P)

0 za
0 protiv
raborg
2010-08-17
???

Mozda bi bilo dobro kada vidite da vec ima zuta, da za nova pitanja stavite svetlo zelenu, svetlo plavu, sivu, svetlo crvenu itd. da se lakse prepozna koji je odgovor na koje pitanje. smiley

3 za
0 protiv
Supervizor
2010-08-17
???

Evo jedan problem,nije puno tezak:

Grupi od 100 zatvorenika preti smrtna kazna.Upravnik zatvora dolazi dan pred izvrsenje smrtne kazne i nudi im mogucnost sa se spasu.Sutradan ce svi zarobljenici biti postrojeni u kolonu i na glavu svakog od njih ce biti stavljena kapa crne ili bele boje.Svaki zatvorenik ne moze videti boju svoje kape,kao ni boje kapa zatvorenika koji stoje iza njega,ali moze videti boje kapa na glavama svih zatvorenika ispred njega u koloni.Svaki zatvorenik moci ce da kaze samo jednu rec bela ili crna.Ukoliko se ta rec poklopi sa bojom kape na njegovoj glavi,bice spasen,u suprotnom mu predstoji smrtna kazna.Zatvorenici imaju vece da se dogovore i osmisle eventualnu strategiju kako bi ih se sto vise spasilo.Koliko najvise  zatvorenika se sigurno moze spasiti od smrtne kazne?

3 za
0 protiv
mc80
2010-08-17
???

Gosn. Smit se rukovao 4 puta. cool

3 za
0 protiv
Petar Mirković
2010-08-17
???

mc80 napisa...

Evo jedan problem,nije puno tezak:

Grupi od 100 zatvorenika preti smrtna kazna.Upravnik zatvora dolazi dan pred izvrsenje smrtne kazne i nudi im mogucnost sa se spasu.Sutradan ce svi zarobljenici biti postrojeni u kolonu i na glavu svakog od njih ce biti stavljena kapa crne ili bele boje.Svaki zatvorenik ne moze videti boju svoje kape,kao ni boje kapa zatvorenika koji stoje iza njega,ali moze videti boje kapa na glavama svih zatvorenika ispred njega u koloni.Svaki zatvorenik moci ce da kaze samo jednu rec bela ili crna.Ukoliko se ta rec poklopi sa bojom kape na njegovoj glavi,bice spasen,u suprotnom mu predstoji smrtna kazna.Zatvorenici imaju vece da se dogovore i osmisle eventualnu strategiju kako bi ih se sto vise spasilo.Koliko najvise  zatvorenika se sigurno moze spasiti od smrtne kazne?

...moći će da kaže samo jednu riječ, to je u redu, ali da li smije da daje znakove rukom, recimo da spusti ruku na rame zatvorenika ispred sebe, možda bi tako mogli svi da se spasu, kad primjeti prvog čija se kapa razlikuje od ostalih, spusti ruku na njegovo rame, taj na čije rame je spuštna ruka izgovara suprotnu boju od onih koji su ispred njega, a svi ispred njega govore suprotno od onog što je on rekao, mislim ja ovo lupam bez veze, jer ispitivati nešto ne uključuje samo čulo vida, nego i mirisa i dodira.

0 za
0 protiv
nobody
2010-08-17
???

nobody napisa...


...moći će da kaže samo jednu riječ, to je u redu, ali da li smije da daje znakove rukom, recimo da spusti ruku na rame zatvorenika ispred sebe, možda bi tako mogli svi da se spasu, kad primjeti prvog čija se kapa razlikuje od ostalih, spusti ruku na njegovo rame, taj na čije rame je spuštna ruka izgovara suprotnu boju od onih koji su ispred njega, a svi ispred njega govore suprotno od onog što je on rekao, mislim ja ovo lupam bez veze, jer ispitivati nešto ne uključuje samo čulo vida, nego i mirisa i dodira.

Nista od toga ne sme,nikakvi pokreti i sl.Sve sto smeju je da izgovore tu jednu,jedinu rec!

0 za
0 protiv
mc80
2010-08-17
???

Petar Mirković napisa...

Gosn. Smit se rukovao 4 puta. cool

...Bravo Petre

1 za
0 protiv
raborg
2010-08-17
???

mc80 napisa...

Evo jedan problem,nije puno tezak:

Grupi od 100 zatvorenika preti smrtna kazna.Upravnik zatvora dolazi dan pred izvrsenje smrtne kazne i nudi im mogucnost sa se spasu.Sutradan ce svi zarobljenici biti postrojeni u kolonu i na glavu svakog od njih ce biti stavljena kapa crne ili bele boje.Svaki zatvorenik ne moze videti boju svoje kape,kao ni boje kapa zatvorenika koji stoje iza njega,ali moze videti boje kapa na glavama svih zatvorenika ispred njega u koloni.Svaki zatvorenik moci ce da kaze samo jednu rec bela ili crna.Ukoliko se ta rec poklopi sa bojom kape na njegovoj glavi,bice spasen,u suprotnom mu predstoji smrtna kazna.Zatvorenici imaju vece da se dogovore i osmisle eventualnu strategiju kako bi ih se sto vise spasilo.Koliko najvise  zatvorenika se sigurno moze spasiti od smrtne kazne?

...Da li mogu da biraju ko ce prvi da odgovara, jer ako krenu od poslednjeg ka prvom moze se sigurno spasiti njih 99

0 za
0 protiv
raborg
2010-08-17
???

Naravno!Siguran sam da si onda dobro uradio cim me to pitas,eto samo jos postavi resenje! :)

0 za
0 protiv
mc80
2010-08-17
???

mc80 napisa...

Naravno!Siguran sam da si onda dobro uradio cim me to pitas,eto samo jos postavi resenje! :)

...Prvo se dogovore da onaj koji (na njegovu zalost ) bude na zacelju prebroji npr. crne sesire i ako broj bude neparan da kaze "crni" u suprotnom "beli".On ce imati 50% da prezivi.Zatim onaj ispred njega takodje prebroji crne sesire medju preostalih 98 i ako im je broj i dalje neparan znaci da je na njegovoj glavi beli, sto ce i reci, u suprotnom ce reci "crni".Svaki sledeci, pazljivo saslusavsi sta izgovara prethodnik i koristeci istu taktiku, ce se izvuci.

1 za
0 protiv
raborg
2010-08-17
???

raborg napisa...

mc80 napisa...

Naravno!Siguran sam da si onda dobro uradio cim me to pitas,eto samo jos postavi resenje! :)

...Prvo se dogovore da onaj koji (na njegovu zalost ) bude na zacelju prebroji npr. crne sesire i ako broj bude neparan da kaze "crni" u suprotnom "beli".On ce imati 50% da prezivi.Zatim onaj ispred njega takodje prebroji crne sesire medju preostalih 98 i ako im je broj i dalje neparan znaci da je na njegovoj glavi beli, sto ce i reci, u suprotnom ce reci "crni".Svaki sledeci, pazljivo saslusavsi sta izgovara prethodnik i koristeci istu taktiku, ce se izvuci.

Da,da to je tacno!Svaka cast!

0 za
0 protiv
mc80
2010-08-17
???

   Zamislite kruzni trg oko kojeg je poredjano 10 kuca.Kuće su redom(bilo kojim) numerisane brojevima od 0 do 9.Postoji izvesno pravilo o broju osoba koje zive u tim kucama.Pravilo kaze:broj osoba koje zive u odredjenoj kuci jednak je broju kuca u kojima zivi tacno toliko osoba, koliki je broj date kuce.Npr. ako 5 osoba zivi u kuci broj 2, tada mora biti tacno 5 kuca sa po dve osobe u svakoj.Pitanje je ,naravno, koliko osoba zivi u svakoj od kuca, ponaosob.

0 za
0 protiv
raborg
2010-08-18
???

raborg napisa...

   Zamislite kruzni trg oko kojeg je poredjano 10 kuca.Kuće su redom(bilo kojim) numerisane brojevima od 0 do 9.Postoji izvesno pravilo o broju osoba koje zive u tim kucama.Pravilo kaze:broj osoba koje zive u odredjenoj kuci jednak je broju kuca u kojima zivi tacno toliko osoba, koliki je broj date kuce.Npr. ako 5 osoba zivi u kuci broj 2, tada mora biti tacno 5 kuca sa po dve osobe u svakoj.Pitanje je ,naravno, koliko osoba zivi u svakoj od kuca, ponaosob.

Da nije mozda ovo resenje-u kuci broj nula zive 8 osoba,u kuci broj 1 zivi jedna osoba u ostalim kucama 2,3,4,5,6,7,8,9 zive nula osoba tj. ne zivi niko.

0 za
0 protiv
Слађан
2010-08-18
???

SLAKI82 napisa...

raborg napisa...

   Zamislite kruzni trg oko kojeg je poredjano 10 kuca.Kuće su redom(bilo kojim) numerisane brojevima od 0 do 9.Postoji izvesno pravilo o broju osoba koje zive u tim kucama.Pravilo kaze:broj osoba koje zive u odredjenoj kuci jednak je broju kuca u kojima zivi tacno toliko osoba, koliki je broj date kuce.Npr. ako 5 osoba zivi u kuci broj 2, tada mora biti tacno 5 kuca sa po dve osobe u svakoj.Pitanje je ,naravno, koliko osoba zivi u svakoj od kuca, ponaosob.

Da nije mozda ovo resenje-u kuci broj nula zive 8 osoba,u kuci broj 1 zivi jedna osoba u ostalim kucama 2,3,4,5,6,7,8,9 zive nula osoba tj. ne zivi niko.

...Ako u kuci 8 nema nikog, onde nema kuca u kojoj zive osam osoba

0 za
0 protiv
raborg
2010-08-18
???

raborg napisa...

SLAKI82 napisa...

raborg napisa...

   Zamislite kruzni trg oko kojeg je poredjano 10 kuca.Kuće su redom(bilo kojim) numerisane brojevima od 0 do 9.Postoji izvesno pravilo o broju osoba koje zive u tim kucama.Pravilo kaze:broj osoba koje zive u odredjenoj kuci jednak je broju kuca u kojima zivi tacno toliko osoba, koliki je broj date kuce.Npr. ako 5 osoba zivi u kuci broj 2, tada mora biti tacno 5 kuca sa po dve osobe u svakoj.Pitanje je ,naravno, koliko osoba zivi u svakoj od kuca, ponaosob.

Da nije mozda ovo resenje-u kuci broj nula zive 8 osoba,u kuci broj 1 zivi jedna osoba u ostalim kucama 2,3,4,5,6,7,8,9 zive nula osoba tj. ne zivi niko.

...Ako u kuci 8 nema nikog, onde nema kuca u kojoj zive osam osoba

onda da preformulisem odgovor usled mog previda ali mislim da sam bio na dobrom putu-u kuci broj nula zive 8 osoba a u kuci broj 8 zivi jedna osoba u ostalim kucama zive nula osoba,mislim da ovako moze

0 za
0 protiv
Слађан
2010-08-18
???

SLAKI82 napisa...

raborg napisa...

SLAKI82 napisa...

raborg napisa...

   Zamislite kruzni trg oko kojeg je poredjano 10 kuca.Kuće su redom(bilo kojim) numerisane brojevima od 0 do 9.Postoji izvesno pravilo o broju osoba koje zive u tim kucama.Pravilo kaze:broj osoba koje zive u odredjenoj kuci jednak je broju kuca u kojima zivi tacno toliko osoba, koliki je broj date kuce.Npr. ako 5 osoba zivi u kuci broj 2, tada mora biti tacno 5 kuca sa po dve osobe u svakoj.Pitanje je ,naravno, koliko osoba zivi u svakoj od kuca, ponaosob.

Da nije mozda ovo resenje-u kuci broj nula zive 8 osoba,u kuci broj 1 zivi jedna osoba u ostalim kucama 2,3,4,5,6,7,8,9 zive nula osoba tj. ne zivi niko.

...Ako u kuci 8 nema nikog, onde nema kuca u kojoj zive osam osoba

onda da preformulisem odgovor usled mog previda ali mislim da sam bio na dobrom putu-u kuci broj nula zive 8 osoba a u kuci broj 8 zivi jedna osoba u ostalim kucama zive nula osoba,mislim da ovako moze

...pa slicno kao i prosli put, ako u kuci 1 nema nikog , onda nema kuca u kojoj zivi jedna osoba

0 za
0 protiv
raborg
2010-08-18
???

raborg napisa...

SLAKI82 napisa...

raborg napisa...

SLAKI82 napisa...

raborg napisa...

   Zamislite kruzni trg oko kojeg je poredjano 10 kuca.Kuće su redom(bilo kojim) numerisane brojevima od 0 do 9.Postoji izvesno pravilo o broju osoba koje zive u tim kucama.Pravilo kaze:broj osoba koje zive u odredjenoj kuci jednak je broju kuca u kojima zivi tacno toliko osoba, koliki je broj date kuce.Npr. ako 5 osoba zivi u kuci broj 2, tada mora biti tacno 5 kuca sa po dve osobe u svakoj.Pitanje je ,naravno, koliko osoba zivi u svakoj od kuca, ponaosob.

Da nije mozda ovo resenje-u kuci broj nula zive 8 osoba,u kuci broj 1 zivi jedna osoba u ostalim kucama 2,3,4,5,6,7,8,9 zive nula osoba tj. ne zivi niko.

...Ako u kuci 8 nema nikog, onde nema kuca u kojoj zive osam osoba

onda da preformulisem odgovor usled mog previda ali mislim da sam bio na dobrom putu-u kuci broj nula zive 8 osoba a u kuci broj 8 zivi jedna osoba u ostalim kucama zive nula osoba,mislim da ovako moze

...pa slicno kao i prosli put, ako u kuci 1 nema nikog , onda nema kuca u kojoj zivi jedna osoba

...ocigledno me ne sluzi koncentracija u ove kasne sate ali probacu jos jednom makar se totalno obrukao smiley 

dakle resenje je:u kuci broj 0 zive 6 osoba,u kuci broj 1 zive 2 osobe,u kuci broj 2 zivi 1 osoba ,u kuci broj 6 zivi 1 osoba u kucama 3,4,5,7,8,9 zivi 0 osoba,

mislim da sam ovog puta dobro izmozgao i da ovaj put funkcionise racunicayes

1 za
0 protiv
Слађан
2010-08-18
???

SLAKI82 napisa...

raborg napisa...

SLAKI82 napisa...

raborg napisa...

SLAKI82 napisa...

raborg napisa...

   Zamislite kruzni trg oko kojeg je poredjano 10 kuca.Kuće su redom(bilo kojim) numerisane brojevima od 0 do 9.Postoji izvesno pravilo o broju osoba koje zive u tim kucama.Pravilo kaze:broj osoba koje zive u odredjenoj kuci jednak je broju kuca u kojima zivi tacno toliko osoba, koliki je broj date kuce.Npr. ako 5 osoba zivi u kuci broj 2, tada mora biti tacno 5 kuca sa po dve osobe u svakoj.Pitanje je ,naravno, koliko osoba zivi u svakoj od kuca, ponaosob.

Da nije mozda ovo resenje-u kuci broj nula zive 8 osoba,u kuci broj 1 zivi jedna osoba u ostalim kucama 2,3,4,5,6,7,8,9 zive nula osoba tj. ne zivi niko.

...Ako u kuci 8 nema nikog, onde nema kuca u kojoj zive osam osoba

onda da preformulisem odgovor usled mog previda ali mislim da sam bio na dobrom putu-u kuci broj nula zive 8 osoba a u kuci broj 8 zivi jedna osoba u ostalim kucama zive nula osoba,mislim da ovako moze

...pa slicno kao i prosli put, ako u kuci 1 nema nikog , onda nema kuca u kojoj zivi jedna osoba

...ocigledno me ne sluzi koncentracija u ove kasne sate ali probacu jos jednom makar se totalno obrukao smiley

dakle resenje je:u kuci broj 0 zive 6 osoba,u kuci broj 1 zive 2 osobe,u kuci broj 2 zivi 1 osoba ,u kuci broj 6 zivi 1 osoba u kucama 3,4,5,7,8,9 zivi 0 osoba,

mislim da sam ovog puta dobro izmozgao i da ovaj put funkcionise racunicayes

...E to je to, bravo

2 za
0 protiv
raborg
2010-08-18
???

SLAKI82 napisa...

dakle resenje je:u kuci broj 0 zive 6 osoba,u kuci broj 1 zive 2 osobe,u kuci broj 2 zivi 1 osoba ,u kuci broj 6 zivi 1 osoba u kucama 3,4,5,7,8,9 zivi 0 osoba,

mislim da sam ovog puta dobro izmozgao i da ovaj put funkcionise racunicayes

... super. Ajde sad isto to, ali sa 1000 kuća devil

2 za
0 protiv
Supervizor
2010-08-18
???

Supervizor napisa...

SLAKI82 napisa...

dakle resenje je:u kuci broj 0 zive 6 osoba,u kuci broj 1 zive 2 osobe,u kuci broj 2 zivi 1 osoba ,u kuci broj 6 zivi 1 osoba u kucama 3,4,5,7,8,9 zivi 0 osoba,

mislim da sam ovog puta dobro izmozgao i da ovaj put funkcionise racunicayes

... super. Ajde sad isto to, ali sa 1000 kuća devil

pa isto je  i sa 1000 kuca samo sto kuca sa brojem nula mora da bude toliko velika da u njoj stane 996 osobe onda bi 996. kuca imala jednu osobu,kuca broj 2  jednu osobu i kuca broj 1 imala bi 2 osobe-(tako da bi jedini problem bio smestiti 996 osobe u kucu broj 0)

laugh

2 za
0 protiv
Слађан
2010-08-18
???

 

   A evo sad jedan koji nisam resio.

   Na raspolaganju je sedam tegova i vaga sa dva tasa.Teret se meri s maksimalno tri tega i to tako da se oni mogu stavljati na bilo koju stranu vage, ili na onu gde je i sam teret ili na suprotnu.Cilj je pronaci tezine ovih sedam tegova(celi brojevi), tako da se moze izmeriti svaka tezina pocevsi od 1kg pa preko svakog sledeceg celog broja do najveceg moguceg.

    Ja sam stigao do 106kg ali znam da to nije najveci moguci broj, a ne znam ni koji je konacan.

1 za
0 protiv
raborg
2010-08-20
???

   E , da, mojih sedam brojeva su : 1, 2, 3, 9, 16, 48, 87

0 za
0 protiv
raborg
2010-08-20
???

Evo jednog interesantnog zadatka,koga nisam uspio da riješim...Već počinjem da se pitam ima li uopšte rješenja?!?

Nalazite se u sobi sa 3 prekidača. U drugoj sobi se nalaze 3 sijalice. Zna se da
svaki prekidač pali tačno jednu sijalicu, i da je svaku sijalicu moguće upaliti nekim
 prekidačem. Smijete da iskoristite prekidače, a zatim samo jednom da uđete u sobu
sa sijalicama, i odredite koji prekidač pali koju.

1 za
0 protiv
limes5
2010-08-24
???

Caka je u tome , što se ne možeš osloniti samo na čulo vida, tj. moraš koristiti i čulo dodira.

Evo kao, pritisneš jedan prekidač a zatim sačekaš nekoliko minuta, a zatim pritisneš drugi prekidač i uđeš u sobu, zatim dodirneš uključene sijalice, ona koja je topla, nju je uključio prvi prekidač, a ona koja je hladna, nju je uključio drugi prekidač, ona što nije uključena reguliše se trećim prekidačem.

3 za
0 protiv
MaLin
2010-08-24
???

 

elnesone napisa...

Caka je u tome , što se ne možeš osloniti samo na čulo vida, tj. moraš koristiti i čulo dodira.

Evo kao, pritisneš jedan prekidač a zatim sačekaš nekoliko minuta, a zatim pritisneš drugi prekidač i uđeš u sobu, zatim dodirneš uključene sijalice, ona koja je topla, nju je uključio prvi prekidač, a ona koja je hladna, nju je uključio drugi prekidač, ona što nije uključena reguliše se trećim prekidačem.

...Aha! Svaka čast! O svemu sam razmišljao u vezi ovog zadatka ali mi to nije palo na pamet! Koncentrisao sam se da riješim problem čulom vida,ali to je izgleda nemoguće.
Još jedom,bravo!

1 za
0 protiv
limes5
2010-08-24
???

evo jednog sličnog zadatka:

 

     Bila jednom jedna jako lepa princeza. Doslo je vreme da se uda i njen otac, kralj, je dugo razmisljao kome da je da. Nakon pazljive selekcije, ostala su mu tri favorita: Osvaldo, Toskano i Ferdinand. 

     Jednog letnjeg, vrlo suncanog dana, otac odluci da pozove sva tri princa i stavi ih na test - ko resi test dobija princezu. Svim princevima su vezane oci, i dato im je 10 minuta za pronalazenje resenja.

      Osvaldo je postavljen ispred dvorca i kralj mu je rekao: "Moj dvorac ima 10 vrata, sva su bela osim jednih. Moras da pronadjes crna vrata."
      Toskano je odveden u bastu gde mu je kralj rekao: "U mom vrtu ima 10 redova ruza, u svakom redu su crvene osim u jednom. Moras da nadjes red plavih."
      Ferdinand je odveden u salu za balove, postavljen je ispred kraljevog stola i kralj mu je rekao: "Ovde ima svi tanjiri su srebrni, osim jednog. Pokazi mi koji je zlatni".
     Uprkos tome sto su im bile vezane oci, jedan princ je uspeo da resi test. Koji i kako?

1 za
0 protiv
Слађан
2010-08-24
???

SLAKI82 napisa...

evo jednog sličnog zadatka:

 

     Bila jednom jedna jako lepa princeza. Doslo je vreme da se uda i njen otac, kralj, je dugo razmisljao kome da je da. Nakon pazljive selekcije, ostala su mu tri favorita: Osvaldo, Toskano i Ferdinand. 

     Jednog letnjeg, vrlo suncanog dana, otac odluci da pozove sva tri princa i stavi ih na test - ko resi test dobija princezu. Svim princevima su vezane oci, i dato im je 10 minuta za pronalazenje resenja.

      Osvaldo je postavljen ispred dvorca i kralj mu je rekao: "Moj dvorac ima 10 vrata, sva su bela osim jednih. Moras da pronadjes crna vrata."
      Toskano je odveden u bastu gde mu je kralj rekao: "U mom vrtu ima 10 redova ruza, u svakom redu su crvene osim u jednom. Moras da nadjes red plavih."
      Ferdinand je odveden u salu za balove, postavljen je ispred kraljevog stola i kralj mu je rekao: "Ovde ima svi tanjiri su srebrni, osim jednog. Pokazi mi koji je zlatni".
     Uprkos tome sto su im bile vezane oci, jedan princ je uspeo da resi test. Koji i kako?

Nisam mogao odmah da resim,a onda sam uocio obojeni detalj u zadatku,i onda je lako.Posto je dan bio vrlo sucan,crna vrata ce biti toplija i tako ce ih prepoznati(pod uslovom da druga dva nisu imala srece da cistom srecom pogode,ruze odnosno tanjir)

3 za
0 protiv
mc80
2010-08-24
???

mc80 napisa...

SLAKI82 napisa...

evo jednog sličnog zadatka:

 

     Bila jednom jedna jako lepa princeza. Doslo je vreme da se uda i njen otac, kralj, je dugo razmisljao kome da je da. Nakon pazljive selekcije, ostala su mu tri favorita: Osvaldo, Toskano i Ferdinand. 

     Jednog letnjeg, vrlo suncanog dana, otac odluci da pozove sva tri princa i stavi ih na test - ko resi test dobija princezu. Svim princevima su vezane oci, i dato im je 10 minuta za pronalazenje resenja.

      Osvaldo je postavljen ispred dvorca i kralj mu je rekao: "Moj dvorac ima 10 vrata, sva su bela osim jednih. Moras da pronadjes crna vrata."
      Toskano je odveden u bastu gde mu je kralj rekao: "U mom vrtu ima 10 redova ruza, u svakom redu su crvene osim u jednom. Moras da nadjes red plavih."
      Ferdinand je odveden u salu za balove, postavljen je ispred kraljevog stola i kralj mu je rekao: "Ovde ima svi tanjiri su srebrni, osim jednog. Pokazi mi koji je zlatni".
     Uprkos tome sto su im bile vezane oci, jedan princ je uspeo da resi test. Koji i kako?

Nisam mogao odmah da resim,a onda sam uocio obojeni detalj u zadatku,i onda je lako.Posto je dan bio vrlo sucan,crna vrata ce biti toplija i tako ce ih prepoznati(pod uslovom da druga dva nisu imala srece da cistom srecom pogode,ruze odnosno tanjir)

...Da tacan je odgovor,do resenja se dolazi slicno kao i kod problema sa sijalicama.

2 za
0 protiv
Слађан
2010-08-24
???

1.zadatak

Date su tri vreće. Svaka sadrži isti broj zlatnika. U dvije vreće su samo pravi zlatnici koji su težine     7.0 g svaki a u jednoj vreći su samo lažni zlatnici težine 7.5 g svaki.  Imamo samo jednu vagu za mjerenje. Koliko mjerenja minimalno treba obaviti da se utvrdi u kojoj vreći su lažni zlatnici.

 

2.zadatak

U zelenoj tegli se nalazi 200 zelenih klikera, a u crvenoj tegli 200 crvenih klikera.  Iz  zelene tegle vadimo 30 klikera i stavljamo u crvenu teglu i dobro ih izmješamo.  Sa velikom kašikom u koju staje 30 klikera izvadimo 30 klikera iz crvene tegle i vratimo u zelenu teglu.

Koja tegla će imati više klikera druge boje?

zelena  tegla   crvenih             ili

crvena  tegla   zelenih

1 za
0 protiv
MaLin
2010-08-24
???

elnesone napisa...

1.zadatak

Date su tri vreće. Svaka sadrži isti broj zlatnika. U dvije vreće su samo pravi zlatnici koji su težine     7.0 g svaki a u jednoj vreći su samo lažni zlatnici težine 7.5 g svaki.  Imamo samo jednu vagu za mjerenje. Koliko mjerenja minimalno treba obaviti da se utvrdi u kojoj vreći su lažni zlatnici.

 

2.zadatak

U zelenoj tegli se nalazi 200 zelenih klikera, a u crvenoj tegli 200 crvenih klikera.  Iz  zelene tegle vadimo 30 klikera i stavljamo u crvenu teglu i dobro ih izmješamo.  Sa velikom kašikom u koju staje 30 klikera izvadimo 30 klikera iz crvene tegle i vratimo u zelenu teglu.

Koja tegla će imati više klikera druge boje?

zelena  tegla   crvenih             ili

crvena  tegla   zelenih

...Meni se cini da ce  uvek imati isto koliko god jednih ili drugih da zagrabe 

0 za
0 protiv
dejan86
2010-08-24
???

elnesone napisa...

 

2.zadatak

U zelenoj tegli se nalazi 200 zelenih klikera, a u crvenoj tegli 200 crvenih klikera.  Iz  zelene tegle vadimo 30 klikera i stavljamo u crvenu teglu i dobro ih izmješamo.  Sa velikom kašikom u koju staje 30 klikera izvadimo 30 klikera iz crvene tegle i vratimo u zelenu teglu.

Koja tegla će imati više klikera druge boje?

zelena  tegla   crvenih             ili

crvena  tegla   zelenih

...obe ce imati isto slazem se sa predhodnim odgovorom!

0 za
0 protiv
Слађан
2010-08-24
???

elnesone napisa...

1.zadatak

Date su tri vreće. Svaka sadrži isti broj zlatnika. U dvije vreće su samo pravi zlatnici koji su težine     7.0 g svaki a u jednoj vreći su samo lažni zlatnici težine 7.5 g svaki.  Imamo samo jednu vagu za mjerenje. Koliko mjerenja minimalno treba obaviti da se utvrdi u kojoj vreći su lažni zlatnici.

 

2.zadatak

U zelenoj tegli se nalazi 200 zelenih klikera, a u crvenoj tegli 200 crvenih klikera.  Iz  zelene tegle vadimo 30 klikera i stavljamo u crvenu teglu i dobro ih izmješamo.  Sa velikom kašikom u koju staje 30 klikera izvadimo 30 klikera iz crvene tegle i vratimo u zelenu teglu.

Koja tegla će imati više klikera druge boje?

zelena  tegla   crvenih             ili

crvena  tegla   zelenih

...zadatak 1

Dovoljno je jedno merenje.. Ako imamo vagu sa dva tasa, onda uzmemo po jedan zlatnik iz svake kese, dva stavimo na po jedan tas. Ako je vaga u ravnotezi preostali zlatnik je lazan, a ako vaga nije u ravnotezi, tezi je iz kese sa laznim zlatnicima.. A ako imamo vagu koja meri tacnu tezinu, iz prve kese uzmemo jedan zlatnik, iz druge dva, iz trece tri.. Da su sve pravi zlatnici, tezina bi bila 42g, ali kako nisu bice teze.Ako je 42.5 lazni su u prvoj kesi, 43g- u drugoj, 43.5 u trecoj

..zadatak 2

odokativnim racunom, najveca verovatnoca je da ce biti isto kugli druge boje, koliko crvenih medju zelenim, toliko i zelenih medju crvenim.. al nisam se mnogo ugubila u zadatak

1 za
0 protiv
modesty_blaise
2010-08-24
???

Evo jedan zadatak slican onom sa  kozom vukom i kupusom koje treba prevesti preko reke u camcu

Na obali reke se nalaze otac i majka sa dve cerke i  dva sina i policajac sa zatvorenikom.

Potrebno ih je sve prebaciti camcem u koju staju samo 2 osobe na drugu stranu .

Pritom:

-Camcem mogu da upravljaju samo otac majka i policajac .

-otac ne sme da ostane nasamo sa cerkama ako nije majka prisutna( neznam zasto al takvo je      pravilo, mozda da nedaj boze ne naskoci na cerke).

Isto tako ni majka ne sme da ostane nasamo sa sinovima ako otac nije tu.

- zatvorenik ne sme da ostane sam sa ostalima ako nije policajac sa njim ali sme da ostane sam na drugoj strani, nece pobeci(valjda zato sto pandur ima pistolj)

1 za
0 protiv
dejan86
2010-08-24
???

dejan86 napisa...

Evo jedan zadatak slican onom sa  kozom vukom i kupusom koje treba prevesti preko reke u camcu

Na obali reke se nalaze otac i majka sa dve cerke i  dva sina i policajac sa zatvorenikom.

Potrebno ih je sve prebaciti camcem u koju staju samo 2 osobe na drugu stranu .

Pritom:

-Camcem mogu da upravljaju samo otac majka i policajac .

-otac ne sme da ostane nasamo sa cerkama ako nije majka prisutna( neznam zasto al takvo je      pravilo, mozda da nedaj boze ne naskoci na cerke).

Isto tako ni majka ne sme da ostane nasamo sa sinovima ako otac nije tu.

- zatvorenik ne sme da ostane sam sa ostalima ako nije policajac sa njim ali sme da ostane sam na drugoj strani, nece pobeci(valjda zato sto pandur ima pistolj)

...1.policajac i zatvorenik

2.vraca se policajac

3.policajac i sin

4 .vraca se policajac i zatvorenik

5.otac i 2. sin prelaze

6. vraca se otac

7.otac i majka prelaze

8.vraca se majka

9.zatvorenik i policajac prelaze

10.vraca se otac

11.otac i majka prelaze

12. vraca se majka

13.majka i  i cerka prelaze

14.vraca se policajac i zatvorenik

15.policajac i 2. cerka prelaze(ostaje zatvorenik)

16.vraca se policajac

17.policajac i zatvorenik prelaze

           svi su presli

3 za
0 protiv
Слађан
2010-08-24
???

SLAKI82 napisa...

dejan86 napisa...

Evo jedan zadatak slican onom sa  kozom vukom i kupusom koje treba prevesti preko reke u camcu

Na obali reke se nalaze otac i majka sa dve cerke i  dva sina i policajac sa zatvorenikom.

Potrebno ih je sve prebaciti camcem u koju staju samo 2 osobe na drugu stranu .

Pritom:

-Camcem mogu da upravljaju samo otac majka i policajac .

-otac ne sme da ostane nasamo sa cerkama ako nije majka prisutna( neznam zasto al takvo je      pravilo, mozda da nedaj boze ne naskoci na cerke).

Isto tako ni majka ne sme da ostane nasamo sa sinovima ako otac nije tu.

- zatvorenik ne sme da ostane sam sa ostalima ako nije policajac sa njim ali sme da ostane sam na drugoj strani, nece pobeci(valjda zato sto pandur ima pistolj)

...1.policajac i zatvorenik

2.vraca se policajac

3.policajac i sin

4 .vraca se policajac i zatvorenik

5.otac i 2. sin prelaze

6. vraca se otac

7.otac i majka prelaze

8.vraca se majka

9.zatvorenik i policajac prelaze

10.vraca se otac

11.otac i majka prelaze

12. vraca se majka

13.majka i  i cerka prelaze

14.vraca se policajac i zatvorenik

15.policajac i 2. cerka prelaze(ostaje zatvorenik)

16.vraca se policajac

17.policajac i zatvorenik prelaze

           svi su presli

...Tacno 

0 za
0 protiv
dejan86
2010-08-24
???

modesty_blaise napisa...

elnesone napisa...

1.zadatak

Date su tri vreće. Svaka sadrži isti broj zlatnika. U dvije vreće su samo pravi zlatnici koji su težine     7.0 g svaki a u jednoj vreći su samo lažni zlatnici težine 7.5 g svaki.  Imamo samo jednu vagu za mjerenje. Koliko mjerenja minimalno treba obaviti da se utvrdi u kojoj vreći su lažni zlatnici.

 

2.zadatak

U zelenoj tegli se nalazi 200 zelenih klikera, a u crvenoj tegli 200 crvenih klikera.  Iz  zelene tegle vadimo 30 klikera i stavljamo u crvenu teglu i dobro ih izmješamo.  Sa velikom kašikom u koju staje 30 klikera izvadimo 30 klikera iz crvene tegle i vratimo u zelenu teglu.

Koja tegla će imati više klikera druge boje?

zelena  tegla   crvenih             ili

crvena  tegla   zelenih

...zadatak 1

Dovoljno je jedno merenje.. Ako imamo vagu sa dva tasa, onda uzmemo po jedan zlatnik iz svake kese, dva stavimo na po jedan tas. Ako je vaga u ravnotezi preostali zlatnik je lazan, a ako vaga nije u ravnotezi, tezi je iz kese sa laznim zlatnicima.. A ako imamo vagu koja meri tacnu tezinu, iz prve kese uzmemo jedan zlatnik, iz druge dva, iz trece tri.. Da su sve pravi zlatnici, tezina bi bila 42g, ali kako nisu bice teze.Ako je 42.5 lazni su u prvoj kesi, 43g- u drugoj, 43.5 u trecoj

..zadatak 2

odokativnim racunom, najveca verovatnoca je da ce biti isto kugli druge boje, koliko crvenih medju zelenim, toliko i zelenih medju crvenim.. al nisam se mnogo ugubila u zadatak

...Drugi zadatak si dobro rješila, što se tiče prvog i to je u redu,imamo vagu koja mjeri tačnu težinu, samo nema potrebe uzimati tri zlatnika iz treće vreće, dovolnjo je uzeti jedan iz prve i dva iz druge i sad ako je težina 21, lažni je iz treće, ako je 21,5 lažni je iz prve, i ako je 22 lažni je iz druge.Svaka čast.

0 za
0 protiv
MaLin
2010-08-24
???

raborg napisa...

   Kolona vojnika dugačka jedan kilometar počinje da maršira ravnomernom brzinom.Posle tačno jednog kilometra zaustavljaju se.U istom trenutku kad je kolona krenula, glasnik koji se nalazi na samom začelju, izdvaja se i počinje da se kreće ka početku, takodje ravnomernom brzinom.Njegova brzina je taman takva da uspe da stigne sam početak kolone , istovremeno se okrene i nastavi nazad ka začelju, gde stigne baš u trenutku kada se i sama kolona zaustavila.Celim putem, brzine kolone kao ni glasnika nisu se menjale.Pitanje glasi: -Koliko je glasnik prepešačio?

    Nadam se da će malo mučenja oko ovog zadatka mnogima doneti zadovoljstvo, kao i da će zadataka biti još.

...jedan km?

0 za
1 protiv
Marija Brcanski
2010-08-25
???

evo par laganih problemčića sa jednog konkursa za posao

1) Поштански службеник је добио коверте у пакетићима од 200 комада. Ако му је за одбројавање 10
коверата потребно 5 секунди, за колико секунди може издвојити 150 коверата ?
2) Три брата располажу са 5 ћупова пуних меда, 5 празних ћупова и 5 ћупова до пола напуњених медом.
Како да поделе мед и ћупове тако да свако од њих добије једнаку количину меда и једнак број ћупова
(пресипање није дозвољено).
3) Мирко и Славко требају за 90 минута прећи пут дугачак 12 км, а имају само један бицикл на коме свако
од њих може развити брзину од 20 км на сат, или ићи пешке брзином 5 км на сат. Могу ли Мирко и Славко
прећи предвиђени пут без закашњења, под условом да истовремено не могу возити бицикл ?

0 za
0 protiv
modesty_blaise
2010-08-26
???

modesty_blaise napisa...

evo par laganih problemčića sa jednog konkursa za posao

1) Поштански службеник је добио коверте у пакетићима од 200 комада. Ако му је за одбројавање 10
коверата потребно 5 секунди, за колико секунди може издвојити 150 коверата ?
2) Три брата располажу са 5 ћупова пуних меда, 5 празних ћупова и 5 ћупова до пола напуњених медом.
Како да поделе мед и ћупове тако да свако од њих добије једнаку количину меда и једнак број ћупова
(пресипање није дозвољено).
3) Мирко и Славко требају за 90 минута прећи пут дугачак 12 км, а имају само један бицикл на коме свако
од њих може развити брзину од 20 км на сат, или ићи пешке брзином 5 км на сат. Могу ли Мирко и Славко
прећи предвиђени пут без закашњења, под условом да истовремено не могу возити бицикл ?

...25 sekundi za prvi zadatak.

0 za
0 protiv
MaLin
2010-08-26
???

modesty_blaise napisa...

evo par laganih problemčića sa jednog konkursa za posao

1) Поштански службеник је добио коверте у пакетићима од 200 комада. Ако му је за одбројавање 10
коверата потребно 5 секунди, за колико секунди може издвојити 150 коверата ?
2) Три брата располажу са 5 ћупова пуних меда, 5 празних ћупова и 5 ћупова до пола напуњених медом.
Како да поделе мед и ћупове тако да свако од њих добије једнаку количину меда и једнак број ћупова
(пресипање није дозвољено).
3) Мирко и Славко требају за 90 минута прећи пут дугачак 12 км, а имају само један бицикл на коме свако
од њих може развити брзину од 20 км на сат, или ићи пешке брзином 5 км на сат. Могу ли Мирко и Славко
прећи предвиђени пут без закашњења, под условом да истовремено не могу возити бицикл ?

...25 sekundi za prvi zadatak.

0 za
0 protiv
MaLin
2010-08-26
???

elnesone napisa...

modesty_blaise napisa...

evo par laganih problemčića sa jednog konkursa za posao

1) Поштански службеник је добио коверте у пакетићима од 200 комада. Ако му је за одбројавање 10
коверата потребно 5 секунди, за колико секунди може издвојити 150 коверата ?
2) Три брата располажу са 5 ћупова пуних меда, 5 празних ћупова и 5 ћупова до пола напуњених медом.
Како да поделе мед и ћупове тако да свако од њих добије једнаку количину меда и једнак број ћупова
(пресипање није дозвољено).
3) Мирко и Славко требају за 90 минута прећи пут дугачак 12 км, а имају само један бицикл на коме свако
од њих може развити брзину од 20 км на сат, или ићи пешке брзином 5 км на сат. Могу ли Мирко и Славко
прећи предвиђени пут без закашњења, под условом да истовремено не могу возити бицикл ?

...25 sekundi za prvi zadatak.

...2. zadatak

1. brat će uzeti 2 puna, 1 polupun i 2 prazna

2. brat isto kao i prvi

3. brat 1 pun, 3 polupuna i 1 prazan

0 za
0 protiv
MaLin
2010-08-26
???

3. zadatak

18 minuta biciklom i 72 minute pješke, kako Mirko, tako i Slavko

0 za
0 protiv
MaLin
2010-08-26
???

elnesone napisa...

3. zadatak

18 minuta biciklom i 72 minute pješke, kako Mirko, tako i Slavko

...to u stvari znaci da npr.Mirko predje prvih 6 km biciklom a ostatak peske,a Slavko prvih 6 km predje peske a ostatak biciklom

2 za
0 protiv
Слађан
2010-08-26
???

@SLAKI82

Pa tako nekako, u isto vrijeme kreću obojica, recimo Mirko biciklom a Slavko pješke,posle 18 minuta Mirko će preći 6 km i ostaviti biciklo a zatim nastaviti pješke, u sledićih 54 minute Slavko će doći do bicikla, i za 18 minuta doći do cilja u isto vrijeme kad i Mirko.

E sad malo šale, valjda se Slavko nije umorio posle napornog pješačenja, te će imati snage da biciklom  postigne brzinu od 20 km na sat, kako je zadatak isključivo matematičke prirode, tj nigdje se ne pominje da u obzir treba uzeti utrošenu energiju i izvršeni rad, ne daj bože uzeti u obzir silu trenja kad se ide pješke i vozi biciklo, mislim da sam dobro rješio, al opet bi volio da modesty kaže da li sam u pravu.

2 za
0 protiv
MaLin
2010-08-26
???

elnesone napisa...

@SLAKI82

Pa tako nekako, u isto vrijeme kreću obojica, recimo Mirko biciklom a Slavko pješke,posle 18 minuta Mirko će preći 6 km i ostaviti biciklo a zatim nastaviti pješke, u sledićih 54 minute Slavko će doći do bicikla, i za 18 minuta doći do cilja u isto vrijeme kad i Mirko.

E sad malo šale, valjda se Slavko nije umorio posle napornog pješačenja, te će imati snage da biciklom  postigne brzinu od 20 km na sat, kako je zadatak isključivo matematičke prirode, tj nigdje se ne pominje da u obzir treba uzeti utrošenu energiju i izvršeni rad, ne daj bože uzeti u obzir silu trenja kad se ide pješke i vozi biciklo, mislim da sam dobro rješio, al opet bi volio da modesty kaže da li sam u pravu.

...Sasvim tacno :) Svaka cast.. Mada nista manje nisam ni ocekivala, brze i precizne, i nadasve tacne odgovore ;)

Ja kao matematicar, verujem da su fizicari uzeli u obzir trenje, atmosferski pritisak, i premor, pa dobili brzinu od 20 km na sat .. a sad utrosena energija, i rad, e to je vec za neki novi zadatak.. Mene vise brine to ostavljanje bicikle na pola puta, ako ga neko ukrade, jadan Slavko do cilja mora pesaka :)

2 za
0 protiv
modesty_blaise
2010-08-26
???

Iju, zadnji zadatak prije 19 dana, meni je ovo najzinimljivija tema, zar da padne u zaborav!?

1.Fudbalski tim je odigrao 3 utakmice:jednu je dobio, jednu igrao neriješeno i jednu izgubio.Tim je na ovim utakmicama dao tri gola i primio jedan.

Kojim rezultatom je završena svaka od ovih utakmica?

 

2.U četverospratnoj kući niko ne stanuje u prizemlju.Stoko stanuje iznad Vaskrsije,ali ispod Maksima, a Sofren ispod Vaskrsije.

Na kom spratu stanuje svako od njih?

 

3.Jedan dječak govori drugom: "Ako ti meni daš polovinu svog novca, ja mogu kupiti tačno jednu olovku."Drugi dječak odgovara:"Ako ti meni daš polovinu svog novca, onda ja mogu kupiti tačno dvije olovke."

Koliko je bilo novca kod prvog dječaka?

 

4.Neki Grk se rodio 5.januara 30. godine prije naše ere, a umro 5.janura 30. godine naše ere.

Koliko je godina živio?

 

5.Dječak je otplovio na čamcu iz mjesta A u mjesto B suprotno toku rijeke (uzvodno).Na kojem će se rastojanju od B nalaziti kroz 30 minuta, ako je brzina čamca 6 km na čas, brzina rijeke 2 km na čas, a rastojanje od A do B iznosi 3 km?

 

6.Čvorak sustiže bumbara koji je od njega udaljen 270 m.Kroz koliko će ga sekundi stići ako bumbar leti brzinom 18 km na čas, a čvorak 72 km na čas?

 

7.U datom množenju odredi cifre koje treba upisati umjesto slova, pa da množenje bude tačno.Ista slova predstavljaju iste cifre, a različita slova različite cifre.

 

                                                                A B B * A V

                                                                 V G G

                                                             A B B     

                                                            A D V G          

 

8.Odredi vrijednost razlomka.Ista pravila kao iz prethodnog zadatka.

 

                                              P*R*O*I*Z*V*O*D    

                                                 R*A*Z*L*I*K*A

 

9.Imamo matricu 3*3.Umjesto zvjedica upiši sledeće brojeve (4,6,7,9,10,11 i 12) tako da se u svakom pravcu dobije zbir 24.

 

                                                               *     *     *

                                                              *     8     *

                                                              *     *      5

 

10.Vaskrsija Gegić i Makso Ćulibrk su zajedno uplatili tiket za igru LOTO.Vaskrsija je platio      4,20 KM, a Makso 10,50 KM. Kobijagi imali su sreće i dobili 127,75 KM.Kako će pošteno podijeliti dobitak.

 

Nadam se da mi nećete zamjeriti na ovolikom broju zadataka, ali mislim da smo zanemarili jednu od najzanimljivijih tema, bar ja tako mislim.

Pozdrav svima i nadam se da će te uživati prilikom rješavanja zadataka.

                                                              

0 za
0 protiv
MaLin
2010-09-15
???

Ajd da bi se vise ljudi ukljucilo, samo jedan od mene, najlaksi:

1. 3:0 ; 0:0; 0:1.
 

0 za
0 protiv
Kalimist
2010-09-15
???

2. zadatak:

         1.sprat:Sofren

         2.sprat:Vaskrsije

         3.sprat:Stoko

        4.sprat:Maksim

0 za
0 protiv
Слађан
2010-09-15
???

3.zadatak

Prvi decak nije imao novac.

0 za
0 protiv
Слађан
2010-09-15
???

9. Постоје два решења.

7. 144*12=1728

8. 0

0 za
0 protiv
Цане Вукић
2010-09-15
???

Петар Марковић napisa...

8. 0

...ili nije definisano.

0 za
0 protiv
Kalimist
2010-09-15
???

@Vaskrsija: normalno da ti niko nece zameriti za broj zadataka, tvoj trud je za pohvalu, ali daj malo teza pitanja :)

0 za
0 protiv
Kalimist
2010-09-15
???

Bravo momci(Kalimist,Slaki82,Petre) svi ste tačno rješili

@Kalimist,

što se tiče 8.  zadatka, rješenje ne bilo definisano kad bi nekom slovu na pozicji nazivnika (imenilac) dodjeli nulu, pa bi u tom slučaju imali dijeljenje nulom, zato dodjeljujemo nekom slovu na poziciji brojnika, pa je samim tim i njegova vrijednost jednaka nuli, tj čitav razlomak ima vrijednost nula.

U pravu si kad kažeš da zadaci nisu teški, nisam ni težio da budu teški, već prije svega zanimljivi, jer među ovoliko dobrih matematičara vrlo je teško naći težak zadatak, što opet po mom mišljenu i ne treba pokušavati, jer ne treba da nas zaboli glava rješavajući ih, nego da se dobro zabavimo i ubijemo dosadu. Ali kad već tražiš, evo 3 specijalno za tebe (mada čisto sumnjam da će i to biti teško za tako dobrog matematičara)

 

1.Oko vrta oblika pravougaonika,čije su dimenzije 40 m i 30 m, treba postaviti stazu koja je svuda jednako široka, tako da njena površina bude 296 m2.

Kolika je širina staze?

 

2.Polovinu bazena napuni jedna cijev, drugu polovinu druga, a obje cijevi su bile otvorene 25 sati.Da su cijevi bile otvorene istovremeno, bazen bi se napunio za 12 sati.

Za koliko bi časova napubila bazen svaka cijev posebno?

 

3.Oluja polomi stablo visine 16 m i pri tom mu vrh dodirne zemlju 8 m daleko od stabla.

Na kojo se visini polomilo stablo?

 

A sad pitanje za sve.

Koji pravilni mnogougao ima 90 dijagonala?

1 za
0 protiv
MaLin
2010-09-15
???

Vaskrsija napisa...

A sad pitanje za sve.

Koji pravilni mnogougao ima 90 dijagonala?

...Mislim da je u pitanju petnaestougao.

0 za
0 protiv
Vladimir Obradović
2010-09-15
???

Vladimir Obradović napisa...

Vaskrsija napisa...

A sad pitanje za sve.

Koji pravilni mnogougao ima 90 dijagonala?

...Mislim da je u pitanju petnaestougao.

...Tako je, svaka čast.

0 za
0 protiv
MaLin
2010-09-15
???

Posto Vaskrsija zadaje bas lake zadatke,evo ja cu zadati jedan malo tezi.

Na jednom ostrvu zivi 600 kameleona i to:300 zelenih,200 plavih i 100 crvenih.Ako se sretnu dva kameleona razlicite boje,oni istovremeno menjaju boju i oba postaju iste boje i to one trece(npr.ako se susretnu zeleni i plavi,oba postaju crveni).Moze li se desiti da posle izvesnog vremena svi kameleoni budu iste boje?

2 za
0 protiv
mc80
2010-09-15
???

Vaskrsija napisa...

što se tiče 8.  zadatka, rješenje ne bilo definisano kad bi nekom slovu na pozicji nazivnika (imenilac) dodjeli nulu, pa bi u tom slučaju imali dijeljenje nulom, zato dodjeljujemo nekom slovu na poziciji brojnika, pa je samim tim i njegova vrijednost jednaka nuli, tj čitav razlomak ima vrijednost nula.

...upravo tako, hteo sam da naglasim da ne mora resenje da bude samo nula. Usput, ti si iz Bosne?

Vaskrsija napisa...

 jer ne treba da nas zaboli glava rješavajući ih, nego da se dobro zabavimo i ubijemo dosadu. Ali kad već tražiš

...moram da ti budem iskren, mada cu verovatno izgledati kao ludak kad ovo napisem: ja HOCU da me zaboli glava dok resavam neki zadatak, mnogo volim izazove, jer je osecaj kad dodjes do necega za sta si se debelo potrudio neopisiv :)

Vaskrsija napisa...

...evo 3 specijalno za tebe (mada čisto sumnjam da će i to biti teško za tako dobrog matematičara)

 

1.Oko vrta oblika pravougaonika,čije su dimenzije 40 m i 30 m, treba postaviti stazu koja je svuda jednako široka, tako da njena površina bude 296 m2.

Kolika je širina staze?

 

2.Polovinu bazena napuni jedna cijev, drugu polovinu druga, a obje cijevi su bile otvorene 25 sati.Da su cijevi bile otvorene istovremeno, bazen bi se napunio za 12 sati.

Za koliko bi časova napubila bazen svaka cijev posebno?

 

3.Oluja polomi stablo visine 16 m i pri tom mu vrh dodirne zemlju 8 m daleko od stabla.

Na kojo se visini polomilo stablo?

...pre svega, hvala na komplimentu, i jos jednom hvala na trudu koji ulazes.

1. 33.49, ako nisam pogresio u racunu.

2. 10h i 15h.

3. 6m.

0 za
0 protiv
Kalimist
2010-09-15
???

mc80 napisa...

Posto Vaskrsija zadaje bas lake zadatke,evo ja cu zadati jedan malo tezi.

Na jednom ostrvu zivi 600 kameleona i to:300 zelenih,200 plavih i 100 crvenih.Ako se sretnu dva kameleona razlicite boje,oni istovremeno menjaju boju i oba postaju iste boje i to one trece(npr.ako se susretnu zeleni i plavi,oba postaju crveni).Moze li se desiti da posle izvesnog vremena svi kameleoni budu iste boje?

...bez nekog dubioznog razmisljanja, mislim da ne moze :)

0 za
0 protiv
Слађан
2010-09-15
???

@Kalimist,

Prvi si pogriješio, i to puno, treći si dobro riješio (pitagorina teorema) .

Da, jesam iz Bosne,Modriča.

@mc80

Možda griješim, al umjesto 300, 200 i 100, možemo reći 3, 2 i  1, prema tome, ne može.Ispravi me ako griješim.

0 za
0 protiv
MaLin
2010-09-15
???

Vaskrsija napisa...

Bravo momci(Kalimist,Slaki82,Petre) svi ste tačno rješili

@Kalimist,

što se tiče 8.  zadatka, rješenje ne bilo definisano kad bi nekom slovu na pozicji nazivnika (imenilac) dodjeli nulu, pa bi u tom slučaju imali dijeljenje nulom, zato dodjeljujemo nekom slovu na poziciji brojnika, pa je samim tim i njegova vrijednost jednaka nuli, tj čitav razlomak ima vrijednost nula.

U pravu si kad kažeš da zadaci nisu teški, nisam ni težio da budu teški, već prije svega zanimljivi, jer među ovoliko dobrih matematičara vrlo je teško naći težak zadatak, što opet po mom mišljenu i ne treba pokušavati, jer ne treba da nas zaboli glava rješavajući ih, nego da se dobro zabavimo i ubijemo dosadu. Ali kad već tražiš, evo 3 specijalno za tebe (mada čisto sumnjam da će i to biti teško za tako dobrog matematičara)

 

1.Oko vrta oblika pravougaonika,čije su dimenzije 40 m i 30 m, treba postaviti stazu koja je svuda jednako široka, tako da njena površina bude 296 m2.

Kolika je širina staze?

 

2.Polovinu bazena napuni jedna cijev, drugu polovinu druga, a obje cijevi su bile otvorene 25 sati.Da su cijevi bile otvorene istovremeno, bazen bi se napunio za 12 sati.

Za koliko bi časova napubila bazen svaka cijev posebno?

 

3.Oluja polomi stablo visine 16 m i pri tom mu vrh dodirne zemlju 8 m daleko od stabla.

Na kojo se visini polomilo stablo?

 

A sad pitanje za sve.

Koji pravilni mnogougao ima 90 dijagonala?

...da nije prvi zadatak 19,4m

0 za
0 protiv
Слађан
2010-09-15
???

Vaskrsija napisa...

@mc80

Možda griješim, al umjesto 300, 200 i 100, možemo reći 3, 2 i  1, prema tome, ne može.Ispravi me ako griješim.

Ako mislis da se ovaj zadatak moze pojednostaviti i svesti na slucaj 3,2,1 onda treba da objasnis precizno zasto je to tako.

0 za
0 protiv
mc80
2010-09-15
???

SLAKI82 napisa...

Vaskrsija napisa...

Bravo momci(Kalimist,Slaki82,Petre) svi ste tačno rješili

@Kalimist,

što se tiče 8.  zadatka, rješenje ne bilo definisano kad bi nekom slovu na pozicji nazivnika (imenilac) dodjeli nulu, pa bi u tom slučaju imali dijeljenje nulom, zato dodjeljujemo nekom slovu na poziciji brojnika, pa je samim tim i njegova vrijednost jednaka nuli, tj čitav razlomak ima vrijednost nula.

U pravu si kad kažeš da zadaci nisu teški, nisam ni težio da budu teški, već prije svega zanimljivi, jer među ovoliko dobrih matematičara vrlo je teško naći težak zadatak, što opet po mom mišljenu i ne treba pokušavati, jer ne treba da nas zaboli glava rješavajući ih, nego da se dobro zabavimo i ubijemo dosadu. Ali kad već tražiš, evo 3 specijalno za tebe (mada čisto sumnjam da će i to biti teško za tako dobrog matematičara)

 

1.Oko vrta oblika pravougaonika,čije su dimenzije 40 m i 30 m, treba postaviti stazu koja je svuda jednako široka, tako da njena površina bude 296 m2.

Kolika je širina staze?

 

2.Polovinu bazena napuni jedna cijev, drugu polovinu druga, a obje cijevi su bile otvorene 25 sati.Da su cijevi bile otvorene istovremeno, bazen bi se napunio za 12 sati.

Za koliko bi časova napubila bazen svaka cijev posebno?

 

3.Oluja polomi stablo visine 16 m i pri tom mu vrh dodirne zemlju 8 m daleko od stabla.

Na kojo se visini polomilo stablo?

 

A sad pitanje za sve.

Koji pravilni mnogougao ima 90 dijagonala?

...da nije prvi zadatak 19,4m

...Ne, nije, najbolje je nacrtati pravougaonik u pravougaoniku, i onda se lijepo postavi zadatak

(40+2x)*(30+2x)-30*40=296

0 za
0 protiv
MaLin
2010-09-15
???

Vaskrsija napisa...

Bravo momci(Kalimist,Slaki82,Petre) svi ste tačno rješili

@Kalimist,

što se tiče 8.  zadatka, rješenje ne bilo definisano kad bi nekom slovu na pozicji nazivnika (imenilac) dodjeli nulu, pa bi u tom slučaju imali dijeljenje nulom, zato dodjeljujemo nekom slovu na poziciji brojnika, pa je samim tim i njegova vrijednost jednaka nuli, tj čitav razlomak ima vrijednost nula.

U pravu si kad kažeš da zadaci nisu teški, nisam ni težio da budu teški, već prije svega zanimljivi, jer među ovoliko dobrih matematičara vrlo je teško naći težak zadatak, što opet po mom mišljenu i ne treba pokušavati, jer ne treba da nas zaboli glava rješavajući ih, nego da se dobro zabavimo i ubijemo dosadu. Ali kad već tražiš, evo 3 specijalno za tebe (mada čisto sumnjam da će i to biti teško za tako dobrog matematičara)

 

1.Oko vrta oblika pravougaonika,čije su dimenzije 40 m i 30 m, treba postaviti stazu koja je svuda jednako široka, tako da njena površina bude 296 m2.

Kolika je širina staze?

 

2.Polovinu bazena napuni jedna cijev, drugu polovinu druga, a obje cijevi su bile otvorene 25 sati.Da su cijevi bile otvorene istovremeno, bazen bi se napunio za 12 sati.

Za koliko bi časova napubila bazen svaka cijev posebno?

 

3.Oluja polomi stablo visine 16 m i pri tom mu vrh dodirne zemlju 8 m daleko od stabla.

Na kojo se visini polomilo stablo?

 

A sad pitanje za sve.

Koji pravilni mnogougao ima 90 dijagonala?

...2.zadatak

ako obe cevi posebno  kao sto pise pune bazen do pola 25 sati,to znaci da ce obe cevi posebno ceo bazen da napune za 50 sati

1 za
0 protiv
Слађан
2010-09-15
???

mc80 napisa...

Vaskrsija napisa...

@mc80

Možda griješim, al umjesto 300, 200 i 100, možemo reći 3, 2 i  1, prema tome, ne može.Ispravi me ako griješim.

Ako mislis da se ovaj zadatak moze pojednostaviti i svesti na slucaj 3,2,1 onda treba da objasnis precizno zasto je to tako.

...Reko možda griješim, prvo sva tri broja su djeljiva sa 100, a drugo, binarnim djeljenjem kad tad moramo napustiti skup pozitvnih cijelih brojeva, al kao šro rekoh, ne znači da sam u pravu.

0 za
0 protiv
MaLin
2010-09-15
???

Vaskrsija napisa...

SLAKI82 napisa...

Vaskrsija napisa...

Bravo momci(Kalimist,Slaki82,Petre) svi ste tačno rješili

@Kalimist,

što se tiče 8.  zadatka, rješenje ne bilo definisano kad bi nekom slovu na pozicji nazivnika (imenilac) dodjeli nulu, pa bi u tom slučaju imali dijeljenje nulom, zato dodjeljujemo nekom slovu na poziciji brojnika, pa je samim tim i njegova vrijednost jednaka nuli, tj čitav razlomak ima vrijednost nula.

U pravu si kad kažeš da zadaci nisu teški, nisam ni težio da budu teški, već prije svega zanimljivi, jer među ovoliko dobrih matematičara vrlo je teško naći težak zadatak, što opet po mom mišljenu i ne treba pokušavati, jer ne treba da nas zaboli glava rješavajući ih, nego da se dobro zabavimo i ubijemo dosadu. Ali kad već tražiš, evo 3 specijalno za tebe (mada čisto sumnjam da će i to biti teško za tako dobrog matematičara)

 

1.Oko vrta oblika pravougaonika,čije su dimenzije 40 m i 30 m, treba postaviti stazu koja je svuda jednako široka, tako da njena površina bude 296 m2.

Kolika je širina staze?

 

2.Polovinu bazena napuni jedna cijev, drugu polovinu druga, a obje cijevi su bile otvorene 25 sati.Da su cijevi bile otvorene istovremeno, bazen bi se napunio za 12 sati.

Za koliko bi časova napubila bazen svaka cijev posebno?

 

3.Oluja polomi stablo visine 16 m i pri tom mu vrh dodirne zemlju 8 m daleko od stabla.

Na kojo se visini polomilo stablo?

 

A sad pitanje za sve.

Koji pravilni mnogougao ima 90 dijagonala?

...da nije prvi zadatak 19,4m

...Ne, nije, najbolje je nacrtati pravougaonik u pravougaoniku, i onda se lijepo postavi zadatak

(40+2x)*(30+2x)-30*40=296

...mislim da je tacnija postavka:

(40+x)*(30+x)=1200+296

tako da ispada da je X=4m odnosno to je sirina staze tj. udaljenost staze od pravouganika 40*30

0 za
0 protiv
Слађан
2010-09-15
???

mc80 napisa...

Na jednom ostrvu zivi 600 kameleona i to:300 zelenih,200 plavih i 100 crvenih.Ako se sretnu dva kameleona razlicite boje,oni istovremeno menjaju boju i oba postaju iste boje i to one trece(npr.ako se susretnu zeleni i plavi,oba postaju crveni).Moze li se desiti da posle izvesnog vremena svi kameleoni budu iste boje?

... Да би могло да се деси да на крају остану сви исте боје, претпоследњи распоред боја мора бити 1, 1, 598. То значи да две боје имају исти број, тј. да је разлика 0, односно да је дељива са 3. Али, то је немогуће, јер су почетне разлике између боја 100, 100 и 200, ниједна разлика није дељива са 3, а при сваком сусрету дешава се -1, -1 и +2 у броју камелеона, тј. разлике бројева задржавају својство (не)дељивости са 3.

Надам се да сам кол'ко тол'ко био јасан...smiley

@Калимист: Рекао бих да изрази 5/0, па и 0/0 нису разломци, јер уопште нису ни бројеви. Питање је било која је вредност разломка.

0 za
0 protiv
Цане Вукић
2010-09-15
???

SLAKI82 napisa...

Vaskrsija napisa...

SLAKI82 napisa...

Vaskrsija napisa...

Bravo momci(Kalimist,Slaki82,Petre) svi ste tačno rješili

@Kalimist,

što se tiče 8.  zadatka, rješenje ne bilo definisano kad bi nekom slovu na pozicji nazivnika (imenilac) dodjeli nulu, pa bi u tom slučaju imali dijeljenje nulom, zato dodjeljujemo nekom slovu na poziciji brojnika, pa je samim tim i njegova vrijednost jednaka nuli, tj čitav razlomak ima vrijednost nula.

U pravu si kad kažeš da zadaci nisu teški, nisam ni težio da budu teški, već prije svega zanimljivi, jer među ovoliko dobrih matematičara vrlo je teško naći težak zadatak, što opet po mom mišljenu i ne treba pokušavati, jer ne treba da nas zaboli glava rješavajući ih, nego da se dobro zabavimo i ubijemo dosadu. Ali kad već tražiš, evo 3 specijalno za tebe (mada čisto sumnjam da će i to biti teško za tako dobrog matematičara)

 

1.Oko vrta oblika pravougaonika,čije su dimenzije 40 m i 30 m, treba postaviti stazu koja je svuda jednako široka, tako da njena površina bude 296 m2.

Kolika je širina staze?

 

2.Polovinu bazena napuni jedna cijev, drugu polovinu druga, a obje cijevi su bile otvorene 25 sati.Da su cijevi bile otvorene istovremeno, bazen bi se napunio za 12 sati.

Za koliko bi časova napubila bazen svaka cijev posebno?

 

3.Oluja polomi stablo visine 16 m i pri tom mu vrh dodirne zemlju 8 m daleko od stabla.

Na kojo se visini polomilo stablo?

 

A sad pitanje za sve.

Koji pravilni mnogougao ima 90 dijagonala?

...da nije prvi zadatak 19,4m

...Ne, nije, najbolje je nacrtati pravougaonik u pravougaoniku, i onda se lijepo postavi zadatak

(40+2x)*(30+2x)-30*40=296

...mislim da je tacnija postavka:

(40+x)*(30+x)=1200+296

tako da ispada da je X=4m odnosno to je sirina staze tj. udaljenost staze od pravouganika 40*30

...Pazi, širina i s desne i lijeve strane, ko i s gornje i donje.

Zato mora biti (40+2x)*(30+2x)=1200+296 a ne (40+x)*(30+x)=1200+296

Na kraju dobiješ kvadratnu jednačinu koja ima dva rješenja, a u obzir dolazi samo pozitivno, tako da x nije četiri, već .........Pa blizu si!

1 za
0 protiv
MaLin
2010-09-15
???

Vaskrsija napisa...

 

...Pazi, širina i s desne i lijeve strane, ko i s gornje i donje.

Zato mora biti (40+2x)*(30+2x)=1200+296 a ne (40+x)*(30+x)=1200+296

Na kraju dobiješ kvadratnu jednačinu koja ima dva rješenja, a u obzir dolazi samo pozitivno, tako da x nije četiri, već .........Pa blizu si!

...da da  kad bolje razmislim stranice od 30 i 40 se povecavaju za X ali dva puta pa je resenje 2m

(da li mi je resenje za 2. zadatak ispravno)

0 za
0 protiv
Слађан
2010-09-15
???

Петар Марковић napisa...

mc80 napisa...

Na jednom ostrvu zivi 600 kameleona i to:300 zelenih,200 plavih i 100 crvenih.Ako se sretnu dva kameleona razlicite boje,oni istovremeno menjaju boju i oba postaju iste boje i to one trece(npr.ako se susretnu zeleni i plavi,oba postaju crveni).Moze li se desiti da posle izvesnog vremena svi kameleoni budu iste boje?

... Да би могло да се деси да на крају остану сви исте боје, претпоследњи распоред боја мора бити 1, 1, 598. То значи да две боје имају исти број, тј. да је разлика 0, односно да је дељива са 3. Али, то је немогуће, јер су почетне разлике између боја 100, 100 и 200, ниједна разлика није дељива са 3, а при сваком сусрету дешава се -1, -1 и +2 у броју камелеона, тј. разлике бројева задржавају својство (не)дељивости са 3.

Надам се да сам кол'ко тол'ко био јасан...smiley

 

Da,to je resenje.Bravo!

0 za
0 protiv
mc80
2010-09-15
???

SLAKI82 napisa...

Vaskrsija napisa...

 

...Pazi, širina i s desne i lijeve strane, ko i s gornje i donje.

Zato mora biti (40+2x)*(30+2x)=1200+296 a ne (40+x)*(30+x)=1200+296

Na kraju dobiješ kvadratnu jednačinu koja ima dva rješenja, a u obzir dolazi samo pozitivno, tako da x nije četiri, već .........Pa blizu si!

...da da  kad bolje razmislim stranice od 30 i 40 se povecavaju za X ali dva puta pa je resenje 2m

(da li mi je resenje za 2. zadatak ispravno)

...BINGO, to je to

0 za
0 protiv
MaLin
2010-09-15
???

 

Evo jednog laganog zadatka:

 

Krava 2,ovca 2,kukavica 4,petao-?

Zameni znak pitanja brojem i objasni sta on oznacava.

1 za
0 protiv
Слађан
2010-09-15
???

SLAKI82 napisa...

 

Evo jednog laganog zadatka:

 

Krava 2,ovca 2,kukavica 4,petao-?

Zameni znak pitanja brojem i objasni sta on oznacava.

...Petao 3 .... možda broj samoglasnika u riječi? :)

1 za
0 protiv
limes5
2010-09-15
???

Miodrag Mihajlović napisa...

SLAKI82 napisa...

 

Evo jednog laganog zadatka:

 

Krava 2,ovca 2,kukavica 4,petao-?

Zameni znak pitanja brojem i objasni sta on oznacava.

...Petao 3 .... možda broj samoglasnika u riječi? :)

...moze da bude odgovor i to ,ali moguc je jos jedan odgovor :)

1 za
0 protiv
Слађан
2010-09-16
???

SLAKI82 napisa...

Miodrag Mihajlović napisa...

SLAKI82 napisa...

 

Evo jednog laganog zadatka:

 

Krava 2,ovca 2,kukavica 4,petao-?

Zameni znak pitanja brojem i objasni sta on oznacava.

...Petao 3 .... možda broj samoglasnika u riječi? :)

...moze da bude odgovor i to ,ali moguc je jos jedan odgovor :)

...Možda 6, svaki sledeći broj je zbir dva prethodna, nešto kao fibonačijev niz.

1 za
0 protiv
MaLin
2010-09-16
???

Vaskrsija napisa...

 

...moze da bude odgovor i to ,ali moguc je jos jedan odgovor :)

...Možda 6, svaki sledeći broj je zbir dva prethodna, nešto kao fibonačijev niz.

...nije,odgovor ima šaljivu notu tj. i nije toliko usko vezan za matematiku :)

1 za
0 protiv
Слађан
2010-09-16
???

@Vaskrsija: ma i postavio sam ga preko pravougaonika u pravougaoniku, ali sam nesto silno pogresno izj..ao racun, radio sam u zurbi jer sam izlazio. Nego, ne rece nista za drugi, napisao si samo za prvi i treci zadatak?

Usput, da ispisem i logiku za drugi. Ako su cevi radile zajedno 25 sati, pretpostavimo da je prva radila x, druga 25-x sati. Dakle, prva je po satu punila 1/2x bazena, druga 1/(2*(25-x)), odnosno za 12 sati prva je napunila 12/2*x, druga 12/(2*(25-x)) bazena, pa je 6/x + 6/(25-x) = 1. Resavanjem se dobija da je x=10, odnosno 25-x=15.

1 za
0 protiv
Kalimist
2010-09-16
???

raborg napisa...

 

   A evo sad jedan koji nisam resio.

   Na raspolaganju je sedam tegova i vaga sa dva tasa.Teret se meri s maksimalno tri tega i to tako da se oni mogu stavljati na bilo koju stranu vage, ili na onu gde je i sam teret ili na suprotnu.Cilj je pronaci tezine ovih sedam tegova(celi brojevi), tako da se moze izmeriti svaka tezina pocevsi od 1kg pa preko svakog sledeceg celog broja do najveceg moguceg.

    Ja sam stigao do 106kg ali znam da to nije najveci moguci broj, a ne znam ni koji je konacan.

...Dal' je neko bar pokušao ovo da reši

0 za
0 protiv
raborg
2010-09-16
???

@Kalimist

Mogu samo da kažem (u vezi 2. zadatka) svaka čast, predobro objašnjeno, u stvari, kao i u prvom zadatku, imamo kvadratnu jednačinu, sva mudrost je postaviti je na pravi način, što si ti odlično uradio, ova kvadratna jednačina ima oba pozitvna rješenja a t o su x1=10 i x2=15.Još jednom bravo.

BTW

ne reče mi zašto si me pitao da li sam iz Bosne.

 

@raborg

Da li se mjerenje na vagi može vršiti bez tegova, na oba tasa stavljaš teret?

 

A sad, jedan zadatak za razmišljanje

Kako bez papira i olovke, za ne više od jedne minute izračunati zbir prvih 200 brojeva iz skupa N?

0 za
0 protiv
MaLin
2010-09-16
???

Vaskrsija napisa...

BTW

ne reče mi zašto si me pitao da li sam iz Bosne.

...zato sto si napisao nazivnik, pa me je interesovalo odakle dolazi taj termin, Bosne ili Crne Gore, njuh me nije prevario :)


Vaskrsija napisa...

Kako bez papira i olovke, za ne više od jedne minute izračunati zbir prvih 200 brojeva iz skupa N?

...minut je podosta vremena da se pomnoze 201 i 100 :)

0 za
0 protiv
Kalimist
2010-09-16
???

@Vaskrsija: Teret je samo jedan tako da ne moze da se koristi za merenje drugog tereta, inace bi zadatak bio trivijalan.Prvi put moras da budes u stanju da izmeris 1kg, drugi put 2kg...uvek koristeci maksimalno tri od ukupno sedam tegova.Kao sto sam vec negde napisao, najbolje sto sam uspeo je uz pomoc brojeva: 1, 2, 3, 9, 16, 48, 87.Kasnije sam pronasao jos nekoliko resenja,ali i ona vode do maksimalne tezine od 106kg(1, 2, 3, 10, 16, 48, 87 ili 1, 2, 3, 9, 16, 55, 87)

0 za
0 protiv
raborg
2010-09-16
???

Veliki matematičar Karl Fidrih Gaus (1777-1855), rano je pokazao svoju matematičku darovitost.On je brže i lakše rješavao zadatke od svojih drugova u razredu.Jednom prilikom njegov učitelj odluči dobro da ga zaposli, pa mu zada da sabere sve brojeve od 1 do 100.Na veliko iznenadjennje svog učitelja,Gaus ubrzo donese ovaj rezultat 5050.Gaus nije sabirao broj po broj, jer bi mu to oduzelo dosta vremena, nego je posmatrajući niz    1,2,3,...,98,99,100 uočio izvjesnu zakonitost, tj. kada spoji 1 i 100, 2 i 99, 3 i 98 itd. zbir je uvjek 101.Takvih parova ima 50 jer je ukupno 100 brojeva.Otuda je traženi zbir 101*50.

@Kalimist, ti si znao za ovu anegdotu?

E sad u vezi termina, kad sam ja išao u školu, koristili smo termine brojnik i nazavnik, te sam se prije par godina iznenadio kad me kćerka mog rođaka nije razumjela, nego mi je rekla da su to brojilac i imenilac, i zaista u novim udžbenicima tako piše, lično za mene su to djeljenik i djelilac, pa nek ih zovu kako hoće.

@raborg

sad mi je jasnija problematika zadatka, ali rješenje ne znam.Neko će već da rješi i to.

0 za
0 protiv
MaLin
2010-09-16
???

Vaskrsija napisa...

Veliki matematičar Karl Fidrih Gaus (1777-1855), rano je pokazao svoju matematičku darovitost.On je brže i lakše rješavao zadatke od svojih drugova u razredu.Jednom prilikom njegov učitelj odluči dobro da ga zaposli, pa mu zada da sabere sve brojeve od 1 do 100.Na veliko iznenadjennje svog učitelja,Gaus ubrzo donese ovaj rezultat 5050.Gaus nije sabirao broj po broj, jer bi mu to oduzelo dosta vremena, nego je posmatrajući niz    1,2,3,...,98,99,100 uočio izvjesnu zakonitost, tj. kada spoji 1 i 100, 2 i 99, 3 i 98 itd. zbir je uvjek 101.Takvih parova ima 50 jer je ukupno 100 brojeva.Otuda je traženi zbir 101*50.

@Kalimist, ti si znao za ovu anegdotu?

...jesam, ali ova zakonitost je vec deo nastavnog programa za srednju skolu, ako se dobro secam.

 

@Grobar: nisam probao da resim taj zadatak, trebalo bi vremena da se poradi na tome, ali me je podsetio na nesto slicno dok sam kao klinac igrao antitablic. Naime, bila je situacija u kojoj su na stolu bili A, 2, 8, pa sam razmisljao koja moja karta bi naterala protivnika da odnese bar nesto sa stola. Kad sam uvideo koja karta je u pitanju, primetio sam jednu simpaticnu stvar koju sam kasnije iskoristio da otkrijem u cemu je stvar kod sledeceg problema. Naime, drugar mi je dao tabelu ( koju cu ja ovde skratiti ),  rekao mi da uzmem neki broj iz tabele, i ako mu kazem u kojim se sve kolonama nalazi taj broj, on ce mi reci koji je broj u pitanju, bez gledanja u tabelu. Evo tabele, a vi mi recite kako je on bez greske uvek uspevao da pogodi broj.

 

  1    2    4    8

  3    3    5    9

  5    6    6   10

  7    7    7   11

  9   10  12  12

 11  11  13  13

 13  14  14  14

 15  15  15  15.

0 za
0 protiv
Kalimist
2010-09-16
???

Kalimist napisa...

Vaskrsija napisa...

Veliki matematičar Karl Fidrih Gaus (1777-1855), rano je pokazao svoju matematičku darovitost.On je brže i lakše rješavao zadatke od svojih drugova u razredu.Jednom prilikom njegov učitelj odluči dobro da ga zaposli, pa mu zada da sabere sve brojeve od 1 do 100.Na veliko iznenadjennje svog učitelja,Gaus ubrzo donese ovaj rezultat 5050.Gaus nije sabirao broj po broj, jer bi mu to oduzelo dosta vremena, nego je posmatrajući niz    1,2,3,...,98,99,100 uočio izvjesnu zakonitost, tj. kada spoji 1 i 100, 2 i 99, 3 i 98 itd. zbir je uvjek 101.Takvih parova ima 50 jer je ukupno 100 brojeva.Otuda je traženi zbir 101*50.

@Kalimist, ti si znao za ovu anegdotu?

...jesam, ali ova zakonitost je vec deo nastavnog programa za srednju skolu, ako se dobro secam.

 

@Grobar: nisam probao da resim taj zadatak, trebalo bi vremena da se poradi na tome, ali me je podsetio na nesto slicno dok sam kao klinac igrao antitablic. Naime, bila je situacija u kojoj su na stolu bili A, 2, 8, pa sam razmisljao koja moja karta bi naterala protivnika da odnese bar nesto sa stola. Kad sam uvideo koja karta je u pitanju, primetio sam jednu simpaticnu stvar koju sam kasnije iskoristio da otkrijem u cemu je stvar kod sledeceg problema. Naime, drugar mi je dao tabelu ( koju cu ja ovde skratiti ),  rekao mi da uzmem neki broj iz tabele, i ako mu kazem u kojim se sve kolonama nalazi taj broj, on ce mi reci koji je broj u pitanju, bez gledanja u tabelu. Evo tabele, a vi mi recite kako je on bez greske uvek uspevao da pogodi broj.

 

  1    2    4    8

  3    3    5    9

  5    6    6   10

  7    7    7   11

  9   10  12  12

 11  11  13  13

 13  14  14  14

 15  15  15  15.

...U pitanju je binarni sistem, odnosno svaki broj se pojavljuje onoliko puta i u odredjenim kolonama toliko puta koliko ima jedinica u njegovom binarnom zapisu.Npr. trojka se pojavljuje u prvoj i drugoj koloni zato sto je 3dec=11bin.Valjda nisam bio konfuzan.

Ovo bi moglo da se primeni u gornjem zadatku samo ako bi moglo da se upotrebi svih sedam tegova prilikom merenja i tada bi njihove tezine bile: 1, 3, 9, 27, 81, 243, 729, znaci brojni sistem s osnovom tri.

0 za
0 protiv
raborg
2010-09-16
???

raborg napisa...

...U pitanju je binarni sistem, odnosno svaki broj se pojavljuje onoliko puta i u odredjenim kolonama toliko puta koliko ima jedinica u njegovom binarnom zapisu.Npr. trojka se pojavljuje u prvoj i drugoj koloni zato sto je 3dec=11bin.Valjda nisam bio konfuzan.

Ovo bi moglo da se primeni u gornjem zadatku samo ako bi moglo da se upotrebi svih sedam tegova prilikom merenja i tada bi njihove tezine bile: 1, 3, 9, 27, 81, 243, 729, znaci brojni sistem s osnovom tri.

...upravo zato sam i zadao zadatak, Raborze. Upotrebom brojeva iz skupa {20, 21, 22...2n}, moze se zapisati bilo koji prirodni broj do 22n-1, kao zbir elemenata navedenog skupa, od kojih se svaki pojavljuje u jednacini ne vise od jedan put. Pretpostavljam da bi slicna logika mogla da se primeni i na tvoj sistem.

0 za
0 protiv
Kalimist
2010-09-16
???

Kalimist napisa...

raborg napisa...

...U pitanju je binarni sistem, odnosno svaki broj se pojavljuje onoliko puta i u odredjenim kolonama toliko puta koliko ima jedinica u njegovom binarnom zapisu.Npr. trojka se pojavljuje u prvoj i drugoj koloni zato sto je 3dec=11bin.Valjda nisam bio konfuzan.

Ovo bi moglo da se primeni u gornjem zadatku samo ako bi moglo da se upotrebi svih sedam tegova prilikom merenja i tada bi njihove tezine bile: 1, 3, 9, 27, 81, 243, 729, znaci brojni sistem s osnovom tri.

...upravo zato sam i zadao zadatak, Raborze. Upotrebom brojeva iz skupa {20, 21, 22...2n}, moze se zapisati bilo koji prirodni broj do 22n-1, kao zbir elemenata navedenog skupa, od kojih se svaki pojavljuje u jednacini ne vise od jedan put. Pretpostavljam da bi slicna logika mogla da se primeni i na tvoj sistem.

...U pravu si , ali kako tegovi mogu da budu na obe strane u ovom slucaju postoji i oduzimanje pa sam zato rekao da je brojni sistem s osnovom tri pravo resenje.Samo sto , kao sto i sam uvidjas, u zadatku je dozvoljeno koristiti maksimalno tri kamena od ponudjenih sedam prilikom svakog merenje, tako da sve prethodno napisano pada u vodu.

0 za
0 protiv
raborg
2010-09-16
???

Vaskrsija napisa...

@ Kalamist

zar ne bi bilo 2(n+1)-1

...naravno da je tako, pisem u brzini i pravim gluposti, a Supervizor i dalje ne dozvoljava edit postova :)

0 za
0 protiv
Kalimist
2010-09-16
???

@raborg

Ako pretpostavimo da su tegovi elementi određenog skupa, gdje bi svaki sledeći bio veći od prethodnog, onda bi imali

sedam jedinica, 21 uređen par, i 34 uređene trojke što znači da možemo pronaći ukupno 62 zbira, kako je moguće i oduzimanje tj. teret mase od 7 kg možemo imjeriti tako što na jedan tas stavimo teg od 9kg a na drugi teret i teg mase 2 kg, moguće je 50 razlika, i sad ako ni jedna nije nula, moguće je izmjeriti 112 tereta.Znam da ovo nije dobro, al nećemo doći do rješenja ako ne pokušamo.

0 za
0 protiv
MaLin
2010-09-16
???

Vaskrsija napisa...

@raborg

Ako pretpostavimo da su tegovi elementi određenog skupa, gdje bi svaki sledeći bio veći od prethodnog, onda bi imali

sedam jedinica, 21 uređen par, i 34 uređene trojke što znači da možemo pronaći ukupno 62 zbira, kako je moguće i oduzimanje tj. teret mase od 7 kg možemo imjeriti tako što na jedan tas stavimo teg od 9kg a na drugi teret i teg mase 2 kg, moguće je 50 razlika, i sad ako ni jedna nije nula, moguće je izmjeriti 112 tereta.Znam da ovo nije dobro, al nećemo doći do rješenja ako ne pokušamo.

...Vaskrsija, dobar ti je rezon, mada sam to svakako i sam pokusao.Tacno je da je kod sabiranja tj. kad tegove stavljamo na istu stranu moguce napraviti 7 jedinica, 21 dvojku i ipak 35 trojki.Medjutim, kod oduzimanja, pored 21 dvojke moguće su 3x35 trojki , sto daje ukupno 189 mogucih kombinacija.Problem sto je nemoguce odrediti takvih sedam tezina kojim bi bi bili pokriveni svi brojevi od 1 do 189.

Zamislimo da postoje ukupno tri tega.Tada bi ukupan broj kombinacija(malo je problematican izraz) bio  3+3+1+3+3=13 i bilo bi moguce dobiti sve te brojeve uz pomoc 1, 3, 9.U drugoj situaciji sa ukupno cetiri tega bilo bi 4+6+4+6+12=32 mogucnosti,ali nema sanse da se uz pomoc bilo koja cetiri broja pokriju sve one, a da ne govorimo o ovom nasem slucaju kada ih je sedam.Mislim da problem zadatka i jeste u tome sto ne postoji neki odredjen sistem koji bi mogao da se generalizuje na razlicit broj ponudjenih tegova.

Mene ovo muci s vremena na vreme, otkako sam video zadatak, a tome ima,uz rizik da ispadnem freak , bar tri godine.

0 za
0 protiv
raborg
2010-09-16
???

Da, u pravu si, kasnije sam i sam uočio da kad je razlika u pitanju, to mnogo komplikovanije, pogotovo što se može pojaviti nula, ili neka od jedinica koje već postoje, al u svakom slučaju, ne treba se predavati.

0 za
0 protiv
MaLin
2010-09-17
???

Evo jos jedan zanimljiv zadatak:

Ana je prvi utorak u mesecu provela u Beogradu, a prvi utorak posle
prvog ponedeljka u mesecu provela je u Novom Sadu. Sledeceg meseca,
Ana je prvu sredu provela u Kragujevcu, a prvu sredu posle prvog utorka
u mesecu na Zlatiboru. Gde je Ana te godine provela 8. mart?

2 za
1 protiv
mc80
2010-09-17
???

... На Златибору... те преступне године...

1 za
0 protiv
Цане Вукић
2010-09-17
???

I evo jos jedan lagan:

U fudbalskom prvenstvu jedne drzave takmici se 18 ekipa.Posle 6 odigranih kola ispostavilo se da sve ekipe imaju razlicit broj bodova.Koliko je ukupno neresenih rezultata bilo u tih 6 kola?(u slucaju da ima neko da ne zna za svaku pobedu ekipa dobija 3 boda,za nereseno  bod,za poraz 0 bodova)

2 za
0 protiv
mc80
2010-09-17
???

Петар Марковић napisa...

... На Златибору... те преступне године...

To je tacan odgovor!Verovatno ti je naporno bilo kucati kompletno resenje,ali nema veze ovako si ostavio i ostalima prostora da dodju do njega.Jos jednom,bravo!

1 za
1 protiv
mc80
2010-09-17
???

mc80 napisa...

I evo jos jedan lagan:

U fudbalskom prvenstvu jedne drzave takmici se 18 ekipa.Posle 6 odigranih kola ispostavilo se da sve ekipe imaju razlicit broj bodova.Koliko je ukupno neresenih rezultata bilo u tih 6 kola?(u slucaju da ima neko da ne zna za svaku pobedu ekipa dobija 3 boda,za nereseno  bod,za poraz 0 bodova)

...Bilo je minimum 9 neresenih rezultata ... 

0 za
0 protiv
dejan86
2010-09-17
???

mc80 napisa...

I evo jos jedan lagan:

U fudbalskom prvenstvu jedne drzave takmici se 18 ekipa.Posle 6 odigranih kola ispostavilo se da sve ekipe imaju razlicit broj bodova.Koliko je ukupno neresenih rezultata bilo u tih 6 kola?(u slucaju da ima neko da ne zna za svaku pobedu ekipa dobija 3 boda,za nereseno  bod,za poraz 0 bodova)

...Trebalo bi da ih je osam.Posto je najveci broj poena za prvu ekipu 18, a nemoguce je imati 17, znaci da su ostali brojevi poena 0-16.Da bi se to zadovoljilo 5 ekipa treba da ima po 2 remija a sest po 1, sto daje ukupno 8.

  Bez obzira sto se mc80 nece sloziti , zna se ko ce kod nas da bude na prvom mestu u datom trenutku.

1 za
0 protiv
raborg
2010-09-17
???

raborg napisa...

mc80 napisa...

I evo jos jedan lagan:

U fudbalskom prvenstvu jedne drzave takmici se 18 ekipa.Posle 6 odigranih kola ispostavilo se da sve ekipe imaju razlicit broj bodova.Koliko je ukupno neresenih rezultata bilo u tih 6 kola?(u slucaju da ima neko da ne zna za svaku pobedu ekipa dobija 3 boda,za nereseno  bod,za poraz 0 bodova)

...Trebalo bi da ih je osam.Posto je najveci broj poena za prvu ekipu 18, a nemoguce je imati 17, znaci da su ostali brojevi poena 0-16.Da bi se to zadovoljilo 5 ekipa treba da ima po 2 remija a sest po 1, sto daje ukupno 8.

  Bez obzira sto se mc80 nece sloziti , zna se ko ce kod nas da bude na prvom mestu u datom trenutku.

Hm,kako si ovo zakljucio.Malo pojasni,jer recimo ovako na prvi pogled moze se desiti da ekipa koja ima 12 bodova ima 3 pobede i 3 remija!?

Sto se drugog dela tice,videcemo,mada nasa liga ima 16 klubova.wink

0 za
0 protiv
mc80
2010-09-17
???

mc80 napisa...

raborg napisa...

mc80 napisa...

I evo jos jedan lagan:

U fudbalskom prvenstvu jedne drzave takmici se 18 ekipa.Posle 6 odigranih kola ispostavilo se da sve ekipe imaju razlicit broj bodova.Koliko je ukupno neresenih rezultata bilo u tih 6 kola?(u slucaju da ima neko da ne zna za svaku pobedu ekipa dobija 3 boda,za nereseno  bod,za poraz 0 bodova)

...Trebalo bi da ih je osam.Posto je najveci broj poena za prvu ekipu 18, a nemoguce je imati 17, znaci da su ostali brojevi poena 0-16.Da bi se to zadovoljilo 5 ekipa treba da ima po 2 remija a sest po 1, sto daje ukupno 8.

  Bez obzira sto se mc80 nece sloziti , zna se ko ce kod nas da bude na prvom mestu u datom trenutku.

Hm,kako si ovo zakljucio.Malo pojasni,jer recimo ovako na prvi pogled moze se desiti da ekipa koja ima 12 bodova ima 3 pobede i 3 remija!?

Sto se drugog dela tice,videcemo,mada nasa liga ima 16 klubova.wink

...U pravu si, nisam do kraja pojasnio.Pojavljuje se dilema kod pojedinih brojeva, kako ih dobiti posto moze na dva nacina (sem sestice koja moze na 3).Medjutim, obzirom da je ukupan broj osvojenih poena u svim odigranim kolima zbir od 1 do 18, izuzevsi 17, dolazimo do brojke od 154 poena.A posto je maksimalan broj poena, u slucaju da su bile sve pobede 162, automatski znaci da je bilo tacno osam remija.

  Onim podatkom od ranije da ima pet ekipa sa po 2 i sest sa po jednim remijem, ja sam samo rekao kako je jedino moguce rasporediti tih osam neresenih utakmica, a ne i zasto.Jel sad Ok.

0 za
0 protiv
raborg
2010-09-17
???

raborg napisa...

mc80 napisa...

raborg napisa...

mc80 napisa...

I evo jos jedan lagan:

U fudbalskom prvenstvu jedne drzave takmici se 18 ekipa.Posle 6 odigranih kola ispostavilo se da sve ekipe imaju razlicit broj bodova.Koliko je ukupno neresenih rezultata bilo u tih 6 kola?(u slucaju da ima neko da ne zna za svaku pobedu ekipa dobija 3 boda,za nereseno  bod,za poraz 0 bodova)

...Trebalo bi da ih je osam.Posto je najveci broj poena za prvu ekipu 18, a nemoguce je imati 17, znaci da su ostali brojevi poena 0-16.Da bi se to zadovoljilo 5 ekipa treba da ima po 2 remija a sest po 1, sto daje ukupno 8.

  Bez obzira sto se mc80 nece sloziti , zna se ko ce kod nas da bude na prvom mestu u datom trenutku.

Hm,kako si ovo zakljucio.Malo pojasni,jer recimo ovako na prvi pogled moze se desiti da ekipa koja ima 12 bodova ima 3 pobede i 3 remija!?

Sto se drugog dela tice,videcemo,mada nasa liga ima 16 klubova.wink

...U pravu si, nisam do kraja pojasnio.Pojavljuje se dilema kod pojedinih brojeva, kako ih dobiti posto moze na dva nacina (sem sestice koja moze na 3).Medjutim, obzirom da je ukupan broj osvojenih poena u svim odigranim kolima zbir od 1 do 18, izuzevsi 17, dolazimo do brojke od 154 poena.A posto je maksimalan broj poena, u slucaju da su bile sve pobede 162, automatski znaci da je bilo tacno osam remija.

  Onim podatkom od ranije da ima pet ekipa sa po 2 i sest sa po jednim remijem, ja sam samo rekao kako je jedino moguce rasporediti tih osam neresenih utakmica, a ne i zasto.Jel sad Ok.

Da.Sad je u redu.Bravo.

0 za
0 protiv
mc80
2010-09-17
???

Posto je Tipsa konacno pobedio, da se bacimo malo na zadatke.Uz napomenu da ne izgubite iz vida onaj , sa sedam tegova.

  Maja i Ana imju zajedno 44 godine.Maja ima dva puta toliko godina koliko je imala Ana, kada je Maji bilo upola toliko godina koliko ce biti Ani, kada Ani bude tri puta toliko godina koliko je bilo Maji, kada je Maji bilo tri puta toliko godina koliko je bilo Ani.Koliko godina ima Maja?

0 za
0 protiv
raborg
2010-09-17
???

raborg napisa...

Posto je Tipsa konacno pobedio, da se bacimo malo na zadatke.Uz napomenu da ne izgubite iz vida onaj , sa sedam tegova.

  Maja i Ana imju zajedno 44 godine.Maja ima dva puta toliko godina koliko je imala Ana, kada je Maji bilo upola toliko godina koliko ce biti Ani, kada Ani bude tri puta toliko godina koliko je bilo Maji, kada je Maji bilo tri puta toliko godina koliko je bilo Ani.Koliko godina ima Maja?

...Pa ovo ne bi rješio ni Izet Fazlinović

0 za
2 protiv
cicvara
2010-09-17
???

raborg napisa...

Posto je Tipsa konacno pobedio, da se bacimo malo na zadatke.Uz napomenu da ne izgubite iz vida onaj , sa sedam tegova.

  Maja i Ana imju zajedno 44 godine.Maja ima dva puta toliko godina koliko je imala Ana, kada je Maji bilo upola toliko godina koliko ce biti Ani, kada Ani bude tri puta toliko godina koliko je bilo Maji, kada je Maji bilo tri puta toliko godina koliko je bilo Ani.Koliko godina ima Maja?

27,5.

0 za
0 protiv
mc80
2010-09-17
???

mc80 napisa...

raborg napisa...

Posto je Tipsa konacno pobedio, da se bacimo malo na zadatke.Uz napomenu da ne izgubite iz vida onaj , sa sedam tegova.

  Maja i Ana imju zajedno 44 godine.Maja ima dva puta toliko godina koliko je imala Ana, kada je Maji bilo upola toliko godina koliko ce biti Ani, kada Ani bude tri puta toliko godina koliko je bilo Maji, kada je Maji bilo tri puta toliko godina koliko je bilo Ani.Koliko godina ima Maja?

27,5.

...Bravo majstore

2 za
0 protiv
raborg
2010-09-17
???

Pretpostavljam da velika vecina zna odgovor na pitanje:"Koje je boje medved koji prolazi pored kuće cije su sve cetiri strane okrenute prema jugu?".Ili:"Lovac, napustivsi svoj sator, krene kilometar na jug, zatim se okrene i krene kilometar na istok , pa opet okrene i podje kilometar na sever i shvati da je tako stigao tacno do svog satora.Ponovo se trazi boja medveda, kojeg je ugledao kako mu rastura sator"

  Pravo pitanje glasi : "Uzevsi u obzir lovcev nacin kretanja,dakle,po kilometar na jug, istok i sever,gde se nalazi jos neograniceno mnogo mesta, izuzev onog u gornjim pitalicama,sa kojih je moguce krenuti i vratiti se na isto mesto?"

0 za
0 protiv
raborg
2010-09-18
???

raborg napisa...

Pretpostavljam da velika vecina zna odgovor na pitanje:"Koje je boje medved koji prolazi pored kuće cije su sve cetiri strane okrenute prema jugu?".Ili:"Lovac, napustivsi svoj sator, krene kilometar na jug, zatim se okrene i krene kilometar na istok , pa opet okrene i podje kilometar na sever i shvati da je tako stigao tacno do svog satora.Ponovo se trazi boja medveda, kojeg je ugledao kako mu rastura sator"

  Pravo pitanje glasi : "Uzevsi u obzir lovcev nacin kretanja,dakle,po kilometar na jug, istok i sever,gde se nalazi jos neograniceno mnogo mesta, izuzev onog u gornjim pitalicama,sa kojih je moguce krenuti i vratiti se na isto mesto?"

...Očigledno je da se lovac, tj. šator nalazi na sjevernom polu, tj. boja medvjeda je bjele boje.

0 za
0 protiv
MaLin
2010-09-18
???

To jesto ocigledno ali to nije bilo pitanje.

0 za
0 protiv
raborg
2010-09-18
???

raborg napisa...

Pretpostavljam da velika vecina zna odgovor na pitanje:"Koje je boje medved koji prolazi pored kuće cije su sve cetiri strane okrenute prema jugu?".Ili:"Lovac, napustivsi svoj sator, krene kilometar na jug, zatim se okrene i krene kilometar na istok , pa opet okrene i podje kilometar na sever i shvati da je tako stigao tacno do svog satora.Ponovo se trazi boja medveda, kojeg je ugledao kako mu rastura sator"

  Pravo pitanje glasi : "Uzevsi u obzir lovcev nacin kretanja,dakle,po kilometar na jug, istok i sever,gde se nalazi jos neograniceno mnogo mesta, izuzev onog u gornjim pitalicama,sa kojih je moguce krenuti i vratiti se na isto mesto?"

...Na juznoj polulopti. Bilo koja tacka koja je 1 km sjeverno od paralele cija je ukupna duzina 1 km

0 za
0 protiv
tiii
2010-09-18
???

raborg napisa...

Pretpostavljam da velika vecina zna odgovor na pitanje:"Koje je boje medved koji prolazi pored kuće cije su sve cetiri strane okrenute prema jugu?".Ili:"Lovac, napustivsi svoj sator, krene kilometar na jug, zatim se okrene i krene kilometar na istok , pa opet okrene i podje kilometar na sever i shvati da je tako stigao tacno do svog satora.Ponovo se trazi boja medveda, kojeg je ugledao kako mu rastura sator"

  Pravo pitanje glasi : "Uzevsi u obzir lovcev nacin kretanja,dakle,po kilometar na jug, istok i sever,gde se nalazi jos neograniceno mnogo mesta, izuzev onog u gornjim pitalicama,sa kojih je moguce krenuti i vratiti se na isto mesto?"

Ako planetu Zemlju zamislimo kao loptu sa centrom u tacki O,a tacku severnog pola oznacimo sa  S,onda trazeno svojstvo ima svaka tacka T na polupravoj OS za koju vazi O-S-T.

3 za
0 protiv
mc80
2010-09-18
???

@ raborg

ono juče je bila šala, u stvari htio sam da kažem da je zadatak zaista bio dobar i da je trebalo biti majstor da se protumači i postavi na pravi način.

@mc80

sve čestitke na rješenju istog, a takođe se slažem i za ovo što si napiso u vezi poslednjeg, jedan klik od mene

E sad ako u svijetu matematike postoji zlatni trougao, a čine ga (ako se ne varam) Njutn, Arhimed i Gaus, onad u nasoj slagalici postoji zlatni četverougao, a čine ga Kalimist,raborg,Petar Marković i mc80.

naravno, ima još dobrih matematičara (slaki82, modesty itd.) ali po mom mišljenju prednjače gore pomenuta četvorica, e još bi samo zamolio one koje rješe zadatak da pokažu i postupak po kojem su to uradili, kao što je mc80 objasni poslednji.

Pozdrav za sve.

0 za
0 protiv
cicvara
2010-09-18
???

SLAKI82 napisa...

 

Evo jednog laganog zadatka:

 

Krava 2,ovca 2,kukavica 4,petao-?

Zameni znak pitanja brojem i objasni sta on oznacava.

...tacan odgovor je 8,prepustam vama da obrazlozite.wink

1 za
0 protiv
Слађан
2010-09-18
???

tiii napisa...

raborg napisa...

Pretpostavljam da velika vecina zna odgovor na pitanje:"Koje je boje medved koji prolazi pored kuće cije su sve cetiri strane okrenute prema jugu?".Ili:"Lovac, napustivsi svoj sator, krene kilometar na jug, zatim se okrene i krene kilometar na istok , pa opet okrene i podje kilometar na sever i shvati da je tako stigao tacno do svog satora.Ponovo se trazi boja medveda, kojeg je ugledao kako mu rastura sator"

  Pravo pitanje glasi : "Uzevsi u obzir lovcev nacin kretanja,dakle,po kilometar na jug, istok i sever,gde se nalazi jos neograniceno mnogo mesta, izuzev onog u gornjim pitalicama,sa kojih je moguce krenuti i vratiti se na isto mesto?"

...Na juznoj polulopti. Bilo koja tacka koja je 1 km sjeverno od paralele cija je ukupna duzina 1 km

...Bravo, tako je.

0 za
0 protiv
raborg
2010-09-18
???

mc80 napisa...

raborg napisa...

Pretpostavljam da velika vecina zna odgovor na pitanje:"Koje je boje medved koji prolazi pored kuće cije su sve cetiri strane okrenute prema jugu?".Ili:"Lovac, napustivsi svoj sator, krene kilometar na jug, zatim se okrene i krene kilometar na istok , pa opet okrene i podje kilometar na sever i shvati da je tako stigao tacno do svog satora.Ponovo se trazi boja medveda, kojeg je ugledao kako mu rastura sator"

  Pravo pitanje glasi : "Uzevsi u obzir lovcev nacin kretanja,dakle,po kilometar na jug, istok i sever,gde se nalazi jos neograniceno mnogo mesta, izuzev onog u gornjim pitalicama,sa kojih je moguce krenuti i vratiti se na isto mesto?"

Ako planetu Zemlju zamislimo kao loptu sa centrom u tacki O,a tacku severnog pola oznacimo sa  S,onda trazeno svojstvo ima svaka tacka T na polupravoj OS za koju vazi O-S-T.

...Nisam te bas najbolje razumeo.Ne mislis valjda na mesta u unutrasnjosti lopte tj. Zemlje.

0 za
0 protiv
raborg
2010-09-18
???

Kretanje lovca podsjeća na kretanje robota u ravni koji se okreće pod uglom od 90 stepeni, tj ako krenemo od sjeverno pola i pređemo put dužine r, ako se posle toga okrenemo pod uglom od 90 stepeni, recimo desni i krećemo prvolinijski, nebitno je koliko pređemo, uvjek ćemo biti na rastojanju r od sjevernog pola, da bi se vratili tamo odakle smo pošli, opet se moramo okrenuti ka sjeveru (desno) pod uglo od 90 stepeni.

0 za
0 protiv
MaLin
2010-09-18
???

raborg napisa...

mc80 napisa...

raborg napisa...

Pretpostavljam da velika vecina zna odgovor na pitanje:"Koje je boje medved koji prolazi pored kuće cije su sve cetiri strane okrenute prema jugu?".Ili:"Lovac, napustivsi svoj sator, krene kilometar na jug, zatim se okrene i krene kilometar na istok , pa opet okrene i podje kilometar na sever i shvati da je tako stigao tacno do svog satora.Ponovo se trazi boja medveda, kojeg je ugledao kako mu rastura sator"

  Pravo pitanje glasi : "Uzevsi u obzir lovcev nacin kretanja,dakle,po kilometar na jug, istok i sever,gde se nalazi jos neograniceno mnogo mesta, izuzev onog u gornjim pitalicama,sa kojih je moguce krenuti i vratiti se na isto mesto?"

Ako planetu Zemlju zamislimo kao loptu sa centrom u tacki O,a tacku severnog pola oznacimo sa  S,onda trazeno svojstvo ima svaka tacka T na polupravoj OS za koju vazi O-S-T.

...Nisam te bas najbolje razumeo.Ne mislis valjda na mesta u unutrasnjosti lopte tj. Zemlje.

Ne.Mislim na mesta u vazduhu iznad povrsine Zemlje.

0 za
0 protiv
mc80
2010-09-18
???

Premda nije eksplicitno naznaceno da se traze mesta na povrsini Zemlje, ipak mislim da je bilo potpuno jasno da se i lovac i njegov sator nalaze na zemlji.Bilo kako bilo, tiii je odgovorio(la) savrseno tacno.

0 za
0 protiv
raborg
2010-09-18
???

raborg napisa...

Premda nije eksplicitno naznaceno da se traze mesta na povrsini Zemlje, ipak mislim da je bilo potpuno jasno da se i lovac i njegov sator nalaze na zemlji.Bilo kako bilo, tiii je odgovorio(la) savrseno tacno.

OK.Ja sam  shvatio bilo koja mesta,pa me je to odvelo skroz u drugom pravcu.Da je stajalo..."sa kojih je lovac mogao krenuti",umesto ..."sa kojih je moguce krenuti",verovatno ne bi razmisljao u ovom pravcu.

1 za
0 protiv
mc80
2010-09-18
???

SLAKI82 napisa...

SLAKI82 napisa...

 

Evo jednog laganog zadatka:

 

Krava 2,ovca 2,kukavica 4,petao-?

Zameni znak pitanja brojem i objasni sta on oznacava.

...tacan odgovor je 8,prepustam vama da obrazlozite.wink

...krava-mu-2

ovca-be-2

kukavica-kuku-4

petao-kukuriku-8

2 za
0 protiv
tiii
2010-09-18
???

tiii napisa...

SLAKI82 napisa...

SLAKI82 napisa...

 

Evo jednog laganog zadatka:

 

Krava 2,ovca 2,kukavica 4,petao-?

Zameni znak pitanja brojem i objasni sta on oznacava.

...tacan odgovor je 8,prepustam vama da obrazlozite.wink

...krava-mu-2

ovca-be-2

kukavica-kuku-4

petao-kukuriku-8

...Bravo!

1 za
0 protiv
Слађан
2010-09-18
???

Na pravougaonom stolu rasporedjeno je(bez ikakvog prekrivanja) 25 novcica,tako da nije moguce postaviti jos jedan novcic koji ne bi delimicno pekrivao neki od vec rasporedjenih novcica.Dokazati da sto moze biti prekriven sa 100 novcica.(Novcici su kruznog oblika jednakih poluprecnika.Smatra se da je novcic na stolu,ako je njegov centar na stolu.)

0 za
0 protiv
mc80
2010-09-19
???

mc80 napisa...

Na pravougaonom stolu rasporedjeno je(bez ikakvog prekrivanja) 25 novcica,tako da nije moguce postaviti jos jedan novcic koji ne bi delimicno pekrivao neki od vec rasporedjenih novcica.Dokazati da sto moze biti prekriven sa 100 novcica.(Novcici su kruznog oblika jednakih poluprecnika.Smatra se da je novcic na stolu,ako je njegov centar na stolu.)

...Mc80, nisam bas najbolje razumeo zadatak.Da li treba dodati jos 75 novcica na postojecu situaciju tako da se povrsina stola ne vidi,ili rasporediti ponovo svih 100 novcica tako da se pokrije cela povrsina, ili trece, dokazati da se na sto moze postaviti 100 novcica tako da se ne preklapaju.

0 za
0 protiv
raborg
2010-09-20
???

raborg napisa...

mc80 napisa...

Na pravougaonom stolu rasporedjeno je(bez ikakvog prekrivanja) 25 novcica,tako da nije moguce postaviti jos jedan novcic koji ne bi delimicno pekrivao neki od vec rasporedjenih novcica.Dokazati da sto moze biti prekriven sa 100 novcica.(Novcici su kruznog oblika jednakih poluprecnika.Smatra se da je novcic na stolu,ako je njegov centar na stolu.)

...Mc80, nisam bas najbolje razumeo zadatak.Da li treba dodati jos 75 novcica na postojecu situaciju tako da se povrsina stola ne vidi,ili rasporediti ponovo svih 100 novcica tako da se pokrije cela povrsina, ili trece, dokazati da se na sto moze postaviti 100 novcica tako da se ne preklapaju.

Ovo sto sam ti oznacio plavom bojom treba dokazati.

0 za
0 protiv
mc80
2010-09-20
???

Da probam: Ako zamislimo da je sto kvadratnog oblika 10x10, a novcici po 1, onda cemo lako smestiti 100 novcica ako ih samo poslazemo jedan do drugoga.

Veci problem je kako postaviti 25, pa da nema mesta za jos neki. Pocnemo da redjamo novcice od 0,9, tako da ni sa jedne stane ne mozemo da dodamo novcic. Razmak izmedju njih bude 0,8 i opet nema mesta za jos neki novcic.( 2x 0,9 + 5x1 + 4x0,8 = 10)

Nadam se da ste me razumeli. Matematiku ne znam pa ne znam ni da postavim zadatak ni da izvedem dokaz. Mislim da je resenje u redu iako sam isla obrnutim redom.

0 za
0 protiv
petmarija
2010-09-20
???

petmarija napisa...

Da probam: Ako zamislimo da je sto kvadratnog oblika 10x10, a novcici po 1, onda cemo lako smestiti 100 novcica ako ih samo poslazemo jedan do drugoga.

Veci problem je kako postaviti 25, pa da nema mesta za jos neki. Pocnemo da redjamo novcice od 0,9, tako da ni sa jedne stane ne mozemo da dodamo novcic. Razmak izmedju njih bude 0,8 i opet nema mesta za jos neki novcic.( 2x 0,9 + 5x1 + 4x0,8 = 10)

Nadam se da ste me razumeli. Matematiku ne znam pa ne znam ni da postavim zadatak ni da izvedem dokaz. Mislim da je resenje u redu iako sam isla obrnutim redom.

Ne znam ni sam sta bih ti rekao na ovo.Izvini,ali potpuno si omanula,nazalost.sad

2 za
0 protiv
mc80
2010-09-20
???

mc80 napisa...

Na pravougaonom stolu rasporedjeno je(bez ikakvog prekrivanja) 25 novcica,tako da nije moguce postaviti jos jedan novcic koji ne bi delimicno pekrivao neki od vec rasporedjenih novcica.Dokazati da sto moze biti prekriven sa 100 novcica.(Novcici su kruznog oblika jednakih poluprecnika.Smatra se da je novcic na stolu,ako je njegov centar na stolu.)

...Poenta je pronaci najvece moguce dimenzije stola , pri opisanim uslovima kad je 25 novcica na stolu, a onda pokazati da 100 novcica moze prekriti taj sto.Uzecemo da je poluprecnik novcica r i zamislicemo krug oko njegovog centra koji ima poluprecnik 2r.Raspored novcica poluprecnika r takav da ne moze da sa postavi vise nijedan novcic moze se zameniti rasporedom krugova(oko tih novcica) poluprecnika manjeg od 2r (za delic manjeg) tako da njima bude pokriven ceo sto.Idealan nacin za raspored krugova je oblik saća odnosno oko kruga u centru simetricno se postavi sest krugova tako da se izmedju njih ne vidi nikakav razmak tj. potpuno pokrivaju povrsinu stola na delu na kome se nalaze.Uz sve prethodno napisano najvece dimenzije stola ,koji je potpuno prekriven krugovima poluprecnika nesto manjeg od 2r ,odnosno na kome se nalaze novcici poluprecnika r je <9sqrt(3)r x <14r.Raspored je 5x5.

  S druge strane,  kada 100 novcica u rasporedu 10x10 poredjamo na nacin kako su bili poredjani i krugovi , moze se prekriti sto dimenzija 19/2sqrt(3)r x 29/2.

  Nisam hteo da smaram izracunavanjem samih dimenzija, ali ako treba moze.Mozda postoji i neki drugi, jednostavniji nacin al` se nadam da je i ovo Ok.

0 za
0 protiv
raborg
2010-09-21
???

raborg napisa...

mc80 napisa...

Na pravougaonom stolu rasporedjeno je(bez ikakvog prekrivanja) 25 novcica,tako da nije moguce postaviti jos jedan novcic koji ne bi delimicno pekrivao neki od vec rasporedjenih novcica.Dokazati da sto moze biti prekriven sa 100 novcica.(Novcici su kruznog oblika jednakih poluprecnika.Smatra se da je novcic na stolu,ako je njegov centar na stolu.)

...Poenta je pronaci najvece moguce dimenzije stola , pri opisanim uslovima kad je 25 novcica na stolu, a onda pokazati da 100 novcica moze prekriti taj sto.Uzecemo da je poluprecnik novcica r i zamislicemo krug oko njegovog centra koji ima poluprecnik 2r.Raspored novcica poluprecnika r takav da ne moze da sa postavi vise nijedan novcic moze se zameniti rasporedom krugova(oko tih novcica) poluprecnika manjeg od 2r (za delic manjeg) tako da njima bude pokriven ceo sto.Idealan nacin za raspored krugova je oblik saća odnosno oko kruga u centru simetricno se postavi sest krugova tako da se izmedju njih ne vidi nikakav razmak tj. potpuno pokrivaju povrsinu stola na delu na kome se nalaze.Uz sve prethodno napisano najvece dimenzije stola ,koji je potpuno prekriven krugovima poluprecnika nesto manjeg od 2r ,odnosno na kome se nalaze novcici poluprecnika r je <9sqrt(3)r x <14r.Raspored je 5x5.

  S druge strane,  kada 100 novcica u rasporedu 10x10 poredjamo na nacin kako su bili poredjani i krugovi , moze se prekriti sto dimenzija 19/2sqrt(3)r x 29/2.

  Nisam hteo da smaram izracunavanjem samih dimenzija, ali ako treba moze.Mozda postoji i neki drugi, jednostavniji nacin al` se nadam da je i ovo Ok.

Ovo sto sam oznacio plavim:Ta zamena nije bijektivna!?Poluprecnik u nekim slucajevima moze biti i manji.Ovo sto sam oznacio crvenim:Zasto je taj raspored idealan!?

P.S.Moguce da je resenje i dobro,samo kada bi ga malo bolje pojasnio,tj.njegovu sustinu,samo izracunavanje me ne zanima,iskreno ovo sto si napisao nisam bas najbolje mogao da skontam!

0 za
0 protiv
mc80
2010-09-21
???

Evo nekoliko zanimljivih zadataka sto mi jutros padose na pamet(zadaci su poredjani od najlakseg ka najtezem):

1.Na jednoj gomili stoji 10,a na drugoj 20 zetona.Dva igraca A i B igraju sledecu igru:Najpre igrac A uzme izvesan broj zetona sa jedne od gomila,a zatim u sledecem potezu to isto uradi igrac B.Pobednik je igrac koji uzme poslednji zeton.Koji igrac u ovoj igri ima pobednicku strategiju i kako treba da igra?

 

2.Koja je verovatnoca veca,da pri bacanju 3 kocke za jamb padne zbir 11 ili 12?

 

3.Da li se krug poluprecnika 5 moze prekriti sa tri kruga poluprecnika 2,3 i 4?

 

4.Na gomili stoji 100 zetona..Dva igraca A i B igraju sledecu igru:Najpre igrac A uzme izvesan broj zetona sa gomile,a zatim u sledecem potezu  igrac B moze da  uzme broj zetona takav da je taj broj delilac broja zetona koji je uzeo igrac B.U sledecem potezu igrac A uzima broj zetona koji je delilac broja zetona koji je uzeo igrac B itd.Pobednik je igrac koji uzme poslednji zeton.Koji igrac u ovoj igri ima pobednicku strategiju i kako treba da igra?

 

5.Data je kvadratna tablica 5x5.Dva igraca A i B igraju sledecu igru:Najpre igrac A u tablicu upise neki broj iz skupa 1,2,3...,24,25,a zatim to isto ucini igrac B.Igra se zavrsava kada svi brojevi iz pomenutog skupa budu upisani u tablicu.Pobednik je igrac A,ako se u nekoj vrsti ili koloni tablici nakon upisivanja svih brojeva pojavi zbir 70,u suprotnom pobednik je igrac B.Koji igrac u ovoj igri ima pobednicku strategiju i kako treba da igra?

0 za
0 protiv
mc80
2010-09-21
???

Нешто на брзину:

2. (3+4+5+6+5+4)/216+(2+3+4+5+6+5)/216=13/54

3. Не. Како год да поставимо (концентрично, или да имају заједничку тангенту, итд...) круг полупречника 4 у круг полупречника 5, непокривена површина ће имати тачке које су на растојању 2х5=10, а које морамо покрити са круговима полупречника 2 и 3. ''Најбоље'' како можемо ''растегнути'' 2 и 3 је да се додирују споља (2х2+2х3=10), али у том случају остају непокривени ''бочни'' делови великог круга... Је л' то то?

0 za
0 protiv
Цане Вукић
2010-09-21
???

Ја, дебил, рачунао вероватноћу да падне 11 или 12. Већа је за 11 (први сабирак), него за 12. А, иста је за 11 и 10, 12 и 9 итд...

0 za
0 protiv
Цане Вукић
2010-09-21
???

Петар Марковић napisa...

Нешто на брзину:

2. (3+4+5+6+5+4)/216+(2+3+4+5+6+5)/216=13/54

3. Не. Како год да поставимо (концентрично, или да имају заједничку тангенту, итд...) круг полупречника 4 у круг полупречника 5, непокривена површина ће имати тачке које су на растојању 2х5=10, а које морамо покрити са круговима полупречника 2 и 3. ''Најбоље'' како можемо ''растегнути'' 2 и 3 је да се додирују споља (2х2+2х3=10), али у том случају остају непокривени ''бочни'' делови великог круга... Је л' то то?

2.Prvi sabirak je verovatnoca da padne 11,a drugi da padne 12.Znaci verovatnoca da padne 11 je veca .Dole,u sledecoj poruci si popravio resenje.To je tacno.Bravo!

3.Resenje sto si napisao je vise intuitivne prirode.Korektno matematickii ispravno rsenje bi izgledalo ovako:Neka su zajednicke tacke kruga sa poluprecnikom 4 i poluprecnikom 5 A i B.Tada je AB tetiva kruga popuprecnika 4,pa je tetiva manja ili jednak precniku kruga sa poluprecnikom 4 tj.AB je manje ili jednako 8.Tetivi AB u krugu poluprecnika 5 odgovara  luk duzine 2/5 obima tog kruga(to se lako sracuna).Slicno,krugovi poluprecnika 3 i 2 mogu da pokriju najvise 3/10 i 1/5 obima kruga poluprecnika 5 pa svi zajedno mogu da pokriju najvise 4/10+3/10+2/10=9/10 obima kruga poluprecnika5,tj.ne mogu ga pokriti celog.

0 za
0 protiv
mc80
2010-09-21
???

2. Ма, задатак нисам прочитао како треба први пут...

3. Додатно појашњење. Без обзира на то како је постављен круг полупречника 4, прво се повуче права која садржи центре кругова 4 и 5, а затим нормала на њу која пролази кроз центар круга полупречика 5 (пречнинк круга ''5'') . Ова нормала сече кружницу ''5'' у тачкама које су сигурно непокривене (лако доказиво), па да би се обе покриле, јер су на растојању 10, потребно је поставити ''2'' и ''3'' тако да им је нормала заједничка симетрала, а да имају по једну заједничку тачку са кружницом ''5'' (додирују је изнутра). Али, тако остаје непокривена бар једна од тачака (или обе, у зависности од тога да ли се кружице ''4'' и ''5'' секу) пресека праве и кружнице ''5''. Мислим да је ово и математички ваљан доказ, само што треба још текста, слика итд... А, није спорно, твоје решење је ''елегантније'', али не може ''без папира'' због израчунавања дужине тетива.

Иначе, нешто не добијам да је лук АВ 2/5 обима круга ''5''... И није ми баш тривијално, тј. треба arcsin(4/5)... indecision

0 za
0 protiv
Цане Вукић
2010-09-21
???

mc80 napisa...

raborg napisa...

mc80 napisa...

Na pravougaonom stolu rasporedjeno je(bez ikakvog prekrivanja) 25 novcica,tako da nije moguce postaviti jos jedan novcic koji ne bi delimicno pekrivao neki od vec rasporedjenih novcica.Dokazati da sto moze biti prekriven sa 100 novcica.(Novcici su kruznog oblika jednakih poluprecnika.Smatra se da je novcic na stolu,ako je njegov centar na stolu.)

...Poenta je pronaci najvece moguce dimenzije stola , pri opisanim uslovima kad je 25 novcica na stolu, a onda pokazati da 100 novcica moze prekriti taj sto.Uzecemo da je poluprecnik novcica r i zamislicemo krug oko njegovog centra koji ima poluprecnik 2r.Raspored novcica poluprecnika r takav da ne moze da sa postavi vise nijedan novcic moze se zameniti rasporedom krugova(oko tih novcica) poluprecnika manjeg od 2r (za delic manjeg) tako da njima bude pokriven ceo sto.Idealan nacin za raspored krugova je oblik saća odnosno oko kruga u centru simetricno se postavi sest krugova tako da se izmedju njih ne vidi nikakav razmak tj. potpuno pokrivaju povrsinu stola na delu na kome se nalaze.Uz sve prethodno napisano najvece dimenzije stola ,koji je potpuno prekriven krugovima poluprecnika nesto manjeg od 2r ,odnosno na kome se nalaze novcici poluprecnika r je <9sqrt(3)r x <14r.Raspored je 5x5.

  S druge strane,  kada 100 novcica u rasporedu 10x10 poredjamo na nacin kako su bili poredjani i krugovi , moze se prekriti sto dimenzija 19/2sqrt(3)r x 29/2.

  Nisam hteo da smaram izracunavanjem samih dimenzija, ali ako treba moze.Mozda postoji i neki drugi, jednostavniji nacin al` se nadam da je i ovo Ok.

Ovo sto sam oznacio plavim:Ta zamena nije bijektivna!?Poluprecnik u nekim slucajevima moze biti i manji.Ovo sto sam oznacio crvenim:Zasto je taj raspored idealan!?

P.S.Moguce da je resenje i dobro,samo kada bi ga malo bolje pojasnio,tj.njegovu sustinu,samo izracunavanje me ne zanima,iskreno ovo sto si napisao nisam bas najbolje mogao da skontam!

...Oko novcica poluprecnika r nije moguce postaviti drugi novcic ciji centar bi bio blizi od 2r od centra prvog novcica, a da se oni ne preklope.Dakle , "sigurna zona" oko svakog novcica je krug poluprecnika 2r.Ako tri novcica postavimo tako da im centri budu temena jednakostranicnog trougla stranice 2sqrt(3)r ,odnosno da se krugovi poluprecnika 2r opisani oko njih seku u tezistu tog trougla, moguce je staviti izmedju njih jos jedan novcic koji bi bio tangentan ostalima.Na ovakvoj slici ako se iole smanji stranica jednakostranicnog trougla,nemoguce je postaviti novcic u sredinu, sto implicira cinjenicu da, ako se krugovi opisani oko ova tri novcica seku tako da ne otkrivaju prazan prostor u delu u kome se nalaze nije moguce postaviti dodatni novcic izmedju njih.Kada bi nastavili da dodajemo novcice oko ova tri tako da centar svakog sledeceg cini jednakostranicni trougao sa centrima njemu dva najbliza novcica , dobili bi najgori moguci scenario rasporeda novcica(a uocava se da raspored krugova oko novcica ima oblik "saća").Pri tom treba primetiti da krugovi opisani oko novcica pokrivaju svaki deo prostora.Zato sam situaciju s 25 novcica zamenio ovom s 25 krugova koji pokrivaju ceo sto.I zato sam rekao da je idealan nacin za rapored krugova ,kada se zeli pokriti sto veca povrsina, onaj kada je jedan krug u sredini a sest preostalih poredjanih oko njega sa centrima u temenima jednakostranicnog setougla stranice ne vece od 2sqrt(3)r.Pri svemu ovome izracunao sam da je najveca moguca povrsina stola pokrivenog sa 100 novcica, veca od najvece moguce kada se na njemu nalazi 25 novcica izmedju kojih nije moguce umetnuti jos neki(odnosno veca od najvece moguce povrsine stola kada je on pokriven s 25 krugova poluprecnika 2r ).

0 za
0 protiv
raborg
2010-09-21
???

raborg napisa...

mc80 napisa...

raborg napisa...

mc80 napisa...

Na pravougaonom stolu rasporedjeno je(bez ikakvog prekrivanja) 25 novcica,tako da nije moguce postaviti jos jedan novcic koji ne bi delimicno pekrivao neki od vec rasporedjenih novcica.Dokazati da sto moze biti prekriven sa 100 novcica.(Novcici su kruznog oblika jednakih poluprecnika.Smatra se da je novcic na stolu,ako je njegov centar na stolu.)

...Poenta je pronaci najvece moguce dimenzije stola , pri opisanim uslovima kad je 25 novcica na stolu, a onda pokazati da 100 novcica moze prekriti taj sto.Uzecemo da je poluprecnik novcica r i zamislicemo krug oko njegovog centra koji ima poluprecnik 2r.Raspored novcica poluprecnika r takav da ne moze da sa postavi vise nijedan novcic moze se zameniti rasporedom krugova(oko tih novcica) poluprecnika manjeg od 2r (za delic manjeg) tako da njima bude pokriven ceo sto.Idealan nacin za raspored krugova je oblik saća odnosno oko kruga u centru simetricno se postavi sest krugova tako da se izmedju njih ne vidi nikakav razmak tj. potpuno pokrivaju povrsinu stola na delu na kome se nalaze.Uz sve prethodno napisano najvece dimenzije stola ,koji je potpuno prekriven krugovima poluprecnika nesto manjeg od 2r ,odnosno na kome se nalaze novcici poluprecnika r je <9sqrt(3)r x <14r.Raspored je 5x5.

  S druge strane,  kada 100 novcica u rasporedu 10x10 poredjamo na nacin kako su bili poredjani i krugovi , moze se prekriti sto dimenzija 19/2sqrt(3)r x 29/2.

  Nisam hteo da smaram izracunavanjem samih dimenzija, ali ako treba moze.Mozda postoji i neki drugi, jednostavniji nacin al` se nadam da je i ovo Ok.

Ovo sto sam oznacio plavim:Ta zamena nije bijektivna!?Poluprecnik u nekim slucajevima moze biti i manji.Ovo sto sam oznacio crvenim:Zasto je taj raspored idealan!?

P.S.Moguce da je resenje i dobro,samo kada bi ga malo bolje pojasnio,tj.njegovu sustinu,samo izracunavanje me ne zanima,iskreno ovo sto si napisao nisam bas najbolje mogao da skontam!

...Oko novcica poluprecnika r nije moguce postaviti drugi novcic ciji centar bi bio blizi od 2r od centra prvog novcica, a da se oni ne preklope.Dakle , "sigurna zona" oko svakog novcica je krug poluprecnika 2r.Ako tri novcica postavimo tako da im centri budu temena jednakostranicnog trougla stranice 2sqrt(3)r ,odnosno da se krugovi poluprecnika 2r opisani oko njih seku u tezistu tog trougla, moguce je staviti izmedju njih jos jedan novcic koji bi bio tangentan ostalima.Na ovakvoj slici ako se iole smanji stranica jednakostranicnog trougla,nemoguce je postaviti novcic u sredinu, sto implicira cinjenicu da, ako se krugovi opisani oko ova tri novcica seku tako da ne otkrivaju prazan prostor u delu u kome se nalaze nije moguce postaviti dodatni novcic izmedju njih.Kada bi nastavili da dodajemo novcice oko ova tri tako da centar svakog sledeceg cini jednakostranicni trougao sa centrima njemu dva najbliza novcica , dobili bi najgori moguci scenario rasporeda novcica(a uocava se da raspored krugova oko novcica ima oblik "saća").Pri tom treba primetiti da krugovi opisani oko novcica pokrivaju svaki deo prostora.Zato sam situaciju s 25 novcica zamenio ovom s 25 krugova koji pokrivaju ceo sto.I zato sam rekao da je idealan nacin za rapored krugova ,kada se zeli pokriti sto veca povrsina, onaj kada je jedan krug u sredini a sest preostalih poredjanih oko njega sa centrima u temenima jednakostranicnog setougla stranice ne vece od 2sqrt(3)r.Pri svemu ovome izracunao sam da je najveca moguca povrsina stola pokrivenog sa 100 novcica, veca od najvece moguce kada se na njemu nalazi 25 novcica izmedju kojih nije moguce umetnuti jos neki(odnosno veca od najvece moguce povrsine stola kada je on pokriven s 25 krugova poluprecnika 2r ).

Aha.Mislimda sam sad skontao sta si uradio.Evo sad cu ja izloziti jos jedno resenje ovog zadatka:

Dakle,ako imamo na stolu 25 novcica tako da vise ne mozemo postaviti novi a da ne preklapa neki os preostalih,onda taj sto mozemo prekriti u potpunosti novcicima poluprecnika 2r,ciji se centri poklapaju sa centrima vec postavljenih 25 novcica poluprecnika r.Ako bi postojala tacka koja nije prekrivena sa ovim "vecim" novcicima,onda bi rastojanje od centara novcica bilo vece od 2r,pa onda mozemo postaviti novcic poluprecnika r koji se ne preklapa sa postavljenih 25"malih"novcica,te dobijamo kontradikciju.Kako ceo sto mozemo prekriti sa 25 novcica poluprecnika 2r,to mozemo sto 2 puta manje sirine i duzine prekriti sa 25 novcica poluprenika r,a to je cetvrtina stola iz uslova zadatka ,te citav sto mozemo da prekrijemo sa 100 novcica poluprecnika r.

0 za
0 protiv
mc80
2010-09-21
???

Петар Марковић napisa...

2. Ма, задатак нисам прочитао како треба први пут...

3. Додатно појашњење. Без обзира на то како је постављен круг полупречника 4, прво се повуче права која садржи центре кругова 4 и 5, а затим нормала на њу која пролази кроз центар круга полупречика 5 (пречнинк круга ''5'') . Ова нормала сече кружницу ''5'' у тачкама које су сигурно непокривене (лако доказиво), па да би се обе покриле, јер су на растојању 10, потребно је поставити ''2'' и ''3'' тако да им је нормала заједничка симетрала, а да имају по једну заједничку тачку са кружницом ''5'' (додирују је изнутра). Али, тако остаје непокривена бар једна од тачака (или обе, у зависности од тога да ли се кружице ''4'' и ''5'' секу) пресека праве и кружнице ''5''. Мислим да је ово и математички ваљан доказ, само што треба још текста, слика итд... А, није спорно, твоје решење је ''елегантније'', али не може ''без папира'' због израчунавања дужине тетива.

indecision

U pravu si

Петар Марковић napisa...

2. Ма, задатак нисам прочитао како треба први пут...

3. Додатно појашњење. Без обзира на то како је постављен круг полупречника 4, прво се повуче права која садржи центре кругова 4 и 5, а затим нормала на њу која пролази кроз центар круга полупречика 5 (пречнинк круга ''5'') . Ова нормала сече кружницу ''5'' у тачкама које су сигурно непокривене (лако доказиво), па да би се обе покриле, јер су на растојању 10, потребно је поставити ''2'' и ''3'' тако да им је нормала заједничка симетрала, а да имају по једну заједничку тачку са кружницом ''5'' (додирују је изнутра). Али, тако остаје непокривена бар једна од тачака (или обе, у зависности од тога да ли се кружице ''4'' и ''5'' секу) пресека праве и кружнице ''5''. Мислим да је ово и математички ваљан доказ, само што треба још текста, слика итд... А, није спорно, твоје решење је ''елегантније'', али не може ''без папира'' због израчунавања дужине тетива.

Иначе, нешто не добијам да је лук АВ 2/5 обима круга ''5''... И није ми баш тривијално, тј. треба arcsin(4/5)... indecision

U pravu si.U brzini sam napravio veliki previd.Sad cu da izpravim.Znaci,krug poluprecnika 4,moze da pokriva najvise luk nad tetivom duzine 8,krug poluprecnika 3,luk nad tetivom duzine 6,a krug poluprecnika 2 luk nad tetivom duzine 4(misli se na tetive kruga poluprecnika 5).Posto je tetiva koja odgovara luku duzine trecine obims 5*sqrt(3)>8,znaci da prvi krug poluprecnika 4 pokriva manje od trecine obima kruga poluprecnika 5.Dalje,posto je tetiva koja odgovara luku duzine cetvrtine obima kruga polupr.5 je 5sqrt(2)>6,znaci da krug poluprecnika 3 pokriva manje od cetvrtine obima kruga poluprecnika 5.I na kraju,tetiva koja odgovara luku duzine sestine obima kruga polupr.5 iznosi 5>4 znaci da krug poluprecnika 2 pokriva manje od sestine obima kruga polupr.5.Sve u svemu ova 3 kruga pokrivaju manje od 1/3+1/4+1/6 obima polupr.5 a sto je ocigledno manje od obima tog kruga.

0 za
0 protiv
mc80
2010-09-21
???

mc80 napisa...

Evo nekoliko zanimljivih zadataka sto mi jutros padose na pamet(zadaci su poredjani od najlakseg ka najtezem):

1.Na jednoj gomili stoji 10,a na drugoj 20 zetona.Dva igraca A i B igraju sledecu igru:Najpre igrac A uzme izvesan broj zetona sa jedne od gomila,a zatim u sledecem potezu to isto uradi igrac B.Pobednik je igrac koji uzme poslednji zeton.Koji igrac u ovoj igri ima pobednicku strategiju i kako treba da igra?

 

2.Koja je verovatnoca veca,da pri bacanju 3 kocke za jamb padne zbir 11 ili 12?

 

3.Da li se krug poluprecnika 5 moze prekriti sa tri kruga poluprecnika 2,3 i 4?

 

4.Na gomili stoji 100 zetona..Dva igraca A i B igraju sledecu igru:Najpre igrac A uzme izvesan broj zetona sa gomile,a zatim u sledecem potezu  igrac B moze da  uzme broj zetona takav da je taj broj delilac broja zetona koji je uzeo igrac B.U sledecem potezu igrac A uzima broj zetona koji je delilac broja zetona koji je uzeo igrac B itd.Pobednik je igrac koji uzme poslednji zeton.Koji igrac u ovoj igri ima pobednicku strategiju i kako treba da igra?

 

5.Data je kvadratna tablica 5x5.Dva igraca A i B igraju sledecu igru:Najpre igrac A u tablicu upise neki broj iz skupa 1,2,3...,24,25,a zatim to isto ucini igrac B.Igra se zavrsava kada svi brojevi iz pomenutog skupa budu upisani u tablicu.Pobednik je igrac A,ako se u nekoj vrsti ili koloni tablici nakon upisivanja svih brojeva pojavi zbir 70,u suprotnom pobednik je igrac B.Koji igrac u ovoj igri ima pobednicku strategiju i kako treba da igra?

1. Igrac A moze uvijek pobijediti ako igra na sljedeci nacin: Prvo sa gomile na kojoj je 20 zetona uzima 10, tako da na obe gomile ostaje po 10 zetona. Zatim u svakom svom sljedecem potezu igrac A uzima isto onoliko zetona koliko je uzeo igrac B u prethodnom, samo sa druge gomile, tako da poslije svakog poteza igraca A na gomilama ostaje jednak broj zetona. Na kraju ce igrac B morati da uzme poslednji zeton (poslednje zetone) sa jedne od gomila, pa igrac A  uzima preostale sa druge gomile i pobjeduje.

0 za
0 protiv
tiii
2010-09-21
???

Pa Mc80, sad kad vidim tvoje resenje shvatam da nam je rezon bio skoro isti.Samo sto si ti elegantno izbegao da izracunas dimenzije stola a ja sam morao i to da uradim.Inace, zadatak je vrlo dobar.

1 za
0 protiv
raborg
2010-09-21
???

raborg napisa...

Pa Mc80, sad kad vidim tvoje resenje shvatam da nam je rezon bio skoro isti.Samo sto si ti elegantno izbegao da izracunas dimenzije stola a ja sam morao i to da uradim.Inace, zadatak je vrlo dobar.

tiii napisa...

mc80 napisa...

Evo nekoliko zanimljivih zadataka sto mi jutros padose na pamet(zadaci su poredjani od najlakseg ka najtezem):

1.Na jednoj gomili stoji 10,a na drugoj 20 zetona.Dva igraca A i B igraju sledecu igru:Najpre igrac A uzme izvesan broj zetona sa jedne od gomila,a zatim u sledecem potezu to isto uradi igrac B.Pobednik je igrac koji uzme poslednji zeton.Koji igrac u ovoj igri ima pobednicku strategiju i kako treba da igra?

 

2.Koja je verovatnoca veca,da pri bacanju 3 kocke za jamb padne zbir 11 ili 12?

 

3.Da li se krug poluprecnika 5 moze prekriti sa tri kruga poluprecnika 2,3 i 4?

 

4.Na gomili stoji 100 zetona..Dva igraca A i B igraju sledecu igru:Najpre igrac A uzme izvesan broj zetona sa gomile,a zatim u sledecem potezu  igrac B moze da  uzme broj zetona takav da je taj broj delilac broja zetona koji je uzeo igrac B.U sledecem potezu igrac A uzima broj zetona koji je delilac broja zetona koji je uzeo igrac B itd.Pobednik je igrac koji uzme poslednji zeton.Koji igrac u ovoj igri ima pobednicku strategiju i kako treba da igra?

 

5.Data je kvadratna tablica 5x5.Dva igraca A i B igraju sledecu igru:Najpre igrac A u tablicu upise neki broj iz skupa 1,2,3...,24,25,a zatim to isto ucini igrac B.Igra se zavrsava kada svi brojevi iz pomenutog skupa budu upisani u tablicu.Pobednik je igrac A,ako se u nekoj vrsti ili koloni tablici nakon upisivanja svih brojeva pojavi zbir 70,u suprotnom pobednik je igrac B.Koji igrac u ovoj igri ima pobednicku strategiju i kako treba da igra?

1. Igrac A moze uvijek pobijediti ako igra na sljedeci nacin: Prvo sa gomile na kojoj je 20 zetona uzima 10, tako da na obe gomile ostaje po 10 zetona. Zatim u svakom svom sljedecem potezu igrac A uzima isto onoliko zetona koliko je uzeo igrac B u prethodnom, samo sa druge gomile, tako da poslije svakog poteza igraca A na gomilama ostaje jednak broj zetona. Na kraju ce igrac B morati da uzme poslednji zeton (poslednje zetone) sa jedne od gomila, pa igrac A  uzima preostale sa druge gomile i pobjeduje.

Bravo,to je resenje!

0 za
0 protiv
mc80
2010-09-22
???

raborg napisa...

Pa Mc80, sad kad vidim tvoje resenje shvatam da nam je rezon bio skoro isti.Samo sto si ti elegantno izbegao da izracunas dimenzije stola a ja sam morao i to da uradim.Inace, zadatak je vrlo dobar.

Da,da ideja je ista sa posmatranjem novcica poluprecnika 2r.

0 za
0 protiv
mc80
2010-09-22
???

3 km

0 za
0 protiv
Đenka
2010-09-22
???

Reseni su zadaci 1,2,3 zasad.Ostala su jos dva-najteza.Ajde malo da pomognem.Prvi zadatak(broj 4) se resava tako sto treba osmisliti pobednicku strategiju za igraca A(pogadjanjem i kasnije analiziranjem utvrditi koji je broj zetona  najpovoljniji da se uzme u 1.potezu,a delom se koristi i ideja iz zadatka broj 1).U drugom zadatku(broj 5),opet treba osmisliti pobednicku strategiju za igraca A,koji nekako treba da izvrsi particiju ovog skupa na dvoclane podskupove i jedan jednoclan,tako da kasnije na svaki potez igraca B ima adekvatan "odgovor" .Eto,mislim da sam dosta pomogaosmiley(a i neka sam ova 2 zadatka su malo teza,mada ih moze resavati svako nezavisno od matematickog znanja koje poseduje).

 

4.Na gomili stoji 100 zetona..Dva igraca A i B igraju sledecu igru:Najpre igrac A uzme izvesan broj zetona sa gomile,a zatim u sledecem potezu  igrac B moze da  uzme broj zetona takav da je taj broj delilac broja zetona koji je uzeo igrac B.U sledecem potezu igrac A uzima broj zetona koji je delilac broja zetona koji je uzeo igrac B itd.Pobednik je igrac koji uzme poslednji zeton.Koji igrac u ovoj igri ima pobednicku strategiju i kako treba da igra?

 

5.Data je kvadratna tablica 5x5.Dva igraca A i B igraju sledecu igru:Najpre igrac A u tablicu upise neki broj iz skupa 1,2,3...,24,25,a zatim to isto ucini igrac B.Igra se zavrsava kada svi brojevi iz pomenutog skupa budu upisani u tablicu.Pobednik je igrac A,ako se u nekoj vrsti ili koloni tablici nakon upisivanja svih brojeva pojavi zbir 70,u suprotnom pobednik je igrac B.Koji igrac u ovoj igri ima pobednicku strategiju i kako treba da igra?

0 za
0 protiv
mc80
2010-09-22
???

Au,svaka vama cast ,taman skupim snage da vas citam - a vi napredujete dalje bez namere da stanete,gde ste bili celo leto dok sam dangubio :) ...

U jednom selu,berberin brije samo one koji se ne briju sami...Ko brije berberina?

0 za
0 protiv
NeparniValjak
2010-09-22
???

NeparniValjak napisa...

Au,svaka vama cast ,taman skupim snage da vas citam - a vi napredujete dalje bez namere da stanete,gde ste bili celo leto dok sam dangubio :) ...

U jednom selu,berberin brije samo one koji se ne briju sami...Ko brije berberina?

...Berberina brije izvjesni gospodin Rasel ( koji nije iz tog sela). :D

1 za
0 protiv
limes5
2010-09-22
???

NeparniValjak napisa...

Au,svaka vama cast ,taman skupim snage da vas citam - a vi napredujete dalje bez namere da stanete,gde ste bili celo leto dok sam dangubio :) ...

U jednom selu,berberin brije samo one koji se ne briju sami...Ko brije berberina?

...niko, jer je berberin zena.

1 za
0 protiv
tiii
2010-09-24
???

 

tiii napisa...

NeparniValjak napisa...

Au,svaka vama cast ,taman skupim snage da vas citam - a vi napredujete dalje bez namere da stanete,gde ste bili celo leto dok sam dangubio :) ...

U jednom selu,berberin brije samo one koji se ne briju sami...Ko brije berberina?

...niko, jer je berberin zena.

...Ja se ne bih složio sa tim....mislim,složio bih se da berberina ne brije niko,ali ne zato što je berberin žena. Malo bolje pročitaj zadatak. Evo ti jedno pitanje koje ce ti mozda pomoci. Da li se žena brije? :)

0 za
0 protiv
limes5
2010-09-24
???

Miodrag Mihajlović napisa...

 

tiii napisa...

NeparniValjak napisa...

Au,svaka vama cast ,taman skupim snage da vas citam - a vi napredujete dalje bez namere da stanete,gde ste bili celo leto dok sam dangubio :) ...

U jednom selu,berberin brije samo one koji se ne briju sami...Ko brije berberina?

...niko, jer je berberin zena.

...Ja se ne bih složio sa tim....mislim,složio bih se da berberina ne brije niko,ali ne zato što je berberin žena. Malo bolje pročitaj zadatak. Evo ti jedno pitanje koje ce ti mozda pomoci. Da li se žena brije? :)

...ok, briju se i zene ali pretpostavljam da se ovdje misli na bradu.

0 za
0 protiv
tiii
2010-09-25
???

 

tiii napisa...

Miodrag Mihajlović napisa...

 

tiii napisa...

NeparniValjak napisa...

Au,svaka vama cast ,taman skupim snage da vas citam - a vi napredujete dalje bez namere da stanete,gde ste bili celo leto dok sam dangubio :) ...

U jednom selu,berberin brije samo one koji se ne briju sami...Ko brije berberina?

...niko, jer je berberin zena.

...Ja se ne bih složio sa tim....mislim,složio bih se da berberina ne brije niko,ali ne zato što je berberin žena. Malo bolje pročitaj zadatak. Evo ti jedno pitanje koje ce ti mozda pomoci. Da li se žena brije? :)

...ok, briju se i zene ali pretpostavljam da se ovdje misli na bradu.

...Hehe,nisam mislio na to! :) U zadatku kaže da berberin brije samo one koji se ne briju sami.Ako je berberin žena,pretpostavimo da ona ne brije bradu :),samim tim se ne brije sama...Zar ne? 

0 za
0 protiv
limes5
2010-09-25
???

I tako u krug...

OK, ne moze.
 

0 za
0 protiv
tiii
2010-09-26
???

Vladimir Obradović napisa...

Moje rešenje je: 2km i 414m.

...i jos 21cm

0 za
0 protiv
Milan Jovanović
2010-09-26
???

4.Na gomili stoji 100 zetona..Dva igraca A i B igraju sledecu igru:Najpre igrac A uzme izvesan broj zetona sa gomile,a zatim u sledecem potezu  igrac B moze da  uzme broj zetona takav da je taj broj delilac broja zetona koji je uzeo igrac B.U sledecem potezu igrac A uzima broj zetona koji je delilac broja zetona koji je uzeo igrac B itd.Pobednik je igrac koji uzme poslednji zeton.Koji igrac u ovoj igri ima pobednicku strategiju i kako treba da igra?

 

5.Data je kvadratna tablica 5x5.Dva igraca A i B igraju sledecu igru:Najpre igrac A u tablicu upise neki broj iz skupa 1,2,3...,24,25,a zatim to isto ucini igrac B.Igra se zavrsava kada svi brojevi iz pomenutog skupa budu upisani u tablicu.Pobednik je igrac A,ako se u nekoj vrsti ili koloni tablici nakon upisivanja svih brojeva pojavi zbir 70,u suprotnom pobednik je igrac B.Koji igrac u ovoj igri ima pobednicku strategiju i kako treba da igra?

Posto se niko ne javlja ili su problemi zaboravljeni,evo resenja oba,mozda ce nekog zanimati.

 

4.Igrac A ima pobednicku strategiju.Uzima u 1.potezu 4 zetona,a zatim igra simetricno.

5..Ne umanjujuci opstost mozemo pretpostaviti da je ta tablica isecak sahovske table cija se ugaona polja A1,E1,E5,A5(ovo uvodim samo jer nisam u mogucnosti da slikama prikazem resenje,pa sam uveo u ovom slucaju najpogodniju notaciju).Igrac A ima pobednicku strategiju.On najpre izvrsi particiju skupa {1,2,...25}na 12 dvoclanih skupova {1,22},{2.21}...{11,12} ,{23,25} i jedan jednoclan skup{24}.A 25 polja ove tablice takodje partitivno razbije na sledece skupove {A1},{A2,A3},{A4,A5},{B1,C1},{D1,E1},{D2,D3},{D4,D5},{C2,C3},{C4,C5},{B2,B3},{B4,B5},{A2.A3},{A4,A5}.Igrac A ostvaruje pobednicku strategiju na sledeci nacin:U polje A1 upisuje broj 24,a zatim na svaki potez igraca B upisuje broj koji je u dvoclanom skupu sa brojem koji je upisao igrac B i to na polje koje je u paru sa poljem na koje je broj upisao igrac B.Posto je zbir brojeva u svakom dvoclanom skupu 23(sem u jednom{23,25}),lako se vidi da  ce ili u vrsti ili koloni u kojoj se nalazi broj 24 biti zbir 70.

1 za
0 protiv
mc80
2010-09-28
???

mc80 napisa...

4.Na gomili stoji 100 zetona..Dva igraca A i B igraju sledecu igru:Najpre igrac A uzme izvesan broj zetona sa gomile,a zatim u sledecem potezu  igrac B moze da  uzme broj zetona takav da je taj broj delilac broja zetona koji je uzeo igrac B.U sledecem potezu igrac A uzima broj zetona koji je delilac broja zetona koji je uzeo igrac B itd.Pobednik je igrac koji uzme poslednji zeton.Koji igrac u ovoj igri ima pobednicku strategiju i kako treba da igra?

 

5.Data je kvadratna tablica 5x5.Dva igraca A i B igraju sledecu igru:Najpre igrac A u tablicu upise neki broj iz skupa 1,2,3...,24,25,a zatim to isto ucini igrac B.Igra se zavrsava kada svi brojevi iz pomenutog skupa budu upisani u tablicu.Pobednik je igrac A,ako se u nekoj vrsti ili koloni tablici nakon upisivanja svih brojeva pojavi zbir 70,u suprotnom pobednik je igrac B.Koji igrac u ovoj igri ima pobednicku strategiju i kako treba da igra?

Posto se niko ne javlja ili su problemi zaboravljeni,evo resenja oba,mozda ce nekog zanimati.

 

4.Igrac A ima pobednicku strategiju.Uzima u 1.potezu 4 zetona,a zatim igra simetricno.

5..Ne umanjujuci opstost mozemo pretpostaviti da je ta tablica isecak sahovske table cija se ugaona polja A1,E1,E5,A5(ovo uvodim samo jer nisam u mogucnosti da slikama prikazem resenje,pa sam uveo u ovom slucaju najpogodniju notaciju).Igrac A ima pobednicku strategiju.On najpre izvrsi particiju skupa {1,2,...25}na 12 dvoclanih skupova {1,22},{2.21}...{11,12} ,{23,25} i jedan jednoclan skup{24}.A 25 polja ove tablice takodje partitivno razbije na sledece skupove {A1},{A2,A3},{A4,A5},{B1,C1},{D1,E1},{D2,D3},{D4,D5},{C2,C3},{C4,C5},{B2,B3},{B4,B5},{A2.A3},{A4,A5}.Igrac A ostvaruje pobednicku strategiju na sledeci nacin:U polje A1 upisuje broj 24,a zatim na svaki potez igraca B upisuje broj koji je u dvoclanom skupu sa brojem koji je upisao igrac B i to na polje koje je u paru sa poljem na koje je broj upisao igrac B.Posto je zbir brojeva u svakom dvoclanom skupu 23(sem u jednom{23,25}),lako se vidi da  ce ili u vrsti ili koloni u kojoj se nalazi broj 24 biti zbir 70.

...Svaka ti cast ako si uspeo da resis 5. zadatak.

0 za
0 protiv
raborg
2010-09-28
???

raborg napisa...


...Svaka ti cast ako si uspeo da resis 5. zadatak.

Nisam ga ni radio,jer onaj sto je meni postavio,odmah mi je i pokazao resenje.Mada,zadatak je bas simpatican :)

0 za
0 protiv
mc80
2010-09-29
???

raborg napisa...

   Kolona vojnika dugačka jedan kilometar počinje da maršira ravnomernom brzinom.Posle tačno jednog kilometra zaustavljaju se.U istom trenutku kad je kolona krenula, glasnik koji se nalazi na samom začelju, izdvaja se i počinje da se kreće ka početku, takodje ravnomernom brzinom.Njegova brzina je taman takva da uspe da stigne sam početak kolone , istovremeno se okrene i nastavi nazad ka začelju, gde stigne baš u trenutku kada se i sama kolona zaustavila.Celim putem, brzine kolone kao ni glasnika nisu se menjale.Pitanje glasi: -Koliko je glasnik prepešačio?

    Nadam se da će malo mučenja oko ovog zadatka mnogima doneti zadovoljstvo, kao i da će zadataka biti još.

...1km

0 za
0 protiv
ance_aa@yahoo.com
2010-09-29
???

mc80 napisa...

4.Na gomili stoji 100 zetona..Dva igraca A i B igraju sledecu igru:Najpre igrac A uzme izvesan broj zetona sa gomile,a zatim u sledecem potezu  igrac B moze da  uzme broj zetona takav da je taj broj delilac broja zetona koji je uzeo igrac B.U sledecem potezu igrac A uzima broj zetona koji je delilac broja zetona koji je uzeo igrac B itd.Pobednik je igrac koji uzme poslednji zeton.Koji igrac u ovoj igri ima pobednicku strategiju i kako treba da igra?

 

5.Data je kvadratna tablica 5x5.Dva igraca A i B igraju sledecu igru:Najpre igrac A u tablicu upise neki broj iz skupa 1,2,3...,24,25,a zatim to isto ucini igrac B.Igra se zavrsava kada svi brojevi iz pomenutog skupa budu upisani u tablicu.Pobednik je igrac A,ako se u nekoj vrsti ili koloni tablici nakon upisivanja svih brojeva pojavi zbir 70,u suprotnom pobednik je igrac B.Koji igrac u ovoj igri ima pobednicku strategiju i kako treba da igra?

Posto se niko ne javlja ili su problemi zaboravljeni,evo resenja oba,mozda ce nekog zanimati.

 

4.Igrac A ima pobednicku strategiju.Uzima u 1.potezu 4 zetona,a zatim igra simetricno.

5..Ne umanjujuci opstost mozemo pretpostaviti da je ta tablica isecak sahovske table cija se ugaona polja A1,E1,E5,A5(ovo uvodim samo jer nisam u mogucnosti da slikama prikazem resenje,pa sam uveo u ovom slucaju najpogodniju notaciju).Igrac A ima pobednicku strategiju.On najpre izvrsi particiju skupa {1,2,...25}na 12 dvoclanih skupova {1,22},{2.21}...{11,12} ,{23,25} i jedan jednoclan skup{24}.A 25 polja ove tablice takodje partitivno razbije na sledece skupove {A1},{A2,A3},{A4,A5},{B1,C1},{D1,E1},{D2,D3},{D4,D5},{C2,C3},{C4,C5},{B2,B3},{B4,B5},{A2.A3},{A4,A5}.Igrac A ostvaruje pobednicku strategiju na sledeci nacin:U polje A1 upisuje broj 24,a zatim na svaki potez igraca B upisuje broj koji je u dvoclanom skupu sa brojem koji je upisao igrac B i to na polje koje je u paru sa poljem na koje je broj upisao igrac B.Posto je zbir brojeva u svakom dvoclanom skupu 23(sem u jednom{23,25}),lako se vidi da  ce ili u vrsti ili koloni u kojoj se nalazi broj 24 biti zbir 70.

...Napokon. Hvala za rjesenje 5-og zadatka, vec danima razbijam glavu sa njim, a spremam ispit.

1 za
0 protiv
tiii
2010-09-29
???

tiii napisa...


...Napokon. Hvala za rjesenje 5-og zadatka, vec danima razbijam glavu sa njim, a spremam ispit.

Nema na cemu.smiley

1 za
0 protiv
mc80
2010-09-30
???

Na ivici provalije stoji pijanac koji sa verovatnocom p cini jedan korak unapred(i upada u provaliju),a sa verovatnocom 1-p cini korak unazad(ka sigurnosti).Odrediti verovatnocu da on nece upasti u provaliju(smatra se da pijanac pravi korake jednake duzine).
 

1 za
2 protiv
mc80
2010-09-30
???

mc80 napisa...

Na ivici provalije stoji pijanac koji sa verovatnocom p cini jedan korak unapred(i upada u provaliju),a sa verovatnocom 1-p cini korak unazad(ka sigurnosti).Odrediti verovatnocu da on nece upasti u provaliju(smatra se da pijanac pravi korake jednake duzine).
 

...Da nije mozda  (1-p)2 / (1-p+p2)

0 za
0 protiv
raborg
2010-09-30
???

NeparniValjak napisa...

Au,svaka vama cast ,taman skupim snage da vas citam - a vi napredujete dalje bez namere da stanete,gde ste bili celo leto dok sam dangubio :) ...

U jednom selu,berberin brije samo one koji se ne briju sami...Ko brije berberina?

...jedan od primera Raselovog paradoksa...

0 za
0 protiv
Miško Brada
2010-09-30
???

Od cifara 0,1,2,3,4,5,6,7,8 i 9 (upotrebiti svaku cifru samo jednom) sastaviti 5 dvocifrenih brojeva tako da njihov proizvod bude najveći moguć. I obrazložiti odgovor,čisto da ne nagađamo. wink

0 za
0 protiv
limes5
2010-10-01
???

limes5 napisa...

Od cifara 0,1,2,3,4,5,6,7,8 i 9 (upotrebiti svaku cifru samo jednom) sastaviti 5 dvocifrenih brojeva tako da njihov proizvod bude najveći moguć. I obrazložiti odgovor,čisto da ne nagađamo. wink

...Zadatak je prost i ne znam sta bi se obrazlagalo.Brojevi su: 90, 81, 72, 63 i 54.

0 za
0 protiv
raborg
2010-10-01
???

 

raborg napisa...

limes5 napisa...

Od cifara 0,1,2,3,4,5,6,7,8 i 9 (upotrebiti svaku cifru samo jednom) sastaviti 5 dvocifrenih brojeva tako da njihov proizvod bude najveći moguć. I obrazložiti odgovor,čisto da ne nagađamo. wink

...Zadatak je prost i ne znam sta bi se obrazlagalo.Brojevi su: 90, 81, 72, 63 i 54.

...Mislio sam da se da obrazloženje,zašto baš taj odgovor...Zašto nisu brojevi npr. 94,83,72,61,50...

A ako je ovaj prost sad ću ti obezbediti jedan koji nije prost,ako želiš da lupaš glavu? :)

0 za
0 protiv
limes5
2010-10-01
???

Samo napred

0 za
0 protiv
raborg
2010-10-01
???

@ raborg

U drzavi ima 100 aerodroma.Sa svakog aerodroma poleće jedan avion i slece na najblizi aerodrom.Rastojanje izmedju svaka dva aerodroma je razlicito.Dokazati da ni na jedan aerodrom ne moze da sleti vise od 5 aviona. 

0 za
0 protiv
limes5
2010-10-01
???

limes5 napisa...

@ raborg

U drzavi ima 100 aerodroma.Sa svakog aerodroma poleće jedan avion i slece na najblizi aerodrom.Rastojanje izmedju svaka dva aerodroma je razlicito.Dokazati da ni na jedan aerodrom ne moze da sleti vise od 5 aviona. 

...Zamislimo, umesto aerodroma, 100 tacaka rasporedjenih po datoj povrsini i uocimo jednu od njih A i njoj najblizu tacku B.Treca tacka C , da bi bila bliza tacki A mora da bude sa njene strane simetrale na duz AB.Istovremeno , da bi rastojanje AC bilo vece nego AB, ugao izmedju duzi AB i AC mora da bude veci od 600, odnosno za sledecu tacku je ostalo nesto manje od 3000  kruznog isecka.Nastavljajuci dalje da dodajemo tacke po istom principu, stizemo do sedme tacke, odnosno seste oko tacke A.Za nju je ostalo manje od 600 kruznog isecka a drugi uslov , postavljen na pocetku, kaze da mora da bude vise od 600 .Znaci nemoguce je postaviti sest tacaka oko tacke A, odnosno na jedan aerodrom moze sleteti najvise pet aviona.

  Valjda ce biti dovoljno.Problem oko zadataka s dokazima je u objasnjavanju istih, samo recima.

1 za
0 protiv
raborg
2010-10-01
???

 

raborg napisa...

limes5 napisa...

@ raborg

U drzavi ima 100 aerodroma.Sa svakog aerodroma poleće jedan avion i slece na najblizi aerodrom.Rastojanje izmedju svaka dva aerodroma je razlicito.Dokazati da ni na jedan aerodrom ne moze da sleti vise od 5 aviona. 

...Zamislimo, umesto aerodroma, 100 tacaka rasporedjenih po datoj povrsini i uocimo jednu od njih A i njoj najblizu tacku B.Treca tacka C , da bi bila bliza tacki A mora da bude sa njene strane simetrale na duz AB.Istovremeno , da bi rastojanje AC bilo vece nego AB, ugao izmedju duzi AB i AC mora da bude veci od 600, odnosno za sledecu tacku je ostalo nesto manje od 3000  kruznog isecka.Nastavljajuci dalje da dodajemo tacke po istom principu, stizemo do sedme tacke, odnosno seste oko tacke A.Za nju je ostalo manje od 600 kruznog isecka a drugi uslov , postavljen na pocetku, kaze da mora da bude vise od 600 .Znaci nemoguce je postaviti sest tacaka oko tacke A, odnosno na jedan aerodrom moze sleteti najvise pet aviona.

  Valjda ce biti dovoljno.Problem oko zadataka s dokazima je u objasnjavanju istih, samo recima.

Šta drugo da kažem nego: Svaka ti čast majstore! yes

0 za
0 protiv
limes5
2010-10-01
???

@limes5: Hvala.Iako vise volim zadatke koji nisu dokazi, ovom nisam mogao da odolim.

1 za
0 protiv
raborg
2010-10-01
???

raborg napisa...

mc80 napisa...

Na ivici provalije stoji pijanac koji sa verovatnocom p cini jedan korak unapred(i upada u provaliju),a sa verovatnocom 1-p cini korak unazad(ka sigurnosti).Odrediti verovatnocu da on nece upasti u provaliju(smatra se da pijanac pravi korake jednake duzine).
 

...Da nije mozda  (1-p)2 / (1-p+p2)

Hm,ne dobije se to resenje!?Ajde napisi celo resenje,pa da vidimo.

0 za
0 protiv
mc80
2010-10-02
???

mc80 napisa...

raborg napisa...

mc80 napisa...

Na ivici provalije stoji pijanac koji sa verovatnocom p cini jedan korak unapred(i upada u provaliju),a sa verovatnocom 1-p cini korak unazad(ka sigurnosti).Odrediti verovatnocu da on nece upasti u provaliju(smatra se da pijanac pravi korake jednake duzine).
 

...Da nije mozda  (1-p)2 / (1-p+p2)

Hm,ne dobije se to resenje!?Ajde napisi celo resenje,pa da vidimo.

...  Evo, mc80:

Posto p i 1-p daju zbir 1 , iskljuceno je da pijanac pravi korake u stranu ili stoji u mestu.Kako korak napred dovodi do ishoda dogadjaja "pad" a koraci unazad mogu da idu u beskonacno , lakse je izracunati verovatnocu pada, pa oduzimanjem iste od 1 dobija se verovatnoca "da nece upasti".Kao sto je i receno, verovatnoca za korak napred je p, a ako se pomeri korak nazad s verovatnocom 1-p potrebno je da nacini dva koraka napred da bi pao , sto cini ukupno (1-p)*p2 ,Verovatnoca da upadne kada je dva koraka odmaknut je (1-p)2*p3 i tako progresivno za svaki korak dalje.E sad zbir svih ovih verovatnoca, koje cine geometrijski niz gde je a1=p, q=(1-p)*p, daju verovatnocu da ce pasti u provaliju.Taj zbir je p/(p2 -p+1).Jos da od 1 oduzmemo poslednji izraz i to je to.

2 za
0 protiv
raborg
2010-10-02
???

raborg napisa...

mc80 napisa...

raborg napisa...

mc80 napisa...

Na ivici provalije stoji pijanac koji sa verovatnocom p cini jedan korak unapred(i upada u provaliju),a sa verovatnocom 1-p cini korak unazad(ka sigurnosti).Odrediti verovatnocu da on nece upasti u provaliju(smatra se da pijanac pravi korake jednake duzine).
 

...Da nije mozda  (1-p)2 / (1-p+p2)

Hm,ne dobije se to resenje!?Ajde napisi celo resenje,pa da vidimo.

...  Evo, mc80:

Posto p i 1-p daju zbir 1 , iskljuceno je da pijanac pravi korake u stranu ili stoji u mestu.Kako korak napred dovodi do ishoda dogadjaja "pad" a koraci unazad mogu da idu u beskonacno , lakse je izracunati verovatnocu pada, pa oduzimanjem iste od 1 dobija se verovatnoca "da nece upasti".Kao sto je i receno, verovatnoca za korak napred je p, a ako se pomeri korak nazad s verovatnocom 1-p potrebno je da nacini dva koraka napred da bi pao , sto cini ukupno (1-p)*p2 ,Verovatnoca da upadne kada je dva koraka odmaknut je (1-p)2*p3 i tako progresivno za svaki korak dalje.E sad zbir svih ovih verovatnoca, koje cine geometrijski niz gde je a1=p, q=(1-p)*p, daju verovatnocu da ce pasti u provaliju.Taj zbir je p/(p2 -p+1).Jos da od 1 oduzmemo poslednji izraz i to je to.

E ovako.U ovom tvom resenju opisane su samo neke putanje pijanca,tj.putanje kada on prvo ide n koraka unazad,a zatim n+1 unapred.Sta je npr. sa putanjom,kada on ide recimo 5 koraka unazad,pa 2 unapred,pa opet 3 unazad?(tj.vise puta naizmenicno napred nazad)

0 za
0 protiv
mc80
2010-10-03
???

U pravu si mc80, moracu jos malo da mozgam.

0 za
0 protiv
raborg
2010-10-03
???

raborg napisa...

U pravu si mc80, moracu jos malo da mozgam.

Mozda sam propustio da kazem da je ovaj problem malo potezi.Eto razmisli jos malo,nadam se da ces ga resiti smiley

0 za
0 protiv
mc80
2010-10-03
???

@mc80

Već nekoliko puta si dokazao da si vrhunski matematičar, svaka čast, što rekao naš narod, kapa dole, problemi koje postavljaš uglavnom su iz  kombinatorike i vjerovatnoće i statistike,ako sam nešto mrzio iz matematike, to je upravo vjerovatnoća i statistika, ali sam uvjek i uvjek ću poštovati ljude koji dobro poznaju tu oblast, po mom mišljenu to su vrhunski matematičari, e sad zašto sve ovo pišem, ova tema nosi naslov "zanimljivi matematički problemi", kombinatorika i bože pomozi, al ViS, je nešto mnogo ozbiljnije, te možda ne bi bilo loše, kad već postavljeaš probleme ovog tipa, da to bude nova tema, recimo "viša matematika" il nešto u tom fazonu, mislim da da kviz slagalica i ne bi bio toliko popularan, kad bi se tamo rješavali neki superteški zadaci, popularan je upravo zbog toga što je težina problema prilagođena većini, svako može da pronađeno nešto u čemu je dobar.

3 za
0 protiv
nobody
2010-10-03
???

nobody napisa...

@mc80

Već nekoliko puta si dokazao da si vrhunski matematičar, svaka čast, što rekao naš narod, kapa dole, problemi koje postavljaš uglavnom su iz  kombinatorike i vjerovatnoće i statistike,ako sam nešto mrzio iz matematike, to je upravo vjerovatnoća i statistika, ali sam uvjek i uvjek ću poštovati ljude koji dobro poznaju tu oblast, po mom mišljenu to su vrhunski matematičari, e sad zašto sve ovo pišem, ova tema nosi naslov "zanimljivi matematički problemi", kombinatorika i bože pomozi, al ViS, je nešto mnogo ozbiljnije, te možda ne bi bilo loše, kad već postavljeaš probleme ovog tipa, da to bude nova tema, recimo "viša matematika" il nešto u tom fazonu, mislim da da kviz slagalica i ne bi bio toliko popularan, kad bi se tamo rješavali neki superteški zadaci, popularan je upravo zbog toga što je težina problema prilagođena većini, svako može da pronađeno nešto u čemu je dobar.

OK.Slazem se da ima tezih zadataka koji zahtevaju neko minimalno poznavanje verovatnoce i kombinatorike,ali opet i oni spadaju u temu Zanimljivi matematicki problemi.Ubuduce cu kada postavim problem naznaciti da treba neko odredjeno znanje iz matematike,mada vecinu problema mogu svi da resavaju.

0 za
0 protiv
mc80
2010-10-04
???

Evo 2 laksa,dosta laksa problema od ovog prethodnog:

1.Da li se krug moze prekriti sa dva manja kruga?

2.Iz sahovske table izrezana su polja A1 i H8.Da li se ova novodobijena tabla moze prekriti sa 31 dominom velicine 1x2(misli se da da domina pokriva tacno 2 susedna sahovska polja)?

0 za
0 protiv
mc80
2010-10-04
???

2. A1 i H8 su crna polja,tako da ostane 30 crnih i 32 bijela polja.Domina pokriva dva susjedna polja,jedno crno i jedno bijelo  tako da je nemoguće sa 31 dominom pokriti takvu šahovsku tablu.

3 za
0 protiv
limes5
2010-10-04
???

limes5 napisa...

2. A1 i H8 su crna polja,tako da ostane 30 crnih i 32 bijela polja.Domina pokriva dva susjedna polja,jedno crno i jedno bijelo  tako da je nemoguće sa 31 dominom pokriti takvu šahovsku tablu.

Bravo!To je resenje.

0 za
0 protiv
mc80
2010-10-04
???

mc80 napisa...

Evo 2 laksa,dosta laksa problema od ovog prethodnog:

1.Da li se krug moze prekriti sa dva manja kruga?

 

...Ne može se prekriti sa dva manja!

2 za
0 protiv
Слађан
2010-10-04
???

mc80 napisa...

1.Da li se krug moze prekriti sa dva manja kruga?

... Решење је слично као за кругове полупречника 2, 3, 4 и 5 од пре неколико дана, зар не?

А, пијанцем разбијам главу с времена на време и сконтао сам да уопште није наиван проблем...

1 za
0 protiv
Цане Вукић
2010-10-04
???

Na nekom srednjevjekovnom sudu sudilo se ovako:

Sudija bi optuženom pružio dvije cedulje,od kojih je na jednoj stajalo "ŽIVOT" a na drugoj "SMRT".Optušeni je zavezanih očiju morao odabrati jednu od njih.Ako je na njoj pisalo "ŽIVOT",bio je oslobođen,a u protivnom slučaju bio je ubijen.

Jedan optuženi saznao je da mu sudija nije "naklonjen",i da je na obe cedulje koje će mu ponuditi napisao "SMRT".On ipak nije stradao,već je poznavanje namjere sudije da ga sa sigurnošću  uništi iskoristio tako da se sa sigurnošću spasio.Kako?

2 za
0 protiv
limes5
2010-10-04
???

limes5 napisa...

Na nekom srednjevjekovnom sudu sudilo se ovako:

Sudija bi optuženom pružio dvije cedulje,od kojih je na jednoj stajalo "ŽIVOT" a na drugoj "SMRT".Optušeni je zavezanih očiju morao odabrati jednu od njih.Ako je na njoj pisalo "ŽIVOT",bio je oslobođen,a u protivnom slučaju bio je ubijen.

Jedan optuženi saznao je da mu sudija nije "naklonjen",i da je na obe cedulje koje će mu ponuditi napisao "SMRT".On ipak nije stradao,već je poznavanje namjere sudije da ga sa sigurnošću  uništi iskoristio tako da se sa sigurnošću spasio.Kako?

...Malo je nezahvalno odgovoriti nakon 20ak minuta i ostalim takmicarima uskratiti mogucnost da odgovore,mozda je ideja otvoriti jednu temu ZADACI a drugu ODGOVORI,a dobra je pitalica...

0 za
0 protiv
NeparniValjak
2010-10-04
???

 

NeparniValjak napisa...

limes5 napisa...

Na nekom srednjevjekovnom sudu sudilo se ovako:

Sudija bi optuženom pružio dvije cedulje,od kojih je na jednoj stajalo "ŽIVOT" a na drugoj "SMRT".Optušeni je zavezanih očiju morao odabrati jednu od njih.Ako je na njoj pisalo "ŽIVOT",bio je oslobođen,a u protivnom slučaju bio je ubijen.

Jedan optuženi saznao je da mu sudija nije "naklonjen",i da je na obe cedulje koje će mu ponuditi napisao "SMRT".On ipak nije stradao,već je poznavanje namjere sudije da ga sa sigurnošću  uništi iskoristio tako da se sa sigurnošću spasio.Kako?

...Malo je nezahvalno odgovoriti nakon 20ak minuta i ostalim takmicarima uskratiti mogucnost da odgovore,mozda je ideja otvoriti jednu temu ZADACI a drugu ODGOVORI,a dobra je pitalica...

...Jako nesebično s' tvoje strane!  Sviđa mi se ta ideja. Hajde onda da sačekamo još malo da vidimo hoće li još neko pokušati da riješi,a ako takvih ne bude onda ostaje da im ti objasniš kako je ovaj "sudijin ljubimac" izvukao živu glavu... smiley

0 za
0 protiv
limes5
2010-10-04
???

Петар Марковић napisa...

mc80 napisa...

1.Da li se krug moze prekriti sa dva manja kruga?

... Решење је слично као за кругове полупречника 2, 3, 4 и 5 од пре неколико дана, зар не?

А, пијанцем разбијам главу с времена на време и сконтао сам да уопште није наиван проблем...

Sto se kruga tice ako postavimo najpre jedan od dva kruga on nikako ne moze prekriti dve dijametralno suprotne tacke.Te tacke ne moze prekriti ni drugi manji krug.

A za pijanca,pa nije lak zadatak,mada ne i nesto preterano tezak.

1 za
0 protiv
mc80
2010-10-04
???

limes5 napisa...

Na nekom srednjevjekovnom sudu sudilo se ovako:

Sudija bi optuženom pružio dvije cedulje,od kojih je na jednoj stajalo "ŽIVOT" a na drugoj "SMRT".Optušeni je zavezanih očiju morao odabrati jednu od njih.Ako je na njoj pisalo "ŽIVOT",bio je oslobođen,a u protivnom slučaju bio je ubijen.

Jedan optuženi saznao je da mu sudija nije "naklonjen",i da je na obe cedulje koje će mu ponuditi napisao "SMRT".On ipak nije stradao,već je poznavanje namjere sudije da ga sa sigurnošću  uništi iskoristio tako da se sa sigurnošću spasio.Kako?

Da probam.Mozda je izvukao jednu cedulju,pokazao je prisutnima u sudnici i rekao:Ja biram cedulju koja je u ruci sudije.

0 za
1 protiv
mc80
2010-10-04
???

mc80 napisa...

limes5 napisa...

Na nekom srednjevjekovnom sudu sudilo se ovako:

Sudija bi optuženom pružio dvije cedulje,od kojih je na jednoj stajalo "ŽIVOT" a na drugoj "SMRT".Optušeni je zavezanih očiju morao odabrati jednu od njih.Ako je na njoj pisalo "ŽIVOT",bio je oslobođen,a u protivnom slučaju bio je ubijen.

Jedan optuženi saznao je da mu sudija nije "naklonjen",i da je na obe cedulje koje će mu ponuditi napisao "SMRT".On ipak nije stradao,već je poznavanje namjere sudije da ga sa sigurnošću  uništi iskoristio tako da se sa sigurnošću spasio.Kako?

Da probam.Mozda je izvukao jednu cedulju,pokazao je prisutnima u sudnici i rekao:Ja biram cedulju koja je u ruci sudije.

...Dobra ideja,može se i tako spasiti,ali malo mi nelogično zvuči da je on već uzeo jednu cedulju u ruku i pokazao svima a onda rekao da bira onu koja mu nije u ruci...Ako je već nju uzeo,nju je i izabrao...Samo malo bolje da se formuliše,razmišljanje je dobro.wink

0 za
0 protiv
limes5
2010-10-04
???

limes5 napisa...

Na nekom srednjevjekovnom sudu sudilo se ovako:

Sudija bi optuženom pružio dvije cedulje,od kojih je na jednoj stajalo "ŽIVOT" a na drugoj "SMRT".Optušeni je zavezanih očiju morao odabrati jednu od njih.Ako je na njoj pisalo "ŽIVOT",bio je oslobođen,a u protivnom slučaju bio je ubijen.

Jedan optuženi saznao je da mu sudija nije "naklonjen",i da je na obe cedulje koje će mu ponuditi napisao "SMRT".On ipak nije stradao,već je poznavanje namjere sudije da ga sa sigurnošću  uništi iskoristio tako da se sa sigurnošću spasio.Kako?

...Uzeo je jednu i progutao je. Onda je trazio od sudije da pokaze svoju. Na njoj naravno pise smrt sto znaci da je on izabrao zivot.

6 za
0 protiv
Meda
2010-10-04
???

Meda napisa...

limes5 napisa...

Na nekom srednjevjekovnom sudu sudilo se ovako:

Sudija bi optuženom pružio dvije cedulje,od kojih je na jednoj stajalo "ŽIVOT" a na drugoj "SMRT".Optušeni je zavezanih očiju morao odabrati jednu od njih.Ako je na njoj pisalo "ŽIVOT",bio je oslobođen,a u protivnom slučaju bio je ubijen.

Jedan optuženi saznao je da mu sudija nije "naklonjen",i da je na obe cedulje koje će mu ponuditi napisao "SMRT".On ipak nije stradao,već je poznavanje namjere sudije da ga sa sigurnošću  uništi iskoristio tako da se sa sigurnošću spasio.Kako?

...Uzeo je jednu i progutao je. Onda je trazio od sudije da pokaze svoju. Na njoj naravno pise smrt sto znaci da je on izabrao zivot.

...Bravo! To je rješenje.

0 za
0 protiv
limes5
2010-10-04
???

Meda napisa...

limes5 napisa...

Na nekom srednjevjekovnom sudu sudilo se ovako:

Sudija bi optuženom pružio dvije cedulje,od kojih je na jednoj stajalo "ŽIVOT" a na drugoj "SMRT".Optušeni je zavezanih očiju morao odabrati jednu od njih.Ako je na njoj pisalo "ŽIVOT",bio je oslobođen,a u protivnom slučaju bio je ubijen.

Jedan optuženi saznao je da mu sudija nije "naklonjen",i da je na obe cedulje koje će mu ponuditi napisao "SMRT".On ipak nije stradao,već je poznavanje namjere sudije da ga sa sigurnošću  uništi iskoristio tako da se sa sigurnošću spasio.Kako?

...Uzeo je jednu i progutao je. Onda je trazio od sudije da pokaze svoju. Na njoj naravno pise smrt sto znaci da je on izabrao zivot.

...

limes5 napisa...

mc80 napisa...

limes5 napisa...

Na nekom srednjevjekovnom sudu sudilo se ovako:

Sudija bi optuženom pružio dvije cedulje,od kojih je na jednoj stajalo "ŽIVOT" a na drugoj "SMRT".Optušeni je zavezanih očiju morao odabrati jednu od njih.Ako je na njoj pisalo "ŽIVOT",bio je oslobođen,a u protivnom slučaju bio je ubijen.

Jedan optuženi saznao je da mu sudija nije "naklonjen",i da je na obe cedulje koje će mu ponuditi napisao "SMRT".On ipak nije stradao,već je poznavanje namjere sudije da ga sa sigurnošću  uništi iskoristio tako da se sa sigurnošću spasio.Kako?

Da probam.Mozda je izvukao jednu cedulju,pokazao je prisutnima u sudnici i rekao:Ja biram cedulju koja je u ruci sudije.

...Dobra ideja,može se i tako spasiti,ali malo mi nelogično zvuči da je on već uzeo jednu cedulju u ruku i pokazao svima a onda rekao da bira onu koja mu nije u ruci...Ako je već nju uzeo,nju je i izabrao...Samo malo bolje da se formuliše,razmišljanje je dobro.wink

Mozda moje resenje nije idealno,ali i ideja sa gutanjem isto diskutabilna,jer otkud ja da znam da je on imao pravo da guta cedulje!?

0 za
0 protiv
mc80
2010-10-04
???

 

mc80 napisa...

Meda napisa...

limes5 napisa...

Na nekom srednjevjekovnom sudu sudilo se ovako:

Sudija bi optuženom pružio dvije cedulje,od kojih je na jednoj stajalo "ŽIVOT" a na drugoj "SMRT".Optušeni je zavezanih očiju morao odabrati jednu od njih.Ako je na njoj pisalo "ŽIVOT",bio je oslobođen,a u protivnom slučaju bio je ubijen.

Jedan optuženi saznao je da mu sudija nije "naklonjen",i da je na obe cedulje koje će mu ponuditi napisao "SMRT".On ipak nije stradao,već je poznavanje namjere sudije da ga sa sigurnošću  uništi iskoristio tako da se sa sigurnošću spasio.Kako?

...Uzeo je jednu i progutao je. Onda je trazio od sudije da pokaze svoju. Na njoj naravno pise smrt sto znaci da je on izabrao zivot.

...

limes5 napisa...

mc80 napisa...

limes5 napisa...

Na nekom srednjevjekovnom sudu sudilo se ovako:

Sudija bi optuženom pružio dvije cedulje,od kojih je na jednoj stajalo "ŽIVOT" a na drugoj "SMRT".Optušeni je zavezanih očiju morao odabrati jednu od njih.Ako je na njoj pisalo "ŽIVOT",bio je oslobođen,a u protivnom slučaju bio je ubijen.

Jedan optuženi saznao je da mu sudija nije "naklonjen",i da je na obe cedulje koje će mu ponuditi napisao "SMRT".On ipak nije stradao,već je poznavanje namjere sudije da ga sa sigurnošću  uništi iskoristio tako da se sa sigurnošću spasio.Kako?

Da probam.Mozda je izvukao jednu cedulju,pokazao je prisutnima u sudnici i rekao:Ja biram cedulju koja je u ruci sudije.

...Dobra ideja,može se i tako spasiti,ali malo mi nelogično zvuči da je on već uzeo jednu cedulju u ruku i pokazao svima a onda rekao da bira onu koja mu nije u ruci...Ako je već nju uzeo,nju je i izabrao...Samo malo bolje da se formuliše,razmišljanje je dobro.wink

Mozda moje resenje nije idealno,ali i ideja sa gutanjem isto diskutabilna,jer otkud ja da znam da je on imao pravo da guta cedulje!?

... Priznajem da zvuči malo čudno da ju je progutao. smileyPretpostavimo da je mogao da radi sa njom sta je htjeo. Sad razmišljam šta je drugo mogao da uradi...možda da pocjepa...ili da zapali,mada sigurno nije imao čim...tako da gutanje ostaje jedina mogućnost u kojoj će se cedulja uništiti...

0 za
0 protiv
limes5
2010-10-04
???

mc80 napisa...

raborg napisa...

U pravu si mc80, moracu jos malo da mozgam.

Mozda sam propustio da kazem da je ovaj problem malo potezi.Eto razmisli jos malo,nadam se da ces ga resiti smiley

...Au, kako sad da napisem ovo .

1-(suma kada k ide od 0 do n(beskonacno) od pk+1*(1-p)k *(2k nad k)/(k+1)

Ako je ovo uopste dobro , ne znam samo da li moze, odnosno treba, da se i izracuna ova suma.

1 za
0 protiv
raborg
2010-10-04
???

raborg napisa...


...Au, kako sad da napisem ovo .

1-(suma kada k ide od 0 do n(beskonacno) od pk+1*(1-p)k *(2k nad k)/(k+1)

Ako je ovo uopste dobro , ne znam samo da li moze, odnosno treba, da se i izracuna ova suma.

Ajde pojasni prvo kako si dosao do ovog rezultata,pa cemo onda videti sta cemo sa sumom.smiley

0 za
0 protiv
mc80
2010-10-05
???

U Severnom moru,jedan naftni toranj,ucvrscen za morsko dno,uzdize se 40 metara nad mirnom povrsinom mora.Za vreme jake oluje,toranj se zaljuljao i srusio,ali se nije slomio.Sa susedne platforme su opazili da je vrh tornja nestao u moru 84 metra daleko od tacke,gde je toranj uranjao u vodu.Odrediti dubinu mora na mestu gde je toranj bio.

P.S.Za resenje ovog zadatka dovoljno je znanje matematike iz osnovne skole.

5 za
0 protiv
mc80
2010-10-05
???

Svaka tacka u Dekartovom koordinatnom sistemu,sa celobrojnim kordinatama,obojena je sa jednom od dve boje:plavom ili crvenom.Dokazati da postoje dve horizontalne i dve vertikalne prave,takve da su sve cetiri njihove presecne tacke obojene istom bojom.

0 za
0 protiv
mc80
2010-10-05
???

mc80 napisa...

U Severnom moru,jedan naftni toranj,ucvrscen za morsko dno,uzdize se 40 metara nad mirnom povrsinom mora.Za vreme jake oluje,toranj se zaljuljao i srusio,ali se nije slomio.Sa susedne platforme su opazili da je vrh tornja nestao u moru 84 metra daleko od tacke,gde je toranj uranjao u vodu.Odrediti dubinu mora na mestu gde je toranj bio.

P.S.Za resenje ovog zadatka dovoljno je znanje matematike iz osnovne skole.

... 68,2m

1 za
0 protiv
Vladimir Obradović
2010-10-05
???

Da li kocku mozemo podeliti na 101 manju kocku?
 

2 za
0 protiv
mc80
2010-10-05
???

Vladimir Obradović napisa...

mc80 napisa...

U Severnom moru,jedan naftni toranj,ucvrscen za morsko dno,uzdize se 40 metara nad mirnom povrsinom mora.Za vreme jake oluje,toranj se zaljuljao i srusio,ali se nije slomio.Sa susedne platforme su opazili da je vrh tornja nestao u moru 84 metra daleko od tacke,gde je toranj uranjao u vodu.Odrediti dubinu mora na mestu gde je toranj bio.

P.S.Za resenje ovog zadatka dovoljno je znanje matematike iz osnovne skole.

... 68,2m

Bravo!To je resenje.Svaka cast!

0 za
0 protiv
mc80
2010-10-05
???

mc80 napisa...

Svaka tacka u Dekartovom koordinatnom sistemu,sa celobrojnim kordinatama,obojena je sa jednom od dve boje:plavom ili crvenom.Dokazati da postoje dve horizontalne i dve vertikalne prave,takve da su sve cetiri njihove presecne tacke obojene istom bojom.

...Bil mogle bit ove tačke

A(0,0)   B(2,0)    C(0,2)  i  D(2,2)

Recimo horizontalne prolaze kroz tačke A B i C D  a vertikalne kroz tačke A C i B D

0 za
0 protiv
MaLin
2010-10-05
???

Vaskrsija napisa...

mc80 napisa...

Svaka tacka u Dekartovom koordinatnom sistemu,sa celobrojnim kordinatama,obojena je sa jednom od dve boje:plavom ili crvenom.Dokazati da postoje dve horizontalne i dve vertikalne prave,takve da su sve cetiri njihove presecne tacke obojene istom bojom.

...Bil mogle bit ove tačke

A(0,0)   B(2,0)    C(0,2)  i  D(2,2)

Recimo horizontalne prolaze kroz tačke A B i C D  a vertikalne kroz tačke A C i B D

Ne,ne procitaj ponovo zadatak.treba dokazati da uvek postoje takve 4 prave....

0 za
0 protiv
mc80
2010-10-05
???

mc80 napisa...

raborg napisa...


...Au, kako sad da napisem ovo .

1-(suma kada k ide od 0 do n(beskonacno) od pk+1*(1-p)k *(2k nad k)/(k+1)

Ako je ovo uopste dobro , ne znam samo da li moze, odnosno treba, da se i izracuna ova suma.

Ajde pojasni prvo kako si dosao do ovog rezultata,pa cemo onda videti sta cemo sa sumom.smiley

...  Prvo da kazem da je zadatak odlican.Kljucna stvar je bila kada sam shvatio da treba posmatrati, ne kako ce pijanac da dospe u neku poziciju pa da odatle stigne do provalije, vec iskljucivo broj njegovih koraka.Sve njegove moguce putanje mozemo podeliti u klase , upravo prema broju koraka nacinjenih do pada u provaliju.Primecujemo da sve klase sadrze neparne brojeve koraka i da sve(osim prve , koja cini samo jedan korak napred) pocinju jednim korakom u nazad (nazvacemo ga N), a zavrsavaju s dva koraka unapred(PP).Izmedju ovih pocetnih i krajnjih koraka, postoji jednak broj N i P koraka.Poenta svega je, na koliko nacina se P moze rasporediti na 2P mesta, uzimajuci u obzir da kad se gleda s leva na desno ni u jednom trenutku zbir svih P ne sme da bude veci od zbira svih N, za vise od jedan, jer ce se pad u provaliju desiti pre vremena, odnosno to nece biti ta klasa.Npr. klasu 5 cine dve putanje, NPNPP i NNPPP,a ne moze NPPNP jer bi se pad desio posle treceg koraka a to bila klasa 3.Ispisujuci sve ispravne kombinacije za prvih par klasa, zakljucio sam da se njihov broj broj menja po obrascu (2k nad k)/(k+1), npr. za gore pomenutu klasu 5 (tad je k=2) dobija se 2, sto i jeste ukupan broj kombinacija za tu klasu.Jos je samo ostalo da se ovaj izraz udruzi s verovatnocom svake klase pk+1(broj P koraka)*(1-p)k (broj N koraka) i dobije se trazena suma.Naravno, ona se na kraju oduzme od 1.

3 za
0 protiv
raborg
2010-10-05
???

mc80 napisa...

Vaskrsija napisa...

mc80 napisa...

Svaka tacka u Dekartovom koordinatnom sistemu,sa celobrojnim kordinatama,obojena je sa jednom od dve boje:plavom ili crvenom.Dokazati da postoje dve horizontalne i dve vertikalne prave,takve da su sve cetiri njihove presecne tacke obojene istom bojom.

...Bil mogle bit ove tačke

A(0,0)   B(2,0)    C(0,2)  i  D(2,2)

Recimo horizontalne prolaze kroz tačke A B i C D  a vertikalne kroz tačke A C i B D

Ne,ne procitaj ponovo zadatak.treba dokazati da uvek postoje takve 4 prave....

...Dobro, ako ti tako kažeš, a de mi barem reci jesam li makar zaradio dvicu

0 za
0 protiv
MaLin
2010-10-05
???

raborg napisa...

mc80 napisa...

raborg napisa...


...Au, kako sad da napisem ovo .

1-(suma kada k ide od 0 do n(beskonacno) od pk+1*(1-p)k *(2k nad k)/(k+1)

Ako je ovo uopste dobro , ne znam samo da li moze, odnosno treba, da se i izracuna ova suma.

Ajde pojasni prvo kako si dosao do ovog rezultata,pa cemo onda videti sta cemo sa sumom.smiley

...  Prvo da kazem da je zadatak odlican.Kljucna stvar je bila kada sam shvatio da treba posmatrati, ne kako ce pijanac da dospe u neku poziciju pa da odatle stigne do provalije, vec iskljucivo broj njegovih koraka.Sve njegove moguce putanje mozemo podeliti u klase , upravo prema broju koraka nacinjenih do pada u provaliju.Primecujemo da sve klase sadrze neparne brojeve koraka i da sve(osim prve , koja cini samo jedan korak napred) pocinju jednim korakom u nazad (nazvacemo ga N), a zavrsavaju s dva koraka unapred(PP).Izmedju ovih pocetnih i krajnjih koraka, postoji jednak broj N i P koraka.Poenta svega je, na koliko nacina se P moze rasporediti na 2P mesta, uzimajuci u obzir da kad se gleda s leva na desno ni u jednom trenutku zbir svih P ne sme da bude veci od zbira svih N, za vise od jedan, jer ce se pad u provaliju desiti pre vremena, odnosno to nece biti ta klasa.Npr. klasu 5 cine dve putanje, NPNPP i NNPPP,a ne moze NPPNP jer bi se pad desio posle treceg koraka a to bila klasa 3.Ispisujuci sve ispravne kombinacije za prvih par klasa, zakljucio sam da se njihov broj broj menja po obrascu (2k nad k)/(k+1), npr. za gore pomenutu klasu 5 (tad je k=2) dobija se 2, sto i jeste ukupan broj kombinacija za tu klasu.Jos je samo ostalo da se ovaj izraz udruzi s verovatnocom svake klase pk+1(broj P koraka)*(1-p)k (broj N koraka) i dobije se trazena suma.Naravno, ona se na kraju oduzme od 1.

Ako ja nisam nesto prevideo(a mislim da nisam),ovo je tacno resenje!Brojevi oblika (2k nad k)/(k+1) su u matematici poznati kao Katalanovi brojevi.Oni se dobijaju npr.resenjem problema koji je slican ovom,a problem glasi:Pred blagajnom bioskopa u redu ceka 2k osoba da kupi kartu koja kosta 100 dinara.k njih ima novcanicu od 100,a k od 200 dinara.Blagajnik pred pocetak prodaje karata nema novca u kasi tj.kasa je prazna.Na koliko nacina se oni mogu poredjati u niz tako da blgajna radi bez zastoja,tj.da blagajnik u svakom trenutku moze kupcu karte da vrati kusur?

Dakle tvoje resenje je tacno.Sad treba "samo" sumirati ovaj red.Trenutno nemam ideju kako bi se to moglo uraditi i da li uopste moze!?A sad cu ja izloziti drugo resenje(za sve koji hoce da jos razmisljaju o ovom problemu,neka ne citaju ovo dalje sto cu da napisem).

Kretanje pijanca mozemo predstaviti na brojevnoj pravoj.Neka se u pocetku on nalazi u tacki 0 i neka je provalija u tacki -1,a pozitivni celi brojevi mesta u kjima je on "siguran".Neka je p0 verovatnoca stizanja iz tacke 0 u tacku -1.Iz tacke 0 on moze krenuti sa verovatnocom p u tacku -1 ili sa verovatnocom 1-p u tacku +1.Neka p1 oznacava verovatnocu stizanja iz tacke +1 u tacku -1.da bi se stiglo u tacku -1 mora se najpre doci u tacku 0.Verovatnoca stizanja iz +1 u 0 je isto p0 tj.ista kao iz 0 u -1 ,pa je p1=p0^2.tako dobijamo p0=p+(1-p)*p0^2.resavanjem ove kvadratne jednacine,dobijamo da je za p>=1/2 p0=1(znaci tada je skoro sigurno da ce upasti u provaliju).Za p<1/2 p0=p/(1-p),pa su trazene verovatnoce

za p>=1/2    P=0

za p<1/2       P=1-p/(1-p)

1 za
0 protiv
mc80
2010-10-05
???

mc80, resenje ti je istovremeno i jednostavno i ubistveno.Prava poslastica i dokaz zasto je matematika to sto jeste.

PS. Nisam ni znao da sam "reinventirao" Katalanove brojeve.

A evo i jedan zadatak od mene:

  Izmedju dva vertikalna zida postavljene su dve letve o obliku slova X.Njihove duzine su 5 i 8 metara.Ako je vertikalno rastojanje izmedju tacke njihovog preseka i ravnog horizontalnog tla 3 metra, koliko je rastojanje izmedju zidova. 

  Ne uspevam nikako da resim ovaj zadatak.Nekako se uvek zakucam u jednacinu cetvrtog stepena.

0 za
0 protiv
raborg
2010-10-05
???

raborg napisa...

mc80, resenje ti je istovremeno i jednostavno i ubistveno.Prava poslastica i dokaz zasto je matematika to sto jeste.

PS. Nisam ni znao da sam "reinventirao" Katalanove brojeve.

Svaka cast kada si ti naslutio opsti oblik (2k nad k)/(k+1),bez znanja sta su Katalanovi brojevi!Da izgleda da si ih ti "otkrio",samo bi ti jos ostalo da dokazes ono sto si "naslutio".Ali dokaz toga,isto sadrzi jednu strahovito ideju,ali nije puno dugacak.Da,zadatak je vise nego lep,mozda potezi.Iz tvog i mog resenja bi se sada mogli izvesti neki zanimljivi identiteti kao npr. da je suma kada k ide od 0 do beskonacno (2k nad k)/(k+1)*(1/4)^k=2 itd.yes

0 za
0 protiv
mc80
2010-10-06
???

raborg napisa...


A evo i jedan zadatak od mene:

  Izmedju dva vertikalna zida postavljene su dve letve o obliku slova X.Njihove duzine su 5 i 8 metara.Ako je vertikalno rastojanje izmedju tacke njihovog preseka i ravnog horizontalnog tla 3 metra, koliko je rastojanje izmedju zidova.

  Ne uspevam nikako da resim ovaj zadatak.Nekako se uvek zakucam u jednacinu cetvrtog stepena.

A cemu sluzi ovaj podatak da je rastojanje tacke  preseka ove dve daske od horizontalnog tla 3m???Ako daske stoje izmedju 2 zida,rastojanje od tacke do ravnog horizontalnog tla pod njima moze btiti i 50 i 100 metara,to nista ne menja stvar???

0 za
0 protiv
mc80
2010-10-07
???

mc80 napisa...

Da li kocku mozemo podeliti na 101 manju kocku?
 

Da li je neko pokusasao da resi ovaj zadatak?On zahteva visok stepen dovitljivosti,nije bas lak,ali ga moze resavati i ucenik 4.razreda osnovne skole.

1 za
0 protiv
mc80
2010-10-07
???

mc80 napisa...

raborg napisa...


A evo i jedan zadatak od mene:

  Izmedju dva vertikalna zida postavljene su dve letve o obliku slova X.Njihove duzine su 5 i 8 metara.Ako je vertikalno rastojanje izmedju tacke njihovog preseka i ravnog horizontalnog tla 3 metra, koliko je rastojanje izmedju zidova.

  Ne uspevam nikako da resim ovaj zadatak.Nekako se uvek zakucam u jednacinu cetvrtog stepena.

A cemu sluzi ovaj podatak da je rastojanje tacke  preseka ove dve daske od horizontalnog tla 3m???Ako daske stoje izmedju 2 zida,rastojanje od tacke do ravnog horizontalnog tla pod njima moze btiti i 50 i 100 metara,to nista ne menja stvar???

...Mozda nisam bio dovoljno jasan.Jedna od letvi stoji na tlu , oslanjajuci se istovemeno na dno jednog zida, dok joj je drugi kraj naslonjen na drugi zid.Za drugu vazi isto, samo su u pitanju suprotni zidovi.Kada bi se zidovi npr.priblizili i rastojanje izmedju tacke preseks letvi i tla bi se povecalo.

0 za
0 protiv
raborg
2010-10-07
???

mc80 napisa...

mc80 napisa...

Da li kocku mozemo podeliti na 101 manju kocku?
 

Da li je neko pokusasao da resi ovaj zadatak?On zahteva visok stepen dovitljivosti,nije bas lak,ali ga moze resavati i ucenik 4.razreda osnovne skole.

...Jesu li manje kocke jednake medjusobno

1 za
0 protiv
raborg
2010-10-07
???

 

mc80 napisa...

mc80 napisa...

Da li kocku mozemo podeliti na 101 manju kocku?
 

Da li je neko pokusasao da resi ovaj zadatak?On zahteva visok stepen dovitljivosti,nije bas lak,ali ga moze resavati i ucenik 4.razreda osnovne skole.

...Ja mislim da može.

Ako datu kocku izdjelimo tako da ona ima 64 dijela,dakle stranicu joj podjelimo na 4 jedinična dijela,iz nje možemo izdvojiti jednu kocku koja u sebi sadrži 27 tih malih "jediničnih kocki" ,tj. kocku čija je stranica duga 3 jedinična dijela.Dobijamo tu 1 veću kocku (koja je sastavljena iz 27 manjih) i preostalo je još 37 onih jediničnih kocki (64-27=37).Od tih 37 izaberemo 9 koje ćemo podjeliti svaku na po 8 još manjih.(Dakle taj jedinični dio dijelimo na pola).Od tih 9 "jediničnih kocki dobijamo 9*8=72 još manje kocke. Pored ovih 72 imamo još 28 koje su ostale kad smo od 37 oduzeli 9 i još onu najveću,što ukupno daje 72+28+1=101.

2 za
0 protiv
limes5
2010-10-07
???

raborg napisa...

mc80 napisa...

mc80 napisa...

Da li kocku mozemo podeliti na 101 manju kocku?
 

Da li je neko pokusasao da resi ovaj zadatak?On zahteva visok stepen dovitljivosti,nije bas lak,ali ga moze resavati i ucenik 4.razreda osnovne skole.

...Jesu li manje kocke jednake medjusobno

...Na glupo pitanje ne treba odgovarati.Omeli su me Barsa i Lejkersi

0 za
0 protiv
raborg
2010-10-07
???

raborg napisa...

raborg napisa...

mc80 napisa...

mc80 napisa...

Da li kocku mozemo podeliti na 101 manju kocku?
 

Da li je neko pokusasao da resi ovaj zadatak?On zahteva visok stepen dovitljivosti,nije bas lak,ali ga moze resavati i ucenik 4.razreda osnovne skole.

...Jesu li manje kocke jednake medjusobno

...Na glupo pitanje ne treba odgovarati.Omeli su me Barsa i Lejkersi

... Na vrijeme si se ispravio! wink Nego,koji je rezultat? 

0 za
0 protiv
limes5
2010-10-07
???

limes5 napisa...

raborg napisa...

raborg napisa...

mc80 napisa...

mc80 napisa...

Da li kocku mozemo podeliti na 101 manju kocku?
 

Da li je neko pokusasao da resi ovaj zadatak?On zahteva visok stepen dovitljivosti,nije bas lak,ali ga moze resavati i ucenik 4.razreda osnovne skole.

...Jesu li manje kocke jednake medjusobno

...Na glupo pitanje ne treba odgovarati.Omeli su me Barsa i Lejkersi

... Na vrijeme si se ispravio! wink Nego,koji je rezultat? 

...Dobila je Barsa tzv. "svetske sampione".Uvek me je uzasno nerviralo kada pobednike svojih profesionanih liga nazivaju svetskim prvacima.

3 za
0 protiv
raborg
2010-10-07
???

raborg napisa...

limes5 napisa...

raborg napisa...

raborg napisa...

mc80 napisa...

mc80 napisa...

Da li kocku mozemo podeliti na 101 manju kocku?
 

Da li je neko pokusasao da resi ovaj zadatak?On zahteva visok stepen dovitljivosti,nije bas lak,ali ga moze resavati i ucenik 4.razreda osnovne skole.

...Jesu li manje kocke jednake medjusobno

...Na glupo pitanje ne treba odgovarati.Omeli su me Barsa i Lejkersi

... Na vrijeme si se ispravio! wink Nego,koji je rezultat? 

...Dobila je Barsa tzv. "svetske sampione".Uvek me je uzasno nerviralo kada pobednike svojih profesionanih liga nazivaju svetskim prvacima.

...Uhh kako mi je drago!! Inače od inostranih klubova navijam za Barsu.Neka su ih malo spustili na zemlju!

0 za
0 protiv
limes5
2010-10-07
???

mc80 napisa...

Da li kocku mozemo podeliti na 101 manju kocku?
 

...Po mom mišljenju, NE MOŽE

Objašnje, te kocke bi morale imati istu zapreminu,  neka visina jedne od tih kocaka iznosi  n  , kad bi njenu visinu povećali za  n, tad bi morali povećati i širinu i dužinu za n, tj. nova kocka visine 2n bi se sastojala od 8 kocaka visine n  (23 =8), kad bi ponovo to uradili, kocka visine 3n bi se sastojala od 27 kocaka visine n (33=27),  a kako ne postoji cijeli broj čiji je kub jednak 101 i kako je sam broj 101 prost broj, moj zaključak je da NE MOŽE.

0 za
3 protiv
konj_AleksandraM
2010-10-07
???

limes5 napisa...

 

mc80 napisa...

mc80 napisa...

Da li kocku mozemo podeliti na 101 manju kocku?
 

Da li je neko pokusasao da resi ovaj zadatak?On zahteva visok stepen dovitljivosti,nije bas lak,ali ga moze resavati i ucenik 4.razreda osnovne skole.

...Ja mislim da može.

Ako datu kocku izdjelimo tako da ona ima 64 dijela,dakle stranicu joj podjelimo na 4 jedinična dijela,iz nje možemo izdvojiti jednu kocku koja u sebi sadrži 27 tih malih "jediničnih kocki" ,tj. kocku čija je stranica duga 3 jedinična dijela.Dobijamo tu 1 veću kocku (koja je sastavljena iz 27 manjih) i preostalo je još 37 onih jediničnih kocki (64-27=37).Od tih 37 izaberemo 9 koje ćemo podjeliti svaku na po 8 još manjih.(Dakle taj jedinični dio dijelimo na pola).Od tih 9 "jediničnih kocki dobijamo 9*8=72 još manje kocke. Pored ovih 72 imamo još 28 koje su ostale kad smo od 37 oduzeli 9 i još onu najveću,što ukupno daje 72+28+1=101.

Bravo!To je tacno resenje!

0 za
1 protiv
mc80
2010-10-08
???

konj_AleksandraM napisa...

mc80 napisa...

Da li kocku mozemo podeliti na 101 manju kocku?
 

...Po mom mišljenju, NE MOŽE

Objašnje, te kocke bi morale imati istu zapreminu,  neka visina jedne od tih kocaka iznosi  n  , kad bi njenu visinu povećali za  n, tad bi morali povećati i širinu i dužinu za n, tj. nova kocka visine 2n bi se sastojala od 8 kocaka visine n  (23 =8), kad bi ponovo to uradili, kocka visine 3n bi se sastojala od 27 kocaka visine n (33=27),  a kako ne postoji cijeli broj čiji je kub jednak 101 i kako je sam broj 101 prost broj, moj zaključak je da NE MOŽE.

Pogledaj resenje koje je dao limes5.

1 za
0 protiv
mc80
2010-10-08
???

mc80 napisa...

konj_AleksandraM napisa...

mc80 napisa...

Da li kocku mozemo podeliti na 101 manju kocku?
 

...Po mom mišljenju, NE MOŽE

Objašnje, te kocke bi morale imati istu zapreminu,  neka visina jedne od tih kocaka iznosi  n  , kad bi njenu visinu povećali za  n, tad bi morali povećati i širinu i dužinu za n, tj. nova kocka visine 2n bi se sastojala od 8 kocaka visine n  (23 =8), kad bi ponovo to uradili, kocka visine 3n bi se sastojala od 27 kocaka visine n (33=27),  a kako ne postoji cijeli broj čiji je kub jednak 101 i kako je sam broj 101 prost broj, moj zaključak je da NE MOŽE.

Pogledaj resenje koje je dao limes5.

...Pogledo i mogu samo da mu čestitam.

3 za
0 protiv
konj_AleksandraM
2010-10-08
???

raborg napisa...


A evo i jedan zadatak od mene:

  Izmedju dva vertikalna zida postavljene su dve letve o obliku slova X.Njihove duzine su 5 i 8 metara.Ako je vertikalno rastojanje izmedju tacke njihovog preseka i ravnog horizontalnog tla 3 metra, koliko je rastojanje izmedju zidova.

  Ne uspevam nikako da resim ovaj zadatak.Nekako se uvek zakucam u jednacinu cetvrtog stepena.

Da.I ja sam dobio jednacinu cetvrtog stepena,koja nema racionalnih resenja.Ako nisam pogresio u racunu(radio sam na brzinu usmeno) cini mi se da imaju 2 realne nule i to jedna izmedju 2 i 3,a druga izmedju 4 i 5.Polinom sa leve strane jednacine je stvarno grozan.Treba probati nekim drugim pristupom doci do resenja,ako to uopste i moze!?

0 za
0 protiv
mc80
2010-10-08
???

mc80 napisa...

raborg napisa...


A evo i jedan zadatak od mene:

  Izmedju dva vertikalna zida postavljene su dve letve o obliku slova X.Njihove duzine su 5 i 8 metara.Ako je vertikalno rastojanje izmedju tacke njihovog preseka i ravnog horizontalnog tla 3 metra, koliko je rastojanje izmedju zidova.

  Ne uspevam nikako da resim ovaj zadatak.Nekako se uvek zakucam u jednacinu cetvrtog stepena.

Da.I ja sam dobio jednacinu cetvrtog stepena,koja nema racionalnih resenja.Ako nisam pogresio u racunu(radio sam na brzinu usmeno) cini mi se da imaju 2 realne nule i to jedna izmedju 2 i 3,a druga izmedju 4 i 5.Polinom sa leve strane jednacine je stvarno grozan.Treba probati nekim drugim pristupom doci do resenja,ako to uopste i moze!?

...Samo da pitam, vjerovatno ste se sjetili da jednačinu četvrtog stepena metodom zamjene pretvorite u kvadratnu jednačinu.

0 za
0 protiv
konj_AleksandraM
2010-10-08
???

limes5 napisa...

 

mc80 napisa...

mc80 napisa...

Da li kocku mozemo podeliti na 101 manju kocku?
 

Da li je neko pokusasao da resi ovaj zadatak?On zahteva visok stepen dovitljivosti,nije bas lak,ali ga moze resavati i ucenik 4.razreda osnovne skole.

...Ja mislim da može.

Ako datu kocku izdjelimo tako da ona ima 64 dijela,dakle stranicu joj podjelimo na 4 jedinična dijela,iz nje možemo izdvojiti jednu kocku koja u sebi sadrži 27 tih malih "jediničnih kocki" ,tj. kocku čija je stranica duga 3 jedinična dijela.Dobijamo tu 1 veću kocku (koja je sastavljena iz 27 manjih) i preostalo je još 37 onih jediničnih kocki (64-27=37).Od tih 37 izaberemo 9 koje ćemo podjeliti svaku na po 8 još manjih.(Dakle taj jedinični dio dijelimo na pola).Od tih 9 "jediničnih kocki dobijamo 9*8=72 još manje kocke. Pored ovih 72 imamo još 28 koje su ostale kad smo od 37 oduzeli 9 i još onu najveću,što ukupno daje 72+28+1=101.

...Svaka čast limes5

Nije lose, al bi mozda najlakse bilo, podijelit je na 64 manje kocke, zatim uzeti jednu i nju podijeliti na 64 manje kocke i imacemo 63+64=127, zatim uzmemo 27 i od nje napravimo jednu i eto ti 127-27+1=101.

1 za
0 protiv
MaLin
2010-10-08
???

mc80 napisa...

raborg napisa...


A evo i jedan zadatak od mene:

  Izmedju dva vertikalna zida postavljene su dve letve o obliku slova X.Njihove duzine su 5 i 8 metara.Ako je vertikalno rastojanje izmedju tacke njihovog preseka i ravnog horizontalnog tla 3 metra, koliko je rastojanje izmedju zidova.

  Ne uspevam nikako da resim ovaj zadatak.Nekako se uvek zakucam u jednacinu cetvrtog stepena.

Da.I ja sam dobio jednacinu cetvrtog stepena,koja nema racionalnih resenja.Ako nisam pogresio u racunu(radio sam na brzinu usmeno) cini mi se da imaju 2 realne nule i to jedna izmedju 2 i 3,a druga izmedju 4 i 5.Polinom sa leve strane jednacine je stvarno grozan.Treba probati nekim drugim pristupom doci do resenja,ako to uopste i moze!?

...Mora da moze.Zadatak je preuzet odavde http://www.truthtree.com/mathdoor.html ,12. zadatak je u pitanju.Ni za jedan zadatak sa ovog sajta nije zahtevao neko posebno znanje iz matematike, tako da verujem da je u pitanju samo pronaci pravi pristup.Inace, meni je ostalo da resim jos 4 zadatka odavde, medju kojima je i onaj sa sedam tegova.Dugo sam se mucio s njima, pa kako nije islo, resih da ih podelim s vama.

0 za
0 protiv
raborg
2010-10-08
???

konj_AleksandraM napisa...

mc80 napisa...

raborg napisa...


A evo i jedan zadatak od mene:

  Izmedju dva vertikalna zida postavljene su dve letve o obliku slova X.Njihove duzine su 5 i 8 metara.Ako je vertikalno rastojanje izmedju tacke njihovog preseka i ravnog horizontalnog tla 3 metra, koliko je rastojanje izmedju zidova.

  Ne uspevam nikako da resim ovaj zadatak.Nekako se uvek zakucam u jednacinu cetvrtog stepena.

Da.I ja sam dobio jednacinu cetvrtog stepena,koja nema racionalnih resenja.Ako nisam pogresio u racunu(radio sam na brzinu usmeno) cini mi se da imaju 2 realne nule i to jedna izmedju 2 i 3,a druga izmedju 4 i 5.Polinom sa leve strane jednacine je stvarno grozan.Treba probati nekim drugim pristupom doci do resenja,ako to uopste i moze!?

...Samo da pitam, vjerovatno ste se sjetili da jednačinu četvrtog stepena metodom zamjene pretvorite u kvadratnu jednačinu.

...Ne moze zamena jer su brojevi nezgodni.

0 za
0 protiv
raborg
2010-10-08
???

Bogami momci,samo vas gledam i ocekujem da uskoro dokazete Goldbahovu hipotezu,alal vera sa koliko elana se upustate u resavanje...

0 za
0 protiv
NeparniValjak
2010-10-08
???

 

Vaskrsija napisa...

limes5 napisa...

 

mc80 napisa...

mc80 napisa...

Da li kocku mozemo podeliti na 101 manju kocku?
 

Da li je neko pokusasao da resi ovaj zadatak?On zahteva visok stepen dovitljivosti,nije bas lak,ali ga moze resavati i ucenik 4.razreda osnovne skole.

...Ja mislim da može.

Ako datu kocku izdjelimo tako da ona ima 64 dijela,dakle stranicu joj podjelimo na 4 jedinična dijela,iz nje možemo izdvojiti jednu kocku koja u sebi sadrži 27 tih malih "jediničnih kocki" ,tj. kocku čija je stranica duga 3 jedinična dijela.Dobijamo tu 1 veću kocku (koja je sastavljena iz 27 manjih) i preostalo je još 37 onih jediničnih kocki (64-27=37).Od tih 37 izaberemo 9 koje ćemo podjeliti svaku na po 8 još manjih.(Dakle taj jedinični dio dijelimo na pola).Od tih 9 "jediničnih kocki dobijamo 9*8=72 još manje kocke. Pored ovih 72 imamo još 28 koje su ostale kad smo od 37 oduzeli 9 i još onu najveću,što ukupno daje 72+28+1=101.

...Svaka čast limes5

Nije lose, al bi mozda najlakse bilo, podijelit je na 64 manje kocke, zatim uzeti jednu i nju podijeliti na 64 manje kocke i imacemo 63+64=127, zatim uzmemo 27 i od nje napravimo jednu i eto ti 127-27+1=101.

... Naravno,ima više načina kako stići do broja 101.Ovo je prvo rješenje do koga sam došao.Vidio sam i još par mogućnosti, ali nisu bile jednostavnije od ove,pa sam prvo rješenje i "prezentovao".

A što se tiče tvog rješenja...pravo elegantno.Bravo i za tebe!

1 za
0 protiv
limes5
2010-10-08
???

Košarkaški tim čine 5 bekova, 4  centra  i  3  krila.
Na koliko se načina od njih može sastaviti petorka ako u njoj moraju da igraju bar 2 beka i bar 1 centar?

0 za
0 protiv
MaLin
2010-10-16
???

Vaskrsija napisa...

Košarkaški tim čine 5 bekova, 4  centra  i  3  krila.
Na koliko se načina od njih može sastaviti petorka ako u njoj moraju da igraju bar 2 beka i bar 1 centar?

...540

2 za
0 protiv
tiii
2010-10-17
???

tiii napisa...

Vaskrsija napisa...

Košarkaški tim čine 5 bekova, 4  centra  i  3  krila.
Na koliko se načina od njih može sastaviti petorka ako u njoj moraju da igraju bar 2 beka i bar 1 centar?

...540

...Tačno

1 za
0 protiv
MaLin
2010-10-17
???

raborg napisa...

   Kolona vojnika dugačka jedan kilometar počinje da maršira ravnomernom brzinom.Posle tačno jednog kilometra zaustavljaju se.U istom trenutku kad je kolona krenula, glasnik koji se nalazi na samom začelju, izdvaja se i počinje da se kreće ka početku, takodje ravnomernom brzinom.Njegova brzina je taman takva da uspe da stigne sam početak kolone , istovremeno se okrene i nastavi nazad ka začelju, gde stigne baš u trenutku kada se i sama kolona zaustavila.Celim putem, brzine kolone kao ni glasnika nisu se menjale.Pitanje glasi: -Koliko je glasnik prepešačio?

    Nadam se da će malo mučenja oko ovog zadatka mnogima doneti zadovoljstvo, kao i da će zadataka biti još.

...1 km

1 za
0 protiv
Zoran Zdrnja
2010-12-02
???

3km

0 za
0 protiv
Ljiljana Ilic Ex Zdravkovic
2010-12-04
???

MaLin napisa...

Caka je u tome , što se ne možeš osloniti samo na čulo vida, tj. moraš koristiti i čulo dodira.

Evo kao, pritisneš jedan prekidač a zatim sačekaš nekoliko minuta, a zatim pritisneš drugi prekidač i uđeš u sobu, zatim dodirneš uključene sijalice, ona koja je topla, nju je uključio prvi prekidač, a ona koja je hladna, nju je uključio drugi prekidač, ona što nije uključena reguliše se trećim prekidačem.

...a ako su stedljive pa se ne zagrevaju?

0 za
0 protiv
Angie
2010-12-13
???

Angie napisa...

...a ako su stedljive pa se ne zagrevaju?

...Biće da takve nisu bile išćitane kad su smislili oti zadatak, pa šta znam, pratiti stanje na brojilu i izračunati koja je više uštedjela na kraju mjeseca, čisto sumnjam da bi nam čulo mirisa moglo pomoći.

0 za
1 protiv
MaLin
2010-12-13
???

raborg napisa...

   Kolona vojnika dugačka jedan kilometar počinje da maršira ravnomernom brzinom.Posle tačno jednog kilometra zaustavljaju se.U istom trenutku kad je kolona krenula, glasnik koji se nalazi na samom začelju, izdvaja se i počinje da se kreće ka početku, takodje ravnomernom brzinom.Njegova brzina je taman takva da uspe da stigne sam početak kolone , istovremeno se okrene i nastavi nazad ka začelju, gde stigne baš u trenutku kada se i sama kolona zaustavila.Celim putem, brzine kolone kao ni glasnika nisu se menjale.Pitanje glasi: -Koliko je glasnik prepešačio?

    Nadam se da će malo mučenja oko ovog zadatka mnogima doneti zadovoljstvo, kao i da će zadataka biti još.

...raborg napisa...

   Kolona vojnika dugačka jedan kilometar počinje da maršira ravnomernom brzinom.Posle tačno jednog kilometra zaustavljaju se.U istom trenutku kad je kolona krenula, glasnik koji se nalazi na samom začelju, izdvaja se i počinje da se kreće ka početku, takodje ravnomernom brzinom.Njegova brzina je taman takva da uspe da stigne sam početak kolone , istovremeno se okrene i nastavi nazad ka začelju, gde stigne baš u trenutku kada se i sama kolona zaustavila.Celim putem, brzine kolone kao ni glasnika nisu se menjale.Pitanje glasi: -Koliko je glasnik prepešačio?

    Nadam se da će malo mučenja oko ovog zadatka mnogima doneti zadovoljstvo, kao i da će zadataka biti još.

...pola kilometra

1 za
1 protiv
vladan radovic
2011-01-12
???

Evo jednog kojeg sam procitao u jednom udzbeniku: date su terazije (vaga) i 12 naizgled identicnih loptica. 11 su iste tezine, a jedna se razlikuje po tezini - ne znamo da li je laksa ili teza. Kuglice su numerisane. Treba odrediti iz samo tri merenja koja se razlikuje po tezini i kako.

2 za
0 protiv
Dakinsljam
2011-01-20
???

Ako su kuglice obilježene sa 1,2,3,4,5,6,7,8,9,10,11,12, u PRVOM MJERENJU upoređujemo
1,2,3,4 sa 5,6,7,8 i ako ni jedna ne preteže upoređujemo u DRUGOM MJERENJU 9,10 sa 11,12.
Ako su ponovo u ravnoteži u TREĆEM MJERENJU upoređujemo 12 sa 1 i ako preteže 12 onda je ona lažna i teža, a ako preteže 1, onda je 12 lažna i lakša.

Ako je u DRUGOM MJERENJU preteže 9,10 u TREĆEM MJERENJU upoređujemo 9,11 sa 1,2.
Ako su jednake lažna i teža je 10, ako preteže 9,11 lažna i teža je 9, ako su 9,11 lakše od 1,2  onda je je 11 lažna i lakša.

Pretpostavimo da su u PRVOM MJERENJU pretegle 1,2,3,4 u DRUGOM MJERENJU upoređujemo 1,2,5 sa 3,4,9.
Ukoliko su jednake u TREĆEM MJERENJU upoređujemo 6 sa 7.
Ako su jednake lažna i lakša je 8, inače lažna je ona koja je lakša.
Ako u DRUGOM MJERENJU pretegne 1,2,5 onda u TREĆEM MJERENJU upoređujemo 1 sa 9 i ako 1 pretegne, onda je ona lažna i teža od ostalih, dok je u slučaju ravnoteže 2 lažna i teža.
( u ovom mjerenje ne može 9 pretegnuti)
 

I preostao je slučaj kada je 1,2,5 lakše od 2,4,9 tada u TREĆEM MJERENJU upoređujemo 4,5 sa 9,10.
Ako je 4,5 lakše onda je lažna i lakša 5, ako je teže onda je lažna i teža 4, dok je u slučaju ravnoteže lažna i teža kuglica 3.

 

Eto braćo draga ako je vama ovdje išta jasno, baš mi je drago, pošto meni aba baš ništa nije, a rješenje sam prepiso samo neću nikom da kažem, jer hoću da vi pomislite kako sam ja pametan.
U svakom slučaju nisam loš za jednog konja.
Zdravo i budite mi dobri i veseli, i dosta je braćo više svađe.

 

2 za
1 protiv
konj_AleksandraM
2011-01-20
???

  Bukefalu, dobro ti je resenje, samo sto se u prvom slucaju kada su 1,2,3,4 jednaki s 5, 6,7,8 , u drugom merenju uporedjuju 9,10 sa 11 i npr. 1, a ne sa 11,12.

1 za
0 protiv
raborg
2011-01-20
???

Mislim da osim ovoga sto ti je raborg zamerio nemas ni slucaj 10-laksa, mozda i jos nesto...

0 za
0 protiv
Dakinsljam
2011-01-21
???

Srela se dva prijatelja, koja se nisu vidjeli poduže vrijeme, i započeli razgovor.

-Zdravo, kako si?

-Dobro sam, kako si ti, koliko djece imaš?

-Imam tri kćerke.

-Koliko imaju godina?

-Proizvod njihovih godina je 36 a zbir je jednak broju one kuće tamo?

-Žao mi je, nisi mi dovoljno rekao!

-Da, u pravu si, najstarija kćerka svira klavir.

Koliko goidna imaju kćerke?

2 za
0 protiv
cicvara
2011-01-21
???

raborg napisa...

   Kolona vojnika dugačka jedan kilometar počinje da maršira ravnomernom brzinom.Posle tačno jednog kilometra zaustavljaju se.U istom trenutku kad je kolona krenula, glasnik koji se nalazi na samom začelju, izdvaja se i počinje da se kreće ka početku, takodje ravnomernom brzinom.Njegova brzina je taman takva da uspe da stigne sam početak kolone , istovremeno se okrene i nastavi nazad ka začelju, gde stigne baš u trenutku kada se i sama kolona zaustavila.Celim putem, brzine kolone kao ni glasnika nisu se menjale.Pitanje glasi: -Koliko je glasnik prepešačio?

    Nadam se da će malo mučenja oko ovog zadatka mnogima doneti zadovoljstvo, kao i da će zadataka biti još.

...Tri kilometra

0 za
0 protiv
Dragana Malesevic Adzic
2011-01-21
???

Dragana Malesevic Adzic napisa...

raborg napisa...

   Kolona vojnika dugačka jedan kilometar počinje da maršira ravnomernom brzinom.Posle tačno jednog kilometra zaustavljaju se.U istom trenutku kad je kolona krenula, glasnik koji se nalazi na samom začelju, izdvaja se i počinje da se kreće ka početku, takodje ravnomernom brzinom.Njegova brzina je taman takva da uspe da stigne sam početak kolone , istovremeno se okrene i nastavi nazad ka začelju, gde stigne baš u trenutku kada se i sama kolona zaustavila.Celim putem, brzine kolone kao ni glasnika nisu se menjale.Pitanje glasi: -Koliko je glasnik prepešačio?

    Nadam se da će malo mučenja oko ovog zadatka mnogima doneti zadovoljstvo, kao i da će zadataka biti još.

...Tri kilometra

...Imas gore resenje, medju prvim postovima.

0 za
0 protiv
raborg
2011-01-21
???

cicvara napisa...

Srela se dva prijatelja, koja se nisu vidjeli poduže vrijeme, i započeli razgovor.

-Zdravo, kako si?

-Dobro sam, kako si ti, koliko djece imaš?

-Imam tri kćerke.

-Koliko imaju godina?

-Proizvod njihovih godina je 36 a zbir je jednak broju one kuće tamo?

-Žao mi je, nisi mi dovoljno rekao!

-Da, u pravu si, najstarija kćerka svira klavir.

Koliko goidna imaju kćerke?

...

Njihove su godine 6,3, i 2. Nijedna druga kombinacija ne daje isti proizvod.

1 za
0 protiv
Dabić Rašo
2011-01-22
???

Dabić Rašo napisa...

cicvara napisa...

Srela se dva prijatelja, koja se nisu vidjeli poduže vrijeme, i započeli razgovor.

-Zdravo, kako si?

-Dobro sam, kako si ti, koliko djece imaš?

-Imam tri kćerke.

-Koliko imaju godina?

-Proizvod njihovih godina je 36 a zbir je jednak broju one kuće tamo?

-Žao mi je, nisi mi dovoljno rekao!

-Da, u pravu si, najstarija kćerka svira klavir.

Koliko goidna imaju kćerke?

...

Njihove su godine 6,3, i 2. Nijedna druga kombinacija ne daje isti proizvod.

...Nijedna druga, pa neće biti
1,2,18 ili 1,4,9, ili 1,6,6,1 ili 3,3,4 pa čak i ova 1,1,36 iako ne bi bilo realno da najstarija kćerka ima 36 godina, dok malđe sestre imaju po jednu, ali svašta je moguće...

1 za
0 protiv
cicvara
2011-01-22
???

cicvara napisa...

Dabić Rašo napisa...

cicvara napisa...

Srela se dva prijatelja, koja se nisu vidjeli poduže vrijeme, i započeli razgovor.

-Zdravo, kako si?

-Dobro sam, kako si ti, koliko djece imaš?

-Imam tri kćerke.

-Koliko imaju godina?

-Proizvod njihovih godina je 36 a zbir je jednak broju one kuće tamo?

-Žao mi je, nisi mi dovoljno rekao!

-Da, u pravu si, najstarija kćerka svira klavir.

Koliko goidna imaju kćerke?

...

Njihove su godine 6,3, i 2. Nijedna druga kombinacija ne daje isti proizvod.

...Nijedna druga, pa neće biti
1,2,18 ili 1,4,9, ili 1,6,6,1 ili 3,3,4 pa čak i ova 1,1,36 iako ne bi bilo realno da najstarija kćerka ima 36 godina, dok malđe sestre imaju po jednu, ali svašta je moguće...

... moze i 2,2,9 samo to verovatno nije jer je zbir 13, a verovatno je zadatak tako zamisljen da kuce nemaju broj 13. Sad ostaje samo pitanje da li sa 4 godine moze da se svida klavir...

0 za
0 protiv
Supervizor
2011-01-22
???

Можда је у питању клавир, али у пренесеном значењу, па би решење могло бити 1, 2, 18, јер се зна да  "клавир свирају" углавном пунолетни. Поздрав!

1 za
0 protiv
brba
2011-01-22
???

Resenje je 9,2,2.

Stvar je u tome sto je zbir isti kod 6,6,1 i 9,2,2 (13), pa prema tome podatak o broju kuce nije dovoljan. Posto najstarija svira klavir, prva kombinacija otpada jer dve starije imaju isti broj godina.

1 za
0 protiv
Slobodan
2011-01-22
???

Slobodan napisa...

Resenje je 9,2,2.

Stvar je u tome sto je zbir isti kod 6,6,1 i 9,2,2 (13), pa prema tome podatak o broju kuce nije dovoljan. Posto najstarija svira klavir, prva kombinacija otpada jer dve starije imaju isti broj godina.

...Tako je, bravo
 

A sad novi zadatak

 Osam kugli
Slavni majstor logičkih veština je tri svoja učenika ( koji se zovu A, B i C) stavio na test. Uzeo je 8 kugli: 4 crvene i 4 zelene. Svakom je dao po dve kugle i za sebe je zadržao dve. Svako od učenika je mogao da vidi kugle svojih drugara ali nije mogao da vidi svoje dve kugle niti dve kugle koje se nalaze kod učitelja. Učitelj postavlja pitanje osobi A: "Da li znaš koje su boje tvoje dve kugle"? Ukoliko učenik A ne zna odgovor tada on isto to pitanje postavlja drugom učeniku i tako dalje. Odgovori učenika su sledeći:
A: "Ne."
B: "Ne."
C: "Ne."
A: "Ne."
B: "Da!!"
Koje su kugle kod osobe B? Koje su kod osoba A i C? Koje se kugle nalaze kod učitelja ?
3 za
0 protiv
cicvara
2011-01-22
???

Kod ucitelja i kod osobe B se nalaze po jedna crvena i jedna zelena kugla. Osoba A ima 2 crvene (ili 2 zelene), osoba C 2 zelene (ili 2 crvene)...

2 za
0 protiv
Slobodan
2011-01-22
???

@cicvara i slobodan

Jel' sam ja nesto propustio jer ne vidim da pise u zadatku da je broj kuce 13?

Inace, brbin odgvor mi je najmastovitiji © ® yes
 

1 za
0 protiv
Fellow
2011-01-22
???

Fellow napisa...

@cicvara i slobodan

Jel' sam ja nesto propustio jer ne vidim da pise u zadatku da je broj kuce 13?

Inace, brbin odgvor mi je najmastovitiji © ® yes
 

...Pa i ne piše, ali kako možemo zaključiti da je trinaest,

Ako mu je rekao da je proizvod 36 a zbir jednak broju kuće, to su dvije jednačine, koje bi trebalo da budu dovoljne za rješavanje zadatka, a on kaže "Žao mi je, nisi mi dovoljno rekao!" jer zbir 13 se može dobiti na dva načina, provjeri ostale sume, vidjećeš da se svaka od njih može dobit samo na jedan način, tj.  1,6,6 i 2,2,9, kako prva kombinacija otpada, to je rješenje druga.

1 za
0 protiv
cicvara
2011-01-22
???

cicvara napisa...

...Pa i ne piše, ali kako možemo zaključiti da je trinaest,

Ako mu je rekao da je proizvod 36 a zbir jednak broju kuće, to su dvije jednačine, koje bi trebalo da budu dovoljne za rješavanje zadatka, a on kaže "Žao mi je, nisi mi dovoljno rekao!" jer zbir 13 se može dobiti na dva načina, provjeri ostale sume, vidjećeš da se svaka od njih može dobit samo na jedan način, tj.  1,6,6 i 2,2,9, kako prva kombinacija otpada, to je rješenje druga.

...

Hvala na sugestiji, jasno je! yes

Razlaganje broja 36 na cinioce. Samo u dva slucaja  (1,6,6 i 2,2,9) zbir cinilaca daje isti broj (13) pa mu to nije bilo dovoljna informacija, a da je u pitanju neki drugi broj kuce (10,11,14,16,21,38) odmah bi mu bilo jasno. Kad je receno da postoji starija onda je bez sumnje 2,2,9.

Ako cemo da cepidlacimo mozda bi bilo tacno i 1,6,6 jer i dve bliznakinje koje imaju po 6 godina nisu rođene istog minuta (jedna mora biti malo starija).

P.S.

Dodatno me zbunio Supervizor sa

Supervizor napisa...


... moze i 2,2,9 samo to verovatno nije jer je zbir 13, a verovatno je zadatak tako zamisljen da kuce nemaju broj 13. Sad ostaje samo pitanje da li sa 4 godine moze da se svida klavir...

... cheeky koji je ocigledno znao resenje ali nije hteo da kvari zabavu...

2 za
0 protiv
Fellow
2011-01-22
???

http://www.matematiranje.com/Logicki problemi/novi_zanimljivi_problemi.pdf

Koga zanima,moze ae zanimati....

2 za
0 protiv
NeparniValjak
2011-01-22
???

Prvi zadatak: Moze. cak i macka moze. DObra fora

 

1 za
0 protiv
Dakinsljam
2011-01-22
???

Ovaj sa jaganjcima moslim da nije moguce. smiley

0 za
0 protiv
Dakinsljam
2011-01-22
???

Fellow napisa...

Ako cemo da cepidlacimo mozda bi bilo tacno i 1,6,6 jer i dve bliznakinje koje imaju po 6 godina nisu rođene istog minuta (jedna mora biti malo starija).

... to je u stvari problem ovog zadatka:

  • teoretski je moguće da ćerke imaju isto godina, jedna rođena u januaru, a druga u decembru na primer
  • a mogao se čovek razvesti i oženiti drugom dok mu je prva žena bila trudna, pa da ima 2 ćerke između kojih je recimo 4-5 meseci razlika, ali sa različitim ženama.
... cheeky koji je ocigledno znao resenje ali nije hteo da kvari zabavu...

... da, ali sasvim slučajno jer sam video sličan zadatak ranije na nekom drugom sajtu.

1 za
0 protiv
Supervizor
2011-01-22
???

6. zadatak. uključimo sat  što meri 3 minuta da meri jedan ceo krug i pola drugog, a onda uključimo onaj drugi što meri 5 minuta da meri pola kruga.

4. horizontalno drvce se pomeri za pola u desno, a donje vertikalno drvce se pomeri na desni kraj prethodno pomerenog drvceta (i bude  sa gornje strane).donje vertikalno desno drvce ostaje kao ručka.

Nadam se da je Neparni dao onaj link kao temu da to rešavamo(ako sam dobro razumeo). Pretpostavljam da je on to  sve već rešio. Ako je to trebao da radi svako za sebe, ne objavljuje rešenja na forumu, onda je moja greška.

Ova dva pitanja su mi bila logična na prvi pogled.

0 za
1 protiv
Radomir-ue
2011-01-22
???

NeparniValjak napisa...

http://www.matematiranje.com/Logicki problemi/novi_zanimljivi_problemi.pdf

Koga zanima,moze ae zanimati....

Ovaj 11-ti je zanimljiv sto se moze saznati i iz dva postavljena pitanja

0 za
0 protiv
Dakinsljam
2011-01-22
???

rrak-ue napisa...

6. zadatak. uključimo sat  što meri 3 minuta da meri jedan ceo krug i pola drugog, a onda uključimo onaj drugi što meri 5 minuta da meri pola kruga.

zadatak se zove PEŠČANI SATOVI, tesko da tu ima ceo krug.....

0 za
0 protiv
Dakinsljam
2011-01-22
???

To sam tek posle primetio:mislio sam da je u pitanju sat koji nema brojeve i da je krug  kod jednog od 3 minuta a kod drugog 5 minuta.

0 za
0 protiv
Radomir-ue
2011-01-22
???

Evo sad jednog od resenja i sa peščanim satovima:

1. Pustimo istovremeno oba sata peščana sata da mere i onaj od 3 i onaj od 5

2.Po završetku prva tri minuta okrenemo onaj sat što meri 3 minuta da meri ponovo, a ona j od 5 pustimo da meri do kraja (ostalo mu je 2 minuta)

3. Kad izmeri onaj od 5 minuta svoja preostala dva minuta u satu od 3 minuta je ostalo minut peska da iscuri.Okrenemo onaj sat od 5 minuta.

4.Pustimo  da mere ostatak od jednog minuta  oba sata (čekamo dok kod onog od 3 minuta ne istekne predviđeni minut).

5. Po isteku tog jednog minuta (kod onog od 3 minuta pratimo ) okrenemo onaj sat što meri 5 minuta i kad istekne taj minut  to je to.

Nadam se da se može zanemariti vreme okretanja.

Možda neko ima drugo rešenje ili ideju?

Hvala Dakinsljam-u na "primedbi".Pozdrav za tebe.

0 za
0 protiv
Radomir-ue
2011-01-22
???

Dakinsljam napisa...

Ovaj 11-ti je zanimljiv sto se moze saznati i iz dva postavljena pitanja

Na ovo molom vas ne obracajte paznju, bilo  je mnogo kasno angry

Mora iz tri pitanja

0 za
0 protiv
Dakinsljam
2011-01-23
???

rrak-ue napisa...

Evo sad jednog od resenja i sa peščanim satovima:

1. Pustimo istovremeno oba sata peščana sata da mere i onaj od 3 i onaj od 5

2.Po završetku prva tri minuta okrenemo onaj sat što meri 3 minuta da meri ponovo, a ona j od 5 pustimo da meri do kraja (ostalo mu je 2 minuta)

3. Kad izmeri onaj od 5 minuta svoja preostala dva minuta u satu od 3 minuta je ostalo minut peska da iscuri.Okrenemo onaj sat od 5 minuta.

4.Pustimo  da mere ostatak od jednog minuta  oba sata (čekamo dok kod onog od 3 minuta ne istekne predviđeni minut).

5. Po isteku tog jednog minuta (kod onog od 3 minuta pratimo ) okrenemo onaj sat što meri 5 minuta i kad istekne taj minut  to je to.

Nadam se da se može zanemariti vreme okretanja.

Možda neko ima drugo rešenje ili ideju?

Hvala Dakinsljam-u na "primedbi".Pozdrav za tebe.

Sad vidim da sam i taj netacno resio sinoc, dobro je pa nisam napisao nista ovde sad....

0 za
1 protiv
Dakinsljam
2011-01-23
???

U sobi se nalaze tri sijalice, a na zidu izvan sobe tri prekidača. Svaki od tih prekidača je povezan sa tačno jednom sijalicom. Soba ima samo jedna vrata koja su zatvorena i nema prozora.  U početku su sve tri sijalice ugašene, a sva tri prekidača okrenuta na dole. Vaš zadatak je da  samo jednim ulaskom u sobu odredite koji prekidač odgovara kojoj sijalici.

Izvinjavam se ako je neko već napisao ovaj ili sličan zadatak, jer ih nisam sve pročitao.

POZDRAV SVIMA I PIŠITE JOŠ DOBRIH ZADATAKA!

1 za
4 protiv
BODA TAJSON, DOŽIVOTNA
2011-02-04
???

BODA TAJSON, DOZIVOTNA napisa...

U sobi se nalaze tri sijalice, a na zidu izvan sobe tri prekidača. Svaki od tih prekidača je povezan sa tačno jednom sijalicom. Soba ima samo jedna vrata koja su zatvorena i nema prozora.  U početku su sve tri sijalice ugašene, a sva tri prekidača okrenuta na dole. Vaš zadatak je da  samo jednim ulaskom u sobu odredite koji prekidač odgovara kojoj sijalici.

Izvinjavam se ako je neko već napisao ovaj ili sličan zadatak, jer ih nisam sve pročitao.

POZDRAV SVIMA I PIŠITE JOŠ DOBRIH ZADATAKA!

...Ukljucis jedan prekidac i sacekas malo onda ukljucis drugi prekidac i udjes u sobu i pipnes sijalice, ona koja je najtoplija je od prvog prekidaca, ona manje toplija je od drugog a ona treca sijalica je hladna jer treci prekidac nije ukljucen ...

2 za
0 protiv
dejan86
2012-02-15
???

Evo jedne zagonetke:

Jedan ćovek u selu bio je najčudniji. Imao je neobične životinje, živeo je u neobičnoj kući, davao je predmetima neobična imena i neobično izgledao. Jednom je odlučio da krene na put u svom čamcu.   Krenuo je u subotu i vratio se za tačno četiri dana. To nebi ništa bilo čudno da se nije vratio OPET U SUBOTU!!!  To može samo on. Ali kako? smiley

7 za
0 protiv
Goga T
2012-02-15
???

  Mozda se  selo zove Subota.

9 za
0 protiv
raborg
2012-02-15
???

Može i to, ali postoji i drugi odgovor. Pogledajte početak zagonetke.

5 za
0 protiv
Goga T
2012-02-15
???

A možda je čamac zvao SUBOT?wink

7 za
0 protiv
Čika Paja
2012-02-15
???

Bravo Čika Pajo! smiley

5 za
0 protiv
Goga T
2012-02-15
???

mc80 napisa...

raborg napisa...

mc80 napisa...

Naravno!Siguran sam da si onda dobro uradio cim me to pitas,eto samo jos postavi resenje! :)

...Prvo se dogovore da onaj koji (na njegovu zalost ) bude na zacelju prebroji npr. crne sesire i ako broj bude neparan da kaze "crni" u suprotnom "beli".On ce imati 50% da prezivi.Zatim onaj ispred njega takodje prebroji crne sesire medju preostalih 98 i ako im je broj i dalje neparan znaci da je na njegovoj glavi beli, sto ce i reci, u suprotnom ce reci "crni".Svaki sledeci, pazljivo saslusavsi sta izgovara prethodnik i koristeci istu taktiku, ce se izvuci.

Da,da to je tacno!Svaka cast!

...lose si postavio pitanje jer u njemu ne stoji da je isti broj crnih i bijelih kapa moglo je bit 3 crne i 97 bijelih ili 46 bijelih i 54 crne ili bilo kako kako onda oni da znaju koja je dje nikako

0 za
2 protiv
Djordje Raicevic
2012-02-18
???

Svejedno je koliko ima kojih sesira, bitna je samo taktika.Razmisli malo.

2 za
0 protiv
raborg
2012-02-18
???

sta bre

1 za
2 protiv
klasicna budala
2012-02-24
???

dejan86 napisa...

BODA TAJSON, DOZIVOTNA napisa...

U sobi se nalaze tri sijalice, a na zidu izvan sobe tri prekidača. Svaki od tih prekidača je povezan sa tačno jednom sijalicom. Soba ima samo jedna vrata koja su zatvorena i nema prozora.  U početku su sve tri sijalice ugašene, a sva tri prekidača okrenuta na dole. Vaš zadatak je da  samo jednim ulaskom u sobu odredite koji prekidač odgovara kojoj sijalici.

Izvinjavam se ako je neko već napisao ovaj ili sličan zadatak, jer ih nisam sve pročitao.

POZDRAV SVIMA I PIŠITE JOŠ DOBRIH ZADATAKA!

...Ukljucis jedan prekidac i sacekas malo onda ukljucis drugi prekidac i udjes u sobu i pipnes sijalice, ona koja je najtoplija je od prvog prekidaca, ona manje toplija je od drugog a ona treca sijalica je hladna jer treci prekidac nije ukljucen ...

...Tako je, bravo ako si sam dosao do rešenja.

Ovo je bilo trik pitanje. Njih inače nmanje cenim nego klasične zadatke koji traže znanje i logično rasuđivanje. 

Jedan takav zadatak sam postavio u drugoj temi, pa ko sam reši, avaka čast.

7 za
0 protiv
BODA TAJSON, DOŽIVOTNA
2012-02-24
???

Čovek je doneo kući dvadeset životinja jednakih po vrsti, polu, obliku, dimenzijama. Posle nekog vremena ustanovio je je da je jedna vrednija od ostalih devetnaest. Koja životinja je u pitanju?

2 za
3 protiv
Daorniam
2012-03-09
???

 

- Koliko rešenja ima jednačina, tj. koliko vrednosti može imati x u jednačini  sin ( pi / (1+x^2) ) = 0 ?

 
2 za
2 protiv
Paun93
2012-03-17
???

Paun93 napisa...

 

- Koliko rešenja ima jednačina, tj. koliko vrednosti može imati x u jednačini  sin ( pi / (1+x^2) ) = 0 ?

 

...Jedno, x=0

1 za
2 protiv
raborg
2012-03-18
???

raborg napisa...

Paun93 napisa...

 

- Koliko rešenja ima jednačina, tj. koliko vrednosti može imati x u jednačini  sin ( pi / (1+x^2) ) = 0 ?

 

...Jedno, x=0

...

0 za
1 protiv
petrovicandjelka1
2012-03-22
???

 Ja mislim 4, dva kad je pi/ (1+x2) jednako o i 2 kad je pi/ (1+x2) jednako 180 stepeni tj. ne znam koliko radijana 2 pi valjda, ne drzite me za rec

0 za
3 protiv
petrovicandjelka1
2012-03-22
???

Ali se to svodi na jedno resenje  x jednako o , sorry nisam obratila paznju.Jer  pi/ (1+x2) ne moze nikad da bude o.

2 za
1 protiv
petrovicandjelka1
2012-03-22
???

Deca predškolskog uzrasta ga rešavaju za 5-10 minuta, programeri za sat vremena,  fakultetski obrazovani ljudi... pa, procenite sami.
 

5 za
1 protiv
Supervizor
2012-03-28
???

???=2, jer osmica ima 2 kruga.

10 za
0 protiv
raborg
2012-03-30
???

raborg napisa...

???=2, jer osmica ima 2 kruga.

... Bravo!

5 za
1 protiv
Supervizor
2012-03-30
???

Supervizor napisa...

raborg napisa...

???=2, jer osmica ima 2 kruga.

... Bravo!

...

Daorniam napisa...

Čovek je doneo kući dvadeset životinja jednakih po vrsti, polu, obliku, dimenzijama. Posle nekog vremena ustanovio je je da je jedna vrednija od ostalih devetnaest. Koja životinja je u pitanju?

Drug kaže školjka, a ja tipujem na neku pticu, koja u glavi ima veći repertoar pesama od ostallih, koje je ovaj čova doneo kući. Inače, ni drug ni ja ne znamo šta je rešenje, pa bi bilo lepo da ga ovde objaviš, a ukoliko želiš da ostali forumaši i dalje razmišljaju o tvom zadatku, odgovor mi pošalji na mejl "skolomir@yahoo.com".

P.S. Da u pitanju ne postoji četvrti uslov  "jednakih po dimenzijama", bio bih siguran da je "političar" tačan odgovor. Doduše, tada verovatno ne bi bila tačna konstatacija "da je jedna vrednija od ostalih devetnaest".

2 za
0 protiv
Dejan Milojkovic
2012-08-15
???

A ja bih rekao da je u pitanju jedina "noseća" ženka od njih dvadeset, ali koja vrsta životinje je u pitanju "ne bih znao kasti".

1 za
0 protiv
Čika Paja
2012-08-16
???

I meni je palo na pamet da bi mogla da bude noseća ženka neke životinje, ali tu sam mogućnost isključio iz prostog razloga - to bi značilo da bi bilo koja životinja mogla da bude u pitanju, a takvo rešenje je nemaštovito i suviše neodređeno. Iz tog razloga više tipujem na osobine ili svojstva pojedine vrste, umesto na stanje.

1 za
0 protiv
Dejan Milojkovic
2012-08-17
???

Mozda je doneo gusenice, pa se jedna razvila u leptira. Leptir je lepsi od gusenice, pa samim tim i vredniji.

1 za
0 protiv
onshort
2012-08-18
???

Možda je reč o golubovima letačima od kojih je jedan imao prsten od nekog plemenitog metala....

2 za
1 protiv
lunali
2012-08-19
???

Pošto se Daorniam još nije oglasio sa odgovorom (a pitanje je da li će), u međuvremenu sam da drugim sajtovima pronašao odgovor i on glasi

"biserna školjka".

 

A sada, evo jednog mog zadatka.

           U svakodnevnom govoru, kad god nam neko ponudi nešto povoljno, neki od nas će postaviti pitanje: "A u čemu je kvaka?" Pretpostavljam da ovaj izraz vodi poreklo od istoimenog komičnog romana "Kvaka 22", a pitanje koje sledi, prati radnju knjige, ali odgovor na njega niti je u knjizi, niti u filmu koji je prema njoj snimljen.

                                                          

                                                          KVAKA 22

           Ratni piloti se nadaju da će nakon uspešno obavljene misije moći da odu kući na odsustvo, ali nadređeni im stalno povećava broj misija. Kada, recimo, nakon poslednje misije treba da odu kući, nadređeni im kaže:

            - Drugovi piloti, uspešno ste obavili ovih deset misija, a sada treba da obavite još deset, pa kada i njih obavite, možete da idete kući.

            Problem je u tome što se ovo stanje ponavlja.

            Piloti iz jednostavnog razloga žele da odu kući - nikada ne znaju da li će da se vrate sa misije na koju upravo kreću.

            Pravila su sledeća:

            Prema statutu komisije, samo ludaci mogu da odu kući.

            Prvi način da pilot ode kući, jeste da se pozove na ludilo. Međutim, ukoliko on kaže komisiji "ja sam lud, pustite me kući", komisija će odbiti njegov zahtev, zato što zna da nijedan ludak to ne bi rekao za sebe (jer ludaci ne misle da su ludi). Dakle, nema glume pred komisijom i pozivanje na ludilo.

            Drugi način da se ode kući, jeste da komisija poštuje pravilo statuta koje glasi:        "Ukoliko komisija utvrdi da je neko od pilota lud, dužna je da ga ga razreši daljeg leta i pusti kući".

            Međutim, komisija nikada samoinicijativno ne utvrđuje da li je neki od pilota lud, pošto joj, očigldeno, ne ide u korist. Pošto to komisija ne radi, jedino što pilotima preostaje, jeste da se sami pozovu na ludilo, ali to nas vraća na prvi način odlaska kući.

           

            Pitanje: Šta pilot treba da kaže komisiji, da bi ga ona pustila kući?

           

            Mala pomoć: Da preformulišem pitanje: Kako da pilot pred komisijom dokaže da je lud, a da ona ne može da zna da li dotični glumi ili ne?

1 za
4 protiv
Dejan Milojkovic
2012-09-25
???

Da trazi da ide u misiju. Time razbija koncept uobicajenog ponasanja i komisija mora da utvrdjuje da li je lud. Odatle je lako. :D

3 za
0 protiv
Dragana-Goran Kalinic
2012-09-28
???

Dragana-Goran Kalinic napisa...

Da trazi da ide u misiju. Time razbija koncept uobicajenog ponasanja i komisija mora da utvrdjuje da li je lud. Odatle je lako. :D

Pilote niko ne pita da li hoće da idu u misiju, oni mora da ide u misiju, to im je posao. Kada bi mogli da biraju da li će da idu u misiju, mogli bi da izaberu i da ne idu, tako da u tom slučaju problem ne ne bi ni postojao - niti bi se plašili da neće da se vrate sa misije, pošto nisu ni otišli, niti bi komisiji morali da dokazuju da su ludi. Tvoj odgovor nije tačan, ali bio si blizu. Saćekaću još neki komentar, pa ću da okačim rešenje, a ako si nestrpljiv, kontaktiraj me na skolomir@yahoo.com.

1 za
1 protiv
Dejan Milojkovic
2012-09-29
???

raborg napisa...

   Kolona vojnika dugačka jedan kilometar počinje da maršira ravnomernom brzinom.Posle tačno jednog kilometra zaustavljaju se.U istom trenutku kad je kolona krenula, glasnik koji se nalazi na samom začelju, izdvaja se i počinje da se kreće ka početku, takodje ravnomernom brzinom.Njegova brzina je taman takva da uspe da stigne sam početak kolone , istovremeno se okrene i nastavi nazad ka začelju, gde stigne baš u trenutku kada se i sama kolona zaustavila.Celim putem, brzine kolone kao ni glasnika nisu se menjale.Pitanje glasi: -Koliko je glasnik prepešačio?

    Nadam se da će malo mučenja oko ovog zadatka mnogima doneti zadovoljstvo, kao i da će zadataka biti još.

...3 km

0 za
1 protiv
Bwangha Mihajlo Pribišević
2012-09-29
???

raborg napisa...

   Kolona vojnika dugačka jedan kilometar počinje da maršira ravnomernom brzinom.Posle tačno jednog kilometra zaustavljaju se.U istom trenutku kad je kolona krenula, glasnik koji se nalazi na samom začelju, izdvaja se i počinje da se kreće ka početku, takodje ravnomernom brzinom.Njegova brzina je taman takva da uspe da stigne sam početak kolone , istovremeno se okrene i nastavi nazad ka začelju, gde stigne baš u trenutku kada se i sama kolona zaustavila.Celim putem, brzine kolone kao ni glasnika nisu se menjale.Pitanje glasi: -Koliko je glasnik prepešačio?

    Nadam se da će malo mučenja oko ovog zadatka mnogima doneti zadovoljstvo, kao i da će zadataka biti još.

...3 km

0 za
1 protiv
Bwangha Mihajlo Pribišević
2012-09-29
???

raborg napisa...

   Kolona vojnika dugačka jedan kilometar počinje da maršira ravnomernom brzinom.Posle tačno jednog kilometra zaustavljaju se.U istom trenutku kad je kolona krenula, glasnik koji se nalazi na samom začelju, izdvaja se i počinje da se kreće ka početku, takodje ravnomernom brzinom.Njegova brzina je taman takva da uspe da stigne sam početak kolone , istovremeno se okrene i nastavi nazad ka začelju, gde stigne baš u trenutku kada se i sama kolona zaustavila.Celim putem, brzine kolone kao ni glasnika nisu se menjale.Pitanje glasi: -Koliko je glasnik prepešačio?

    Nadam se da će malo mučenja oko ovog zadatka mnogima doneti zadovoljstvo, kao i da će zadataka biti još.

...3 km

1 za
1 protiv
Bwangha Mihajlo Pribišević
2012-09-29
???

Pošalji odgovor

Samo prijavljeni korisnici sa 5000 ili više poena mogu da glasaju i da ostavljaju poruke.

 

Stranica kreirana za 0.0221 sec.

Copyright © 2024